MR GMAT IntegratedReasoning 6E

Download as pdf or txt
Download as pdf or txt
You are on page 1of 190

6 th

Edition

GMAT Integrated Reasoning Guide


Joern Meissner

TURBOCHARGE
GMAT and GMAT CAT are registered trademarks
YOUR PREP
of the Graduate Management Admission Council (GMAC).
GMAC does not endorse nor is it affiliated in any way with
the owner of this product or any content herein.

+1 (212) 316-2000 www.manhattanreview.com


Turbocharge your GMAT:
Integrated Reasoning Guide
part of the 6th Edition Series

 Complete & challenging training sets

 Over 50 representative IR practice


questions

 Great mix of Quant-based, Verbal-based,


CR-based, and Logic-based IR questions

 Strategy to secure a perfect 8 score

 Enlisting process of solving each question


type: TA, GI, Two-Part, and MSR

 Text-cum-graphic explanations: Multiple


column-wise sorted images of tables

 Emphasis on logical deduction approach

 Computer-enabled IR formats: Table


Analysis and Multi-Source Reasoning dealt
with no compromise

www.manhattanreview.com
19992016 Manhattan Review. All Rights Reserved.
ii Integrated Reasoning Guide

Copyright and Terms of Use

Copyright and Trademark


All materials herein (including names, terms, trademarks, designs, images, and graphics) are
the property of Manhattan Review, except where otherwise noted. Except as permitted herein,
no such material may be copied, reproduced, displayed or transmitted or otherwise used with-
out the prior written permission of Manhattan Review. You are permitted to use material
herein for your personal, noncommercial use, provided that you do not combine such material
into a combination, collection, or compilation of material. If you have any questions regarding
the use of the material, please contact Manhattan Review at [email protected].

This material may make reference to countries and persons. The use of such references is for
hypothetical and demonstrative purposes only.

Terms of Use
By using this material, you acknowledge and agree to the terms of use contained herein.

No Warranties
This material is provided without warranty, either express or implied, including the implied
warranties of merchantability, of fitness for a particular purpose and noninfringement. Man-
hattan Review does not warrant or make any representations regarding the use, accuracy or
results of the use of this material. This material may make reference to other source materials.
Manhattan Review is not responsible in any respect for the content of such other source ma-
terials, and disclaims all warranties and liabilities with respect to the other source materials.

Limitation on Liability
Manhattan Review shall not be responsible under any circumstances for any direct, indirect,
special, punitive, or consequential damages (Damages) that may arise from the use of this
material. In addition, Manhattan Review does not guarantee the accuracy or completeness of
its course materials, which are provided as is with no warranty, express or implied. Man-
hattan Review assumes no liability for any Damages from errors or omissions in the material,
whether arising in contract, tort or otherwise.

GMAT is a registered trademark of the Graduate Management Admission Council.


GMAC does not endorse, nor is it affiliated in any way with, the owner of this product or any
content herein.
10-Digit International Standard Book Number: (ISBN: 1-62926-070-3)
13-Digit International Standard Book Number: (ISBN: 978-1-62926-070-9)

Last updated on April 20th, 2016.

Manhattan Review, 275 Madison Avenue, Suite 1429, New York, NY 10016.
Phone: +1 (212) 316-2000. E-Mail: [email protected]. Web: www.manhattanreview.com

www.manhattanreview.com ii 19992016 Manhattan Review


Integrated Reasoning Guide iii

About the Turbocharge your GMAT Series

The Turbocharge Your GMAT Series is carefully designed to be clear, comprehensive, and
content-driven. Long regarded as the gold standard in GMAT prep worldwide, Manhattan Re-
views GMAT prep books offer professional GMAT instruction for dramatic score improvement.
Now in its updated 6th edition, the full series is designed to provide GMAT test-takers with
complete guidance for highly successful outcomes. As many students have discovered, Man-
hattan Reviews GMAT books break down the different test sections in a coherent, concise,
and accessible manner. We delve deeply into the content of every single testing area and zero
in on exactly what you need to know to raise your score. The full series is comprised of 16
guides that cover concepts in mathematics and grammar from the most basic through the
most advanced levels, making them a great study resource for all stages of GMAT preparation.
Students who work through all of our books benefit from a substantial boost to their GMAT
knowledge and develop a thorough and strategic approach to taking the GMAT.

 GMAT Math Essentials (ISBN: 978-1-62926-057-0)


 GMAT Number Properties Guide (ISBN: 978-1-62926-058-7)
 GMAT Arithmetics Guide (ISBN: 978-1-62926-059-4)
 GMAT Algebra Guide (ISBN: 978-1-62926-060-0)
 GMAT Geometry Guide (ISBN: 978-1-62926-061-7)
 GMAT Word Problems Guide (ISBN: 978-1-62926-062-4)
 GMAT Sets & Statistics Guide (ISBN: 978-1-62926-063-1)
 GMAT Combinatorics & Probability Guide (ISBN: 978-1-62926-064-8)
 GMAT Data Sufficiency Guide (ISBN: 978-1-62926-065-5)
 GMAT Quantitative Question Bank (ISBN: 978-1-62926-066-2)
 GMAT Sentence Correction Guide (ISBN: 978-1-62926-067-9)
 GMAT Critical Reasoning Guide (ISBN: 978-1-62926-068-6)
 GMAT Reading Comprehension Guide (ISBN: 978-1-62926-069-3)
 GMAT Integrated Reasoning Guide (ISBN: 978-1-62926-070-9)
 GMAT Analytical Writing Guide (ISBN: 978-1-62926-071-6)
 GMAT Vocabulary Builder (ISBN: 978-1-62926-072-3)

19992016 Manhattan Review iii www.manhattanreview.com


iv Integrated Reasoning Guide

About the Company


Manhattan Reviews origin can be traced directly back to an Ivy League MBA classroom in 1999.
While teaching advanced quantitative subjects to MBAs at Columbia Business School in New
York City, Professor Dr. Joern Meissner developed a reputation for explaining complicated
concepts in an understandable way. Remembering their own less-than-optimal experiences
preparing for the GMAT, Prof. Meissners students challenged him to assist their friends, who
were frustrated with conventional GMAT preparation options. In response, Prof. Meissner
created original lectures that focused on presenting GMAT content in a simplified and intel-
ligible manner, a method vastly different from the voluminous memorization and so-called
tricks commonly offered by others. The new approach immediately proved highly popular
with GMAT students, inspiring the birth of Manhattan Review.

Since its founding, Manhattan Review has grown into a multi-national educational services
firm, focusing on GMAT preparation, MBA admissions consulting, and application advisory
services, with thousands of highly satisfied students all over the world. The original lectures
have been continuously expanded and updated by the Manhattan Review team, an enthusiastic
group of master GMAT professionals and senior academics. Our team ensures that Manhattan
Review offers the most time-efficient and cost-effective preparation available for the GMAT.
Please visit www.ManhattanReview.com for further details.

About the Founder


Professor Dr. Joern Meissner has more than 25 years of teaching experience at the graduate
and undergraduate levels. He is the founder of Manhattan Review, a worldwide leader in test
prep services, and he created the original lectures for its first GMAT preparation class. Prof.
Meissner is a graduate of Columbia Business School in New York City, where he received a PhD
in Management Science. He has since served on the faculties of prestigious business schools
in the United Kingdom and Germany. He is a recognized authority in the areas of supply chain
management, logistics, and pricing strategy. Prof. Meissner thoroughly enjoys his research,
but he believes that grasping an idea is only half of the fun. Conveying knowledge to others
is even more fulfilling. This philosophy was crucial to the establishment of Manhattan Review,
and remains its most cherished principle.

www.manhattanreview.com iv 19992016 Manhattan Review


Integrated Reasoning Guide v

The Advantages of Using Manhattan Review


Time efficiency and cost effectiveness.

For most people, the most limiting factor of test preparation is time.
It takes significantly more teaching experience to prepare a student in less time.
Our test preparation approach is tailored for busy professionals. We will teach you
what you need to know in the least amount of time.

Our high-quality and dedicated instructors are committed to helping every student
reach her/his goals.

19992016 Manhattan Review v www.manhattanreview.com


vi Integrated Reasoning Guide

International Phone Numbers and Official Manhattan Review Websites


Manhattan Headquarters +1-212-316-2000 www.manhattanreview.com
USA & Canada +1-800-246-4600 www.manhattanreview.com
Argentina +1-212-316-2000 www.review.com.ar
Australia +61-3-9001-6618 www.manhattanreview.com
Austria +43-720-115-549 www.review.at
Belgium +32-2-808-5163 www.manhattanreview.be
Brazil +1-212-316-2000 www.manhattanreview.com.br
Chile +1-212-316-2000 www.manhattanreview.cl
China +86-20-2910-1913 www.manhattanreview.cn
Czech Republic +1-212-316-2000 www.review.cz
France +33-1-8488-4204 www.review.fr
Germany +49-89-3803-8856 www.review.de
Greece +1-212-316-2000 www.review.com.gr
Hong Kong +852-5808-2704 www.review.hk
Hungary +1-212-316-2000 www.review.co.hu
India +1-212-316-2000 www.review.in
Indonesia +1-212-316-2000 www.manhattanreview.id
Ireland +1-212-316-2000 www.gmat.ie
Italy +39-06-9338-7617 www.manhattanreview.it
Japan +81-3-4589-5125 www.manhattanreview.jp
Malaysia +1-212-316-2000 www.review.my
Mexico +1-212-316-2000 www.manhattanreview.mx
Netherlands +31-20-808-4399 www.manhattanreview.nl
New Zealand +1-212-316-2000 www.review.co.nz
Philippines +1-212-316-2000 www.review.ph
Poland +1-212-316-2000 www.review.pl
Portugal +1-212-316-2000 www.review.pt
Qatar +1-212-316-2000 www.review.qa
Russia +1-212-316-2000 www.manhattanreview.ru
Singapore +65-3158-2571 www.gmat.sg
South Africa +1-212-316-2000 www.manhattanreview.co.za
South Korea +1-212-316-2000 www.manhattanreview.kr
Sweden +1-212-316-2000 www.gmat.se
Spain +34-911-876-504 www.review.es
Switzerland +41-435-080-991 www.review.ch
Taiwan +1-212-316-2000 www.gmat.tw
Thailand +66-6-0003-5529 www.manhattanreview.com
Turkey +1-212-316-2000 www.review.com.tr
United Arab Emirates +1-212-316-2000 www.manhattanreview.ae
United Kingdom +44-20-7060-9800 www.manhattanreview.co.uk
Rest of World +1-212-316-2000 www.manhattanreview.com

www.manhattanreview.com vi 19992016 Manhattan Review


Contents

1 Introduction 1

2 What is Integrated Reasoning? 3


2.1 Integrated Reasoning in the GMAT . . . . . . . . . . . . . . . . . . . . . . . . . . . . . . 3
2.2 What does Integrated Reasoning measure? . . . . . . . . . . . . . . . . . . . . . . . . . 3
2.3 Types of IR Questions . . . . . . . . . . . . . . . . . . . . . . . . . . . . . . . . . . . . . . 4
2.4 What are the Different Parts of Integrated Reasoning? . . . . . . . . . . . . . . . . . . 5
2.4.1 Table Analysis . . . . . . . . . . . . . . . . . . . . . . . . . . . . . . . . . . . . . . 5
2.4.2 Graphics Interpretation . . . . . . . . . . . . . . . . . . . . . . . . . . . . . . . . 7
2.4.3 Two-Part Analysis . . . . . . . . . . . . . . . . . . . . . . . . . . . . . . . . . . . . 11
2.4.4 Multi-Source Reasoning . . . . . . . . . . . . . . . . . . . . . . . . . . . . . . . . 13
2.5 How is the Integrated Reasoning Section Scored? . . . . . . . . . . . . . . . . . . . . . 17

3 Table Analysis 19
3.1 Strategies and Concepts . . . . . . . . . . . . . . . . . . . . . . . . . . . . . . . . . . . . 19
3.2 Commonly Tested Concepts in Table Analysis . . . . . . . . . . . . . . . . . . . . . . 19
3.3 Process of Solving TA Questions . . . . . . . . . . . . . . . . . . . . . . . . . . . . . . . 23
3.4 Examples . . . . . . . . . . . . . . . . . . . . . . . . . . . . . . . . . . . . . . . . . . . . . . 24

4 Graphics Interpretation 43
4.1 Strategies and Concepts . . . . . . . . . . . . . . . . . . . . . . . . . . . . . . . . . . . . 43
4.1.1 Some Terminology . . . . . . . . . . . . . . . . . . . . . . . . . . . . . . . . . . . 43
4.1.2 How to Read a Chart . . . . . . . . . . . . . . . . . . . . . . . . . . . . . . . . . . 45
4.2 Quantitative Concepts Tested in the Graphics Interpretation . . . . . . . . . . . . . 46
4.3 Process of Solving GI Questions . . . . . . . . . . . . . . . . . . . . . . . . . . . . . . . 51
4.4 Examples . . . . . . . . . . . . . . . . . . . . . . . . . . . . . . . . . . . . . . . . . . . . . . 52

5 Two-Part Analysis 79
5.1 Strategies and Concepts . . . . . . . . . . . . . . . . . . . . . . . . . . . . . . . . . . . . 79
5.1.1 Two-Part Quants . . . . . . . . . . . . . . . . . . . . . . . . . . . . . . . . . . . . . 79
5.2 Process of Solving Two-Part Quant Questions . . . . . . . . . . . . . . . . . . . . . . . 79
5.2.1 Two-Part Logic Games . . . . . . . . . . . . . . . . . . . . . . . . . . . . . . . . . 80
5.2.2 Two-Part Critical Reasoning . . . . . . . . . . . . . . . . . . . . . . . . . . . . . . 80
5.3 Examples: Two-Part Quant-based Questions . . . . . . . . . . . . . . . . . . . . . . . . 81
5.4 Examples: Two-Part Verbal-based Questions . . . . . . . . . . . . . . . . . . . . . . . . 89

vii
viii Integrated Reasoning Guide

6 Multi-Source Reasoning 95
6.1 Strategies and Concepts . . . . . . . . . . . . . . . . . . . . . . . . . . . . . . . . . . . . 95
6.2 Process of Solving MSR Questions . . . . . . . . . . . . . . . . . . . . . . . . . . . . . . 96
6.3 Examples . . . . . . . . . . . . . . . . . . . . . . . . . . . . . . . . . . . . . . . . . . . . . . 97

7 Practice Questions 111


7.1 Table Analysis . . . . . . . . . . . . . . . . . . . . . . . . . . . . . . . . . . . . . . . . . . 112
7.2 Graphics Interpretation . . . . . . . . . . . . . . . . . . . . . . . . . . . . . . . . . . . . . 120
7.3 2-Part Analysis . . . . . . . . . . . . . . . . . . . . . . . . . . . . . . . . . . . . . . . . . . 130
7.4 Multi-Source Reasoning . . . . . . . . . . . . . . . . . . . . . . . . . . . . . . . . . . . . . 139

8 Answer Key 147

9 Solutions 149
9.1 Table Analysis . . . . . . . . . . . . . . . . . . . . . . . . . . . . . . . . . . . . . . . . . . 150
9.2 Graphics Interpretation . . . . . . . . . . . . . . . . . . . . . . . . . . . . . . . . . . . . . 160
9.3 2-Part Analysis . . . . . . . . . . . . . . . . . . . . . . . . . . . . . . . . . . . . . . . . . . 166
9.4 Multi-Source Reasoning . . . . . . . . . . . . . . . . . . . . . . . . . . . . . . . . . . . . . 172

10 Talk to Us 177

www.manhattanreview.com viii 19992016 Manhattan Review


Chapter 1

Introduction

Dear Students,

Here at Manhattan Review, we constantly strive to provide you the best educational content
for standardized test preparation. We make a tremendous effort to keep making things better
and better for you. This is especially important with respect to an examination such as the
GMAT: the typical GMAT aspirant is confused with so many test-prep options available. Your
challenge is to choose a book or a tutor that prepares you for attaining your goal. We cannot
not say that we are one of the best. It is you who has to be the judge of that.
The greatest challenge to any test-prep companies is how to present interactive contents in
text form. It was challenge for us, too. We hope you have seen IR books by other leading
test-prep companies; if not, please do look at them. While they have done a fairly good job
presenting content online, they have failed miserably at producing traditional textbooks.
Why did they fail?
Since the Table Analysis and Multi-Source Reasoning IR questions are interactive prompts in
the test, it is a challengeeven for the leading test-prep companiesto effectively present
and explain the content. The ones who dared come up with IR books have only provided a
few questions with no, or few, practice questions. Their textbooks are mere supplementary
material to their online content.
We are proud that we have met the challenge of presenting interactive prompts. This book
is a landmark, in that it presents computer-enabled, interactive content, enriched with never-
before-seen graphics.
The Manhattan Reviews Integrated Reasoning book is holistic and comprehensive in all re-
spects. Should you have any queries, please feel free to write to me at [email protected].

Happy Learning!

Prof. Dr. Joern Meissner


& The Manhattan Review Team

1
2 An introduction to Integrated Reasoning

www.manhattanreview.com 2 19992016 Manhattan Review


Chapter 2

What is Integrated Reasoning?

2.1 Integrated Reasoning in the GMAT

Over the years, the GMAT test has been an undisputed measure of test takers ability to show-
case the skills that matter most in the business-school classroom.
However, did the old GMAT tests still miss something?
It was great at predicting academic success in B-school; however, B-school teaching methodol-
ogy involves applying case studies. The old GMAT tested your quantitative and verbal ability,
but real-world data are not restricted to only quantitative and verbal data. In fact, complex
information can be presented in the form of a graph, text with quantitative information, data
in a tabular format, and text with verbal information. In a nutshell, to make a meaningful
business decision, one may need to integrate multiple types of information.
Critical Reasoning prompts test verbal reasoning, but seldom quantitative reasoning; its scope
is narrow. Integrated Reasoning prompts on the other hand, are more comprehensive.

2.2 What does Integrated Reasoning measure?

GMAC states that the Integrated Reasoning section measures your ability to comprehend and
evaluate multiple types of information: textual, tabular, graphic, visual, quantitative, and ver-
bal, that it applies quantitative and verbal reasoning to solve problems in relation to one
another. This section differs from the Quantitative and Verbal sections in two ways:

(1) IR comprises both quantitative and verbal reasoning, and

(2) IR prompts use four different response formats rather than only the traditional multiple-
choice format used for Quantitative and Verbal questions.

3
4 An introduction to Integrated Reasoning

The four types of questions used in the Integrated Reasoning section are:

(1) Table analysis

(2) Graphics interpretation

(3) Two-part analysis

(4) Multi-source reasoning

In a nutshell, the IR questions assess your ability to apply, infer, evaluate, recognize, and
strategize information from multiple sources.

2.3 Types of IR Questions

Apply Questions

Apply questions measure your ability to comprehend the concepts in the information given
and apply them to a new situation.

Evaluate and Inference Questions

Evaluate questions measure your ability to make decisions based on the quality of information.
These questions are very similar to Critical Reasoning questions, in which you are asked if the
information strengthens/weakens the argument, if there is sufficient information to make a
decision (some aspect of data sufficiency), plus inference and flaws in reasoning.

Recognize Questions

Recognize questions measure your ability to identify in the information some facts or aspects
and their relationships. You may be asked to recognize agreement and disagreement, strength
of correlation between two variables, compare two or more data points, or understand data
and deduce information.

Strategize Questions

Strategize questions test your ability to gain an objective within the constraints. You may be
asked to select a course of action that gives desired results, while optimizing resources.

www.manhattanreview.com 4 19992016 Manhattan Review


An introduction to Integrated Reasoning 5

2.4 What are the Different Parts of Integrated Reasoning?

As stated earlier that there are four parts to Integrated Reasoning: Table Analysis, Graphics
Interpretation, Two-Part Analysis, Multi-Source Reasoning.
Let us look at them one by one.

2.4.1 Table Analysis

The Table Analysis questions present a table similar to a spreadsheet. The table can be sorted
only in ascending order on any of its columns, by selecting the columns title from a drop-
down menu given above the table. A brief text explaining the table is usually provided. The
question presents three statements, values, or expressions. You may be asked whether each
statement is true or false, or if Yes or No can be inferred from the information.
You may be asked to calculate mean, median, or range, calculate ratios, proportions, or prob-
abilities, recognize correlations between two sets of data, compare a data in relation to other
data, or select a statement that best helps explain the data given in prompt.
As with MSR datasets, Table Analysis prompts are interactive.

19992016 Manhattan Review 5 www.manhattanreview.com


6 An introduction to Integrated Reasoning

A sample TA Question

The table presents the Average Maximum Score, Average Minimum Score, and the Intelligence
Quotient for grade X and XII students of City International School for six months.

Months
Col 1 Col 2 Col 3 Col 4 Col 5
Average Average
Intelligence
Months Grade maximum minimum
Quotient
Score Score
1 July X 71 55 115
1 July XII 72 45 116
2 August X 66 58 116
2 August XII 68 55 120
3 September X 72 52 121
3 September XII 75 56 114
4 October X 68 54 121
4 October XII 69 45 120
5 November X 81 62 119
5 November XII 69 51 118
6 December X 71 52 117
6 December XII 70 49 114

For each of the following statements, select Yes if the statement is true based on the infor-
mation provided; otherwise, select No.

Yes No

A Compared to grade XII students, grade X students are more con-


sistent with regards to the Intelligence Quotients attribute, taking
range as a parameter to measure the consistency.

B For any of the given months, the greatest deviation of the Av-
erage Maximum Score and Average Minimum Score for grade XII
students is more than that for grade X students.

C The mean average minimum score for August is more than that
for July and for December.

www.manhattanreview.com 6 19992016 Manhattan Review


An introduction to Integrated Reasoning 7

2.4.2 Graphics Interpretation

Graphics Interpretation questions present a chart, graph, diagram, or other visual form of
information. The chart is followed by two fill-In-the-blank type statements. You have to fill in
the blank by choosing the best option from a drop-down menu.

Graphics Interpretation questions include column charts, line graphs, pie charts, Venn dia-
grams, scatterplots, bubble graphs, flow charts, organization charts, and strategy maps.

Unlike Table Analysis and MSR prompts, Graphic Analysis prompts are static.

A Sample GI Question

The bar graph shows the number of unemployed youth in a country for seven years.

Unemployment
25000

20000
Unemployed

15000

10000

5000

0
2001 2002 2003 2004 2005 2006 2007

Based on the given information, use the drop-down menu to most accurately complete the
following statements:

A. The greatest increase in the number of unemployed youth from any year to the
succeeding year is approximately .
3736
4452
6108
7542
B. The number of unemployed youth in the year 2004 is approximately
16
percent of the number of unemployed youth in the year 2003.
19.5
84
119.5

19992016 Manhattan Review 7 www.manhattanreview.com


8 An introduction to Integrated Reasoning

Other Types of GI Charts/Diagrams

Multiple Column Chart

Commuting
450

400
Commuters (Million)

350

300

250

200

150

100

50

0
2005 2006 2007 2008 2009

Intra-City Bus Local Train Private Taxi

Pie chart

Undergraduate degree distribution

Art
Others
6%
12%
Science
12%

Humanities
13%

Engineering
Management 18%
12%

Commerce
27%

www.manhattanreview.com 8 19992016 Manhattan Review


An introduction to Integrated Reasoning 9

Bubble chart

Revenue to Profit Ratio


7
6 1.05

5 2.75
Years 4 3.15
3 1.26
2 2.85
1 1.85
0
1000 1500 2000 2500 3000
Profit ($K)

Scatter Plot

Age-wise number of patients


Age Vs. Disease X Age Vs. Disease Y
83 750

73 650
Number of disease X patients

Number of disease Y patients

63 550

53 450

43 350

33 250

23 150
12 27 42 57 72 87
Age

19992016 Manhattan Review 9 www.manhattanreview.com


10 An introduction to Integrated Reasoning

Strategy Map

Corporate Strategy Map

Customer
Strive for Improve
Facilitate
customer stake-holder
delight choices
goodwill
Financial

Improve price More cost Increase net


competitiveness control revenue

Improve customer
More control
administration
Processes

over quality
process

Improve Improve
safety of marketing &
plant sales
Development

Improve Improve
CRM
Learning &

knowledge of
knowledge customers

Improve Increase
knowledge of production
management capacity

www.manhattanreview.com 10 19992016 Manhattan Review


An introduction to Integrated Reasoning 11

2.4.3 Two-Part Analysis

Two-Part Analysis questions present a small passage or quantitative problem. There are two
correct choices related to the information given. The choices are given in a tabular format.
The possible answers are listed in the third column. You have to select one choice in each col-
umn. The question may test you on GMAT mathematical concepts, say, two aspects of a right-
circular cylinderHeight and Diameter. Similarly, for verbal concepts, it may ask questions
on strengthening/weakening the argument, inferable/not inferable premises, assumption/fact
distinction, existence of cause/effect, characteristics/predictions distinctions, or strategies ap-
plicable for, say, companies X and Y.
Unlike the Table Analysis and MSR prompts, but like Graphics Interpretation prompts, Two-
Part Analysis prompts are static.

Sample Two-Part Questions

A Quant-Based Two-Part Question:

For a right circular cylinder, r stands for the radius of its base, h for its height, A for its curved
surface area, and V for its volume. In terms of the variables A and V, select in the table the
expressions that represent height h and radius r .
Make only two selections, one in each column.

A B

r h

A2
A 4 V
2V
B A
A2 V
C 8
2A
D V
A
E V

19992016 Manhattan Review 11 www.manhattanreview.com


12 An introduction to Integrated Reasoning

A Verbal-Based Two-Part Question:

Consultant: The decline in market share of the Whito 3-kg detergent pack is a matter of
concern. There could be many predictive reasons. The competing brand, EcoWash, is always
on the lookout to gain Whitos substantial market share. Also, due to adverse judgments in
consumer courts against WhiteMagic detergentthe brand that kept EcoWashs detergent sales
in checkWhiteMagics sales are suffering, providing an opportunity for EcoWash. Usually,
customers prefer the detergent from the brand that already has an established soap cake
selling in the market.
Indicate in the table which cause-and-effect sequence would most likely, according to the con-
sultant, result in a decline of market share of Whito detergent.
Make two selections, one in each column.

A B

Cause Effect

A An increase in the market share of EcoWash soap cake

B An increase in the market share of EcoWash detergent

C An increase in the market share of WhiteMagic detergent

D A decrease in the market share of EcoWash soap cake

E A decrease in the market share of EcoWash detergent

www.manhattanreview.com 12 19992016 Manhattan Review


An introduction to Integrated Reasoning 13

2.4.4 Multi-Source Reasoning

As the name suggests, Multi-Source reasoning questions comprise information from more than
one source. An MSR question will have two or three tabs of information; however, you can view
only one tab at a time, by clicking it. The tab may contain a written passages, graphs, diagrams,
tables, or/and other types of visual information.
A typical MSR dataset will have three questions; however, you will find three to six questions
in Official Guide per dataset. There are two question formats for Multi-Source Reasoning:

(1) Multiple-choice questions

(2) Multiple-dichotomous choice questions

For multiple-choice questions, five answer choices given. You have to select the correct answer
choice. Multiple-dichotomous question presents three statements, values, or expressions. You
may be asked whether, according to the information, each statement is True/False, Agree-
able/Not Agreeable, Acceptable/Not Acceptable, Supported/Not Supported, or can be infer-
able/Not inferable. As with Table Analysis prompts, MSR prompts are interactive.

19992016 Manhattan Review 13 www.manhattanreview.com


14 An introduction to Integrated Reasoning

A sample MSR Question

Tab 1

Mike, Chief economist, on BRICS countries Professor Walter on BRICS countries

The notion that the era of emerging BRICS countries is overand that among them only
China would make it to the group of high-income countriesis outlandish. No doubt the
growth rates in the BRICS group of Brazil, Russia, India, China, and South Africa have been
affected by the global slowdown, and countries such as India have been further singed by
capital-flow reversals. However, this is a temporary phenomenon that will peter out sooner
rather than later. The BRICS countries economies are bound to reinvent the global eco-
nomic orderand even fashion it in their own imageonce the macroeconomic balances
are restored and foreign investment flows rebound, boosted by reforms.
Similarly, the argument that the rest of the BRICS countries will fall by the wayside while
China continues to march ahead is flawed. Chinas growth rate is almost half that of peak
levels. Its also facing a double whammy: 1) its export-led economy has been badly hit by
the slowdown in advanced country markets, and 2) rising wages and a shortage of skills
erode its competitive base, as it struggles to shift over to a domestic consumption-based
growth model. This will probably help other BRICS countries, such as India, make new
inroads into the global markets for manufactured goods, and thus close in fast on China.
In fact, the most recent trends on the global Greenfield investments, which exclude merg-
ers and acquisitions, validate this argument. The numbers show that while the new FDI
(Foreign Direct Investment) projects in China have almost halved after the global slow-
down, the other BRICS countries havent been so badly bruised. On the contrary, the gap
between China and other BRICS countries has in fact shrunk, with India accounting for
30% of the Greenfield FDI projects, as compared to Chinas 40% share.

www.manhattanreview.com 14 19992016 Manhattan Review


An introduction to Integrated Reasoning 15

Tab 2

Mike, Chief economist, on BRICS countries Professor Walter on BRICS countries

The economists assessment that the BRICS era is at an end is right on the money. De-
spite witnessing robust economic growth in the last decade, each of the BRICS countries
faces a unique set of problems today. The recent global economic downturn has exposed
structural infirmities that will prevent these economies from returning to a high-growth
trajectory anytime soon. Besides, it is not realistic to expect these emerging markets to
grow faster from a higher GDP base rather than from their previous low threshold.
In India, the economy is wracked by a Rupee in free-fall, high inflation, and a burgeoning
current account deficit. Recent months have seen significant capital outflows, with foreign
investors opting to park their funds in a recovering U.S. economy. The petering-out of
growth sentiments is directly related to the political leaderships failure to affect a much-
needed second wave of economic reforms. Furthermore, with policy paralysis expected to
continue, the India-growth story remains in limbo. In both Brazil and Russia, the weaken-
ing of commodity prices has hit the economies hard, exposing their over-reliance on nat-
ural resources as cash cows. Meanwhile, South Africas economy has been hurting since a
recession that affected several crucial industries. In China, the economy is transitioning
from resource-intensive, investment-led growth to a consumption-oriented pattern. Add
to this the massive global pressure to appreciate the Yuan, and it is clear that China would
need to affect a not-so-easy overhaul of its economic model to maintain high growth. How-
ever, as the economist points out, given Chinas planned economic model and ability to
move resources without political missteps, it is best placed from among the BRICS nations
to pull out of the middle-income trap. Taken together, the global heft that the BRICS bloc
wielded is over. While these emerging markets will continue to grow, they will need to get
used to moderate rates of growth.

19992016 Manhattan Review 15 www.manhattanreview.com


16 An introduction to Integrated Reasoning

Question:

For each of the following issues, select Agree if, based on the information provided, it can
be inferred that both the commentators would hold similar positions on the issue. Otherwise,
select Disagree.

Agree Disagree

A Acknowledging that the BRICS countries had experienced


good economic growth in the past

B The timeframe in which the BRICS countries will bounce


back to good economic growth

C Chinas ability to transform into a consumption-oriented


economy soon

Number of questions in IR section

There are 12 prompts in the IR section. The following breakdown is a possible part-wise
distribution:

Table Analysis: 23 questions

Two-Parts: 34 questions

Graphics Interpretation: 24 questions

Multi-Source Reasoning: 3 questions, 1 dataset

The Calculator

The new GMAT test window will provide you with a simple calculator at all times
during the Integrated Reasoning section but not during other sections.

The calculator link is located at the upper-left corner of the test window.

Do Use the Calculator

IR quant-based problems may have messy numbers. You must attempt these questions within
a limited time period, so when you face lengthy calculations or messy numbers, use the calcu-
lator.

www.manhattanreview.com 16 19992016 Manhattan Review


An introduction to Integrated Reasoning 17

2.5 How is the Integrated Reasoning Section Scored?

IR scores range from 18, with an increment of one point. Out of 12 prompts, there are a few
experimental questions. How many experimental questions are there in an IR section is not
yet known.

How to achieve a perfect 8?

The IR scoring algorithm is difficult to predict. However, the following table can guide you
towards achieving a perfect score of 8:

Number of
IR Score
correct questions
912 8
89 7
78 6
67 5
56 4
45 3
23 2
01 1

This table is only hypothetical. The scoring will depend on the number of experimental ques-
tions, and also on the difficulty level of the questions you answered correctly; however remem-
ber that the IR section is not an adaptive test. To be safe, assume that to get a score of 8, you
must answer at least 910 questions correctly.

How to get 910 questions correct?

Attempting not-so-familiar IR types of questions may eat up your time. Twelve prompts in 30
minutes means you get 2.5 minutes per prompt. You very well may be short of time in the IR
section. Note also that there are 13 parts (subquestions) per prompt. Every part has to be
correct for you to get the credit for the prompt. There is no partial credit.
Only the Two-Part questions are similar to Quantitative and Verbal ones. So, you should try to
do each Two-Part question in less than 2.5 minutes.
You should attempt all Graphics Interpretation questions, as they only have two parts (sub-
questions), whereas the Table Analysis and some Multi-Source Reasoning questions have three
parts (subquestions).
Though some of the long tables in Table Analysis questions may seem scary, but dont go by
the size of the tables. You have to focus on a particular parts of the tables. So, do attempt all
the TA questions.
Multi-Source Reasoning questions are the most unfamiliar and time-consuming. There may be
23 tabs of information, with three prompts per dataset. So, if your exam clock shows only 2.5

19992016 Manhattan Review 17 www.manhattanreview.com


18 An introduction to Integrated Reasoning

minutes left and you get an MSR dataset, you may not be able to attempt even one promptas
reading 23 wordy tabs of information may take more than three minutes.

The following strategy table below may guide you:

Question Number of parts Average time


Recommendation
Type per question per
part

Two-Part 2 2.5/2 = 1.250 Should attempt

Graphic 2 2.5/2 = 1.250 Should attempt


Interpretation

Table 3 2.5/3 = 0.830 Although the average time per part


Analysis is less than one minute, you really
should attempt the TA questions,
because most of them are
quant-based and thus less wordy
compared to the MSR-dataset ones.

Multi-Source 1 for 1.10 1.50 If you are hard-pressed for time,


Reasoning Multiple Choice guess on MSR prompts. The major
(3 prompts/ questions drawback of this type is that, as
dataset) with the RC type, the MSR dataset
3 for has at least three questions and
Dichotomous 23 tabs of information. The first
Choice questions question invariably takes 45
minutes.

www.manhattanreview.com 18 19992016 Manhattan Review


Chapter 3

Table Analysis

3.1 Strategies and Concepts

As stated earlier, Table Analysis questions have you use an interactive table similar to a spread
sheet, so you must get experience working with Microsoft Excel tables. Table Analysis data are
real-life data and thus may sometimes look ugly. You can infer Information from them using
the listing property and sorting capability of these tables. However, in the GMAT-IR section,
you can sort tables only in ascending order; you do not have the luxury of other Excel features,
such as being able to calculate difference, ratio, mean, median, standard deviation, correlation
factor, etc.

3.2 Commonly Tested Concepts in Table Analysis

Concepts commonly tested in Table Analysis (TA) are listed below. These include calculating
the absolute difference between values, inferring through sorting, and finding mean, median,
and mode. You will be asked to apply the concepts of measures of dispersion, and calculate
absolute deviation, standard deviation, and range.

Calculate absolute deviation


A TA question may ask you to compare the difference of values given in two columns. For
instance, it may ask you to mark Yes or No for the statement: Among the listed 22
cities, City X has the least deviation between Maximum Temperature per day and Minimum
Temperature per day.

Sort and infer


Sorting is a table feature you will use the most. You must utilize it to the fullest. A typical
table may have 410 columns, and 725 rows.
For instance, you may have to mark Yes or No for: All cities that have a Maximum
Temperature per day more than 39 F have as their Minimum Temperature per day at
least 100 F. Your most efficient approach is to sort (by default, ascending order only) the
cities by maximum temperature per day. You can quickly locate the cells with maximum

19
20 Table Analysis: Concepts & Examples

temperature more than 39 F, and then glance at the corresponding cells in the column
of minimum temperature per day. If all the cells have values more than or equal to 39 F,
you mark Yes, else No.

Mean, Median, and Mode


The concepts of median and mean are widely tested in TA prompts.
Mean

Although there may be many types of mean, from the GMAT-IRs perspective, it is
simple means and weighted means that are important.
Simple mean

Simple mean is nothing but the age-old concept: Sum of all numerical entities
divided by the number of entities. Compared to calculating the median, cal-
culating the mean may be calculation-intensive. However, you need not always
calculate its exact value in TA prompts. You can infer it by applying logical
approaches. This book has a couple of great examples with the application of
logical approaches.

Weighted mean

Weighted mean is indeed a mean, but as opposed to the simple mean, in which
each entity has equal importance or weight, the weighted mean takes into ac-
count varying importance or weight of each entity.

Lets take a question: If in a class of 40 boys and 60 girls, the percentage score
of boys is 75%, and that of girls is 85%, what is the percentage score of the entire
class?

The simple mean would be computed by adding the two percentages and divid-
ing the sum by the number of entities (two) , i.e., (75 + 85)/2 = 80%.

However, the computed simple mean 80% is wrong. Since there are more girls
than boys, the girls percentage would have more weight in the total class per-
centage.

Weighted mean =
 
(Boys % score number of boys) + (Girls % score number of girls)
Total number of students
 
(75% 40) + (85% 60) (30% + 51%)
Weighted mean (score) = = = 81%
100 100

Median

www.manhattanreview.com 20 19992016 Manhattan Review


Table Analysis: Concepts & Examples 21

Median is also a widely tested concept in TA prompts. Median is the middle-most


value of a data set when the data are arranged in ascending order (or descending
order, but you can sort tables only in ascending order). To find out the median, you
have to sort the table by the appropriate column.

n+1 th
 
Median = Value of 2 cell, n = number of rows

If the number of rows is even, we take the simple mean of the values of two middle-
most cells.

th th
   
n n
Median = Mean of 2 cell + 2 +1 cell , n = number of rows

Mode

Although the Official Guide writes about mode, it has not been tested so far.

Dispersion

An easy-to-understand definition of dispersion is Spread of data. For instance, for a


series 10, 10, 12, 14, 16, 17, 17, and 20, the spread of data is from 10 to 20.

Deviation

Deviation is a measure of how far data are from the average value. Each datapoint will
have its value of deviation with respect to the average.
Say, for example, you have a series: 10, 12, 14, 16, 18, 20, 22, 24, 26, 28, and 30. The
average (or, in this case, also the mean) of the series is 20. So, the deviations are:

Data point (X) 10 12 14 16 18 20 22 24 26 28 30


Deviation (X 20) 10 8 6 4 2 0 2 4 6 8 10

Note that the farther a data point is from the mean, the greater is the deviation.

Range

Range is tested in TA questions. Range of a series is the difference of Highest & Lowest
values of data points in a series.

Range of a series = Highest value Lowest value

In the example above, Range = 30 10 = 20.

Standard Deviation

19992016 Manhattan Review 21 www.manhattanreview.com


22 Table Analysis: Concepts & Examples

Deviation yields the value of deviation for each data point. Range yields a single value
of deviation for the whole series but takes into account only the highest and the lowest
values; however Standard deviation takes each data point into consideration yet yields a
single value of deviation for the whole series.
You need not know the mathematical formula to calculate standard deviation. An IR
question will never ask you to calculate the value. Mere understanding of the concept is
sufficient.
For those who are curious to know, this is the definition of Standard Deviation (SD):

Standard deviation (SD) is the positive square root of the mean of


squared deviations.

It may not be easy to comprehend the definition, but you need not understand how the
SD is calculated. Here are three examples to help you better understand it:

Example 1 Data Points: 10 10 10 10 10 10 10 10 10 10


Mean = 10, Range = 0, SD = 0.

Example 2 Data Points: 10 10 10 10 10 20 20 20 20 20


Mean = 15, Range = 10, SD = 5.

Example 3 Data Points: 10 11 12 14 16 16 11 17 17 20


Mean = 15.1, Range = 10, SD = 3.08.

Correlation

Correlation is a measure of relationships between any two entities. Two entities are
positively correlated if both of them either increase in tandem or decrease in tandem,
whereas the entities are negatively correlated if one entity decreases in value while the
other entity increases in value, or vice versa. The entities are not correlated if the increase
or decrease in the value of one entity cannot be concluded relative to the increase or
decrease in the value of the other.
We will discuss this concept with the help of an example.

www.manhattanreview.com 22 19992016 Manhattan Review


Table Analysis: Concepts & Examples 23

3.3 Process of Solving TA Questions

(1) Understand the data set


Read the question narration and the column headings of the table. Try to understand the
relationship of a column with other column(s). Sometimes column headings are not very
expressive; however, you can get information about them from the question narration or
any note given outside the table area.
Dont be intimidated by the number of rows and number of columns. You will have to
deal with only two to three columns and a few rows at a time. Quickly scan through
the data given in the table. By doing this, you will grasp whether the data is given in
integers or in some ugly numbers in decimal form. If the data is given in decimals, dont
be intimidated. You can always approximate them to make manageable integers.

(2) Understand the question


As you know, that there are three dichotomous statement type questions in a TA dataset.
You have to mark each statement with either Yes or No. Remember that there is no
partial credit for correct response(s); each of the three responses must be correct for you
to get the credit for the question.
Read the question statement and understand it. Rephrase the statement in your words,
and if needed, translate it into mathematical language.

(3) Develop an approach


There could be two or more ways to approach a question. You may figure out that a
question requires two-way sorting, but you can sort the table by one column at a time.
So, figure out the best approach to solve the question.

(4) Apply the approach


Apply your approach and click the correct radio button. You should jot down the results
of any calculations you do. It is likely that you might need one or two results from
previous questions as intermediate steps for answering the questions that follow.

19992016 Manhattan Review 23 www.manhattanreview.com


24 Table Analysis: Concepts & Examples

3.4 Examples

Here are few GMAT-like questions.


In an actual GMAT exam, you will select columns to sort using a drop-down menu. In this
book, we obviously cannot provide such menus. This does not mean that, as with the other
renowned IR textbooks, that you cannot solve problems. We have provided you images of
sorted tables for each question.
To practice questions on your own, write to us at [email protected], we will send
you filter-enabled tables, so you can practice in a computer-enabled environment.

Example 1

The table shows area, population, and population density for 11 towns.

Population
Area Population
Town Density ('000
('000 square mile) (Million)
people/sq. mile)

A 2.5 4.89 1.96


B 1.6 5.26 3.29
C 4.58 6.78 1.48
D 3.52 8.25 2.34
E 6.87 12.65 1.84
F 9.12 13.58 1.49
G 5.23 12.75 2.44
H 8 16.3 2.04
I 2.68 15.35 5.73
J 9 24.54 2.73
K 4.56 25.54 5.60

For each of the following statements, select Yes if the statement is true based on the infor-
mation provided; otherwise, select No.

Yes No

Population density of town D represents the


A
median population density of all the towns.

The town with the highest population density


B also has the highest population among all the
towns.

The town with the lowest population density


C
also has the smallest area among all the towns.

www.manhattanreview.com 24 19992016 Manhattan Review


Table Analysis: Concepts & Examples 25

Solution, Part A

The question wants you to find out whether population density (P.D.) of town D = the median
population density of all the towns.
To get the value of the median population density, sort the table by column 4 (Population
Density):

Sorted by column 4
Col 1 Col 2 Col 3 Col 4

Population
Area Population
Town Density ('000
('000 square miles) (Million)
people/sq. miles)

C 4.58 6.78 1.48


F 9.12 13.58 1.49 Median of P. D.
E 6.87 12.65 1.84
A 2.5 4.89 1.96 = Value of P. D. for
""#" th
H 8 16.3 2.04 ( ) town
$
D 3.52 8.25 2.34
= Value of P. D. for 6th
G 5.23 12.75 2.44 town
J 9 24.54 2.73
B 1.6 5.26 3.29 = 2.34
K 4.56 25.54 5.60
I 2.68 15.35 5.73

There are 11 rows/towns, thus the middle-most value (Median) in column 4 after sorting it
would be the value of 11+1
2 = 6th town = 2.34.
The population density of town D = median population density of all the towns = 2.34.
The answer for Part A is Yes.

Solution, Part B

The question wants you to find out whether the town that has the highest population density
is the same as the one that has the highest population.
Looking at the table, you find that town I has the highest population density = 5.73 units, but
town K, not I, has the highest population = 25.54 Million among all the towns.
The answer for the part B is No.

19992016 Manhattan Review 25 www.manhattanreview.com


26 Table Analysis: Concepts & Examples

Solution, Part C

The question wants you to find out whether the town with the lowest population density is the
same as the one that has the smallest area.
Looking again at the table, you find that town C has the lowest population density = 1.48 units,
but town B has the smallest area = 1.6 units among all the towns.
The answer for the part C is No.

www.manhattanreview.com 26 19992016 Manhattan Review


Table Analysis: Concepts & Examples 27

Example 2

The table presents quarterly sales and inventory data, in metric tons (MT), for Apex Corpora-
tion for a few steel products.

S - Sales (MT); I - Inventory (MT)

Products Quarter I Quarter II Quarter III Quarter IV Yearly Total

S I S I S I S I S I
CR pipe 24 12 15 8 4 8 13 4 56 32
CR sheet 16 8 12 12 16 4 15 21 59 45
CR tube 15 15 16 8 6 15 8 4 45 42
CR wire 15 0 24 8 18 13 13 12 70 33
HR pipe 24 12 21 8 5 21 12 15 62 56
HR sheet 18 15 15 16 0 0 14 3 47 34
HR tube 8 9 8 21 8 0 9 2 33 32
HR wire 24 26 12 16 15 15 8 4 59 61
HS Billet 21 24 21 0 18 5 15 12 75 41
MS Billet 10 8 15 0 18 2 15 16 58 26
Total 175 129 159 97 108 83 122 93 564 402

For each of the following statements, selectYes if the statement is true based on the infor-
mation provided; otherwise select No.

Yes No

In quarter II, among all products, CR tube sale


A is least deviated from the arithmetic mean sale
for that quarter.

In quarter IV, among all the products, highest


B sales-to-inventory ratio was observed for HR
tubes.

Median sale for quarter I is more than that for


C
each of the other three quarters.

19992016 Manhattan Review 27 www.manhattanreview.com


28 Table Analysis: Concepts & Examples

Solution, Part A

The question wants you to find out whether in the Q2, the deviation (difference) of CR tube
sale from the Q2 arithmetic mean sale is less than that of other products.
In other way, the question statement states that for Q2, the sales of CR tube is closest to Q2
mean sales.
Mathematically, this means:


CR tube sales mean sales for Q2 < Other products sales mean sales for Q2 .

First, you calculate the mean sales for Q2.


Since

Total Q2 sales = 159 MT,

and

Total number of products = 10,

then

Q2 mean sales = 159/10 = 15.9 MT.

So,


CR Tube sales mean sales for Q2 = 16 15.9 = 0.1.

Run through the values in column 4 (Q2 sales) and mentally deduct each value from 15.9. You
will find that the deviation of CR Tube sales = 0.1 is the lowest among that of all the other
products.
The answer for part A is Yes.

www.manhattanreview.com 28 19992016 Manhattan Review


Table Analysis: Concepts & Examples 29

Solution, Part B

The question wants you to find out whether in the Q4, the ratio of sales to inventory for
HR tubes is more than that for other products.
Mathematically, this means that for Q4,

h i h i
Sales Sales
Inventory HR tube > Inventory Other products

h i
Sales
Inventory HR tube = 9/2 = 4.5.

You have nine products for which you need to calculate the sales-to-inventory ratio, but that
would be too time-consuming, and hence it is not advisable to compute.
A better approach would be to calculate a few select values. You know that for HR tubes, the
value is 4.5, which is more than 1. You can skip the products for which sales are less than
their inventory as the sales-to-inventory ratio value would be less than 1. Secondly, you can
mentally divide sales by inventory to get probable contenders that have values, equalling more
than 4. There is only one such product: HR sheets, which has a sales-to-inventory value equal
to 14/3 = 4.67, more than 4.5, so the statement is wrong.
The answer for part B is No.

19992016 Manhattan Review 29 www.manhattanreview.com


30 Table Analysis: Concepts & Examples

Solution, Part C

The question wants you to find out whether median sales for Q1 is more than median sales
for each of Q2, Q3, and Q4.
Mathematically, this means:


Median sales Q1 > Median sales any other quarter

To get the median sales of each quarter, sort the table by its sales columns to get the table:

Sorted by Q1 Sales (Column 2) Sorted by Q2 Sales (Column 4) Sorted by Q3 Sales (Column 6) Sorted by Q4 Sales (Column 8)
Col 1 Col 2 Col 3 Col 1 Col 4 Col 5 Col 1 Col 6 Col 7 Col 1 Col 8 Col 9
Products Quarter I Products Quarter II Products Quarter III Products Quarter IV
S I S I S I S I
HR tube 8 9 HR tube 8 21 HR tube 0 0 HR tube 8 4
MS Billet 10 8 MS Billet 12 12 MS Billet 4 8 MS Billet 8 4
CR tube 15 15 CR tube 12 16 CR tube 5 21 CR tube 9 2
CR wire 15 0 CR wire 15 0 CR wire 6 15 CR wire 12 15
CR sheet 16 8 CR sheet 15 16 CR sheet 8 0 CR sheet 13 4
HR sheet 18 15 HR sheet 15 8 HR sheet 15 15 HR sheet 13 12
HS Billet 21 24 HS Billet 16 8 HS Billet 16 4 HS Billet 14 3
HR wire 24 26 HR wire 21 0 HR wire 18 2 HR wire 15 21
CR pipe 24 12 CR pipe 21 8 CR pipe 18 13 CR pipe 15 16
HR pipe 24 12 HR pipe 24 8 HR pipe 18 5 HR pipe 15 12
Total 175 129 Total 159 97 Total 108 83 Total 122 93

Median Sales for Q1:

n+1 th
 
Median of a series = value of the 2 term

Here, median sales for Q1 equals:

10+1 th
 
Value of 2 product = value of the 5.5th product.

The value of the 5.5th product equals

Average of 16 & 18 = (16 + 18)/2 = 17 MT

Similarly, median sales for Q2, Q3, and Q3 are (15 + 15)/2 = 15, (8 + 23)/2 = 11.5, and (13 +
13)/2 = 13 MT, respectively. Hence, the statement is correct.
The answer for the part C is Yes.

www.manhattanreview.com 30 19992016 Manhattan Review


Table Analysis: Concepts & Examples 31

Example 3

The table gives data for three attributes of 16 brands of cars: 1) Performance, 2) Style, and
3) Value for money. Each attribute for a brand of car is rated out of 10 points by a group of
experts. Total Score was computed as a weighted mean of three scores, using the same weights
for each brand of cars.

Performance Value for Total


Rank Brand Car company Style Score
Score money Score Score
1 3 Series BMW 9 9.1 8.8 9.00
2 7 Series BMW 9.2 7.3 9.7 8.54
3 Expedition Ford Motors 8.8 9 7 8.52
4 Sonata Hyundai 9.3 7.5 9 8.52
5 Fiesta Ford Motors 8 9 6.5 8.10
6 Juke Nissan 7.1 9 6.4 7.72
7 Sonic General Motors 8.3 7.7 5.9 7.58
8 5 Series BMW 8.2 5.3 8.2 7.04
9 Grandeur Hyundai 7 7.8 4.4 6.80
10 JM Hyundai 7.2 6.8 4.7 6.54
11 Cruze General Motors 6.6 5.6 5.5 5.98
12 M Model BMW 5.5 6.4 5.5 5.86
13 Micra Nissan 5.7 5.9 4.8 5.60
14 Maliba General Motors 7.3 4.3 4.5 5.54
15 Dayz Nissan 4.8 3.8 3.4 4.12
16 Mondeo Ford Motors 4.1 4.8 2.2 4

For each of the following statements, select Yes if the statement is true based on the infor-
mation provided; otherwise select No.

Yes No

Considering the median Style scores to com-


A pare Nissan and BMW cars, BMW cars are more
stylish than Nissan cars.

The score for Performance and the score for


B Value-for-money had equal weights in the com-
putation of the total score.

There is a positive correlation between the Per-


C
formance score vs. the Value-for-money score.

19992016 Manhattan Review 31 www.manhattanreview.com


32 Table Analysis: Concepts & Examples

Solution, Part A

The question asks you to compare the median style scores for BMW and Nissan cars. To get
this, sort the table by column 3, Car company. This will group all BMWs and all Nissan cars.
Refer to the sorted table given below. Observe that the median style score for BMW, 6.85, is
more than that for Nissan, 5.9.

Sorted by column 3
Col 3 Col 4 Col 5

Car company Performance Score Style Score Median Style scores for BMW
BMW 8.2 5.3 !"# %&'() * +,# %&'()
BMW 5.5 6.4 =
!
BMW 9.2 7.3
BMW 9 9.1 -./*0.+
Ford Motors 4.1 4.8
= !
= 6.85
Ford Motors 8.8 9
Ford Motors 8 9
General Motors 7.3 4.3
General Motors 6.6 5.6
General Motors 8.3 7.7
Hyundai 7.2 6.8
Hyundai 9.3 7.5
Hyundai 7 7.8 Median Style scores for Nissan
Nissan 4.8 3.8
Nissan 5.7 5.9 = 2nd value = 5.9
Nissan 7.1 9

The answer for Part A is Yes.

Solution, Part B

Let us first understand how the Total Score is calculated. Total Score is a weighted mean score
(weighted average) of the three scores: Performance score (PS), Style score (SS), and Value-for-
money (VFM) score. The question asks you to find out whether the weight of Performance and
that of VFM are same in the computation of Total Score.
However, there is no information given about the Style score.
The optimum approach is to assume that the question statement is true i.e. the weight of
Performance and that of VFM are same. Now you should calculate the weight of Style score at
least twice from any two sets of scores while computing total score. If the values of weights
for Style score are same, the answer is Yes; otherwise No.
Say, the weights of Performance score = weights of the VFM score = 1, and the weights of the
Style score = x.
Now, apply the formula of weighted mean.

www.manhattanreview.com 32 19992016 Manhattan Review


Table Analysis: Concepts & Examples 33

PS weight PS + VFMS weight VFMS + SS weight SS


Weighted mean = ;
total weight

where PS is the Performance score, VFMS is the VFM score, and SS is the Style score.
By plugging in the assumed values of weights in the formula, you get:

PS + VFMS + x SS
Total score =
(2 + x)

To calculate x, plug in Style scores for any two brands of cars into the formula. If the values
of x calculated from each set of data are not same, then the question statement is not true.
Otherwise, try the same approach with one more brand of cars, to be doubly sure that the
question statement is true.
Say, we take the 3-Series and Mondeo brand cars.
For the 3-Series brand, the scores for Performance, Style, VFM, and Total Scores are 9, 9.1, 8.8,
and 9, respectively. Plug in these values into the formula of total score:

(9 + 8.8 + 9.1x)
9= 18 + 9x = 17.8 + 9.1x 0.2 = 0.1x x = 2.
(2 + x)

Now, try the same with the Mondeo brand. You get:

(4.1 + 2.2 + 4.8x)


4= 8 + 4x = 6.3 + 4.8x 1.7 = 0.8x x = 2.125.
(2 + x)

Since the values of x are not same (x = 2 or 2.125), hence the statement is false.
The answer for part B is No.
For your curiosity, the ratio of weights of Performance score to that of VFM score to that of
Style score is 2 : 2 : 1.

Solution, Part C

This is a question on the concept of correlation. Hopefully, you have already gone through
this concept in the book. Nevertheless, we repeat it:

Two entities are positively correlated if both of them either increase in tandem or
decrease in tandem. Conversely, the entities are negatively correlated if the other
entity decreases in value when the first entity increases in value, or vice versa. The
two entities are not correlated if the increase or the decrease in the value of second
entity cannot be concluded relative to the increase or the decrease in the value of
first entity.

To approach this problem, sort the table by column 4, Performance score, so that the values
are arranged in ascending order or, in other words, in the increasing trend.

19992016 Manhattan Review 33 www.manhattanreview.com


34 Table Analysis: Concepts & Examples

The next step is to observe how the entity in column 6, Value for money Score, behaves. As
you can observe in the sorted table given below, the downward-pointing arrows represent
increasing values, while upward-pointing arrows represent decreasing values.

Sorted by column 4
Col 4 Col 5 Col 6
Performance Value for
Style Score
Score money Score
4.1 4.8 2.2
4.8 3.8 3.4
5.5 6.4 5.5
5.7 5.9 4.8
6.6 5.6 5.5
7 7.8 4.4
7.1 9 6.4
7.2 6.8 4.7
7.3 4.3 4.5
8 9 6.5
8.2 5.3 8.2
8.3 7.7 5.9
8.8 9 7
9 9.1 8.8
9.2 7.3 9.7
9.3 7.5 9

Looking at the arrows, you cannot conclude that these entities are perfectly correlated as there
are many downward and many upward arrows. You must do some more analysis.
Again observing the arrows, you will find that the length of downward-pointing arrows is
longer than that for upward-pointing arrows, which means that most values in the column 6
are increasing with the increase in the values in column 4, implying that the entities have,
though not perfect, a strong positive correlation.
The answer for part C is Yes.

www.manhattanreview.com 34 19992016 Manhattan Review


Table Analysis: Concepts & Examples 35

Example 4

Cricket is played in a few select countries. Among these, Cricket is the most popular sport in
a few of them. It is played in three formats: T20: played for four hours; One-Day: played for
8 hours; and Five-Day: played for 8 hours a day for five days.
The table given below presents percentage distribution of Cricket matches national teams
played last year in three formats.

Cricket-Most
Country T20 One-Day Five-Day
Popular sport?

Australia No 65 28 7
Bangladesh Yes 45 35 20
England No 20 35 45
India Yes 9 30 61
New Zealand No 44 38 18
Pakistan Yes 8 36 56
South Africa No 66 10 24
Sri Lanka Yes 4 35 61
West Indies Yes 36 36 28
Zimbabwe No 56 36 8
For each of the following statements, select Would help explain if were true, help explain some
of the information in the table. Otherwise, select Would not help explain.

Would help Would not


explain help explain

Among its three formats, Cricket in its T20 for-


mat is more popular in the countries where
A Cricket is not their most popular sport than in
countries where Cricket is their most popular
sport.

Among the subcontinent countries: India, Pak-


istan, Sri Lanka, and Bangladesh, the One-Day
B
format is the second most preferred type of
Cricket format.

More Five-Day matches were played by each of


C
India and Sri Lanka than by any other countries.

Solution, Part A

The question wants you to find out if more number of T20 Cricket matches are played than
other two formats in the countries where Cricket is not the most popular sport than that in

19992016 Manhattan Review 35 www.manhattanreview.com


36 Table Analysis: Concepts & Examples

countries where it is the most popular sport.


To deduce this, you should sort the table by column 2, Popularity of Sport.

Sorted by column 2
Col1 Col2 Col3 Col4 Col5
Cricket-Most
One- Five-
Country Popular T20
Day Day
sport?
Australia No 65 28 7
England No 20 35 45
New Zealand No 44 38 18
South Africa No 66 10 24
Zimbabwe No 56 36 8
Bangladesh Yes 45 35 20
India Yes 9 30 61
Pakistan Yes 8 36 56
Sri Lanka Yes 4 35 61
West Indies Yes 36 36 28

You see in the table given above that for England, where Cricket is not the most popular sport,
the least number of T20 matches are played than the two other formats. On the other hand, in
the West Indies, where Cricket is the most popular sport, the T20 format is as popular as the
One-Day format. Similarly, in Bangladesh, where Cricket is the most popular sport, the more
number of T20 matches are played than the two other formats.
The analysis shows that the table does not help explain the statement.
The answer to part A is Would not help explain.

www.manhattanreview.com 36 19992016 Manhattan Review


Table Analysis: Concepts & Examples 37

Solution, Part B

The question wants you to determine whether for countries: India, Pakistan, Sri Lanka, and
Bangladesh, the number of One-Day matches played in each country lies between the number
of T20 matches and the number of Five-Day matches.

Sorted by column 2
Col1 Col2 Col3 Col4 Col5
Cricket-Most
Five-
Country Popular T20 One-Day
Day
sport?
Australia No 65 28 7
England No 20 35 45
New Zealand No 44 38 18
South Africa No 66 10 24
Zimbabwe No 56 36 8
Bangladesh Yes 45 > 35 > 20
India Yes 9 < 30 < 61
Pakistan Yes 8 < 36 < 56
Sri Lanka Yes 4 < 35 < 61
West Indies Yes 36 36 28

Looking at the table given above, you can infer that for each of these four countries, the per-
centages of One-Day matches lies between that of T20 and that of Five-Day matches, implying
that One-Day format is the second most preferred type of Cricket format in them.
Alternatively, we can infer that in these four countries, either T20 or Five-Day format is the
most popular.
So the table helps explain the statement.
The answer to part B is Would help explain.

Solution, Part C

The question wants you to find out if more number of Five-Day matches were played by each
of India and by Sri Lanka than by any other countries.
This question seems relatively simple to deduce, but it is not so. The data given in the table is
the percentage distribution of matches played by each country in three formats. You cannot
deduce any information about the absolute number of matches each country played.
The table does not help explain the statement.
The answer to part C is Would not help explain.

19992016 Manhattan Review 37 www.manhattanreview.com


38 Table Analysis: Concepts & Examples

Example 5

Weston Private Limited produces and sells various brands of televisions for select categories
of televisions. The data given in the table shows the Regional Production percentage share,
its relative rank, the Regional Sales percentage shares, and its relative rank for the companys
products relative to competitors products in the corresponding television product category.
No two companies share the same rank.

TV Production, Production, Sales, Sales,


Product Regional Regional Regional Regional
category Share(%) Rank Share(%) Rank
3D LCD 25 3 30 2
3D LED 45 1 55 1
B/W TV 21 3 12 5
FST 64 1 74 1
LCD 28 2 12 3
LED 13 4 10 4
Plasma 12 6 5 8
Smart TV 24 2 32 1
For each of the following statements, select Would help explain, if it would, if true, help
explain some of the information in the table; otherwise, select Would not help explain.

Would help Would not


explain help explain

If Weston Private Limited produces more than


one-fourth of the regional production of any
A category of televisions, then it gains one of the
top two ranks for the production for that cate-
gory of televisions.

No individual company produces more than


B 68% of the regional production for the LCD cat-
egory of televisions.

If Weston Private Limited is ranked last in re-


gional sales for the B/W TV category of televi-
C
sions, then the first ranker would gain a mini-
mum of 46% of the regional sales share.

www.manhattanreview.com 38 19992016 Manhattan Review


Table Analysis: Concepts & Examples 39

Solution, Part A

The question asks its true that if Weston produces more than 25% (1/4th ) of the TVs, then its
production rank is either first or second.
Looking at the table, you will find that there are three categories of TVs for which the percent-
age production share of Weston is more than 25%: 3D LED has 45% of production share and
is ranked 1; FST has 64% of production share and is ranked 1; and LCD has 28% of production
share and is ranked 2.
Hence the table does help explain the statement.
The answer to part A is Would help explain.

Solution, Part B

The question wants you to find out if no company produces more than 68% of LCD TVs.
In the LCD TV category, Weston is ranked second in production share with a production share
of 28%. It means that the first ranker may command production share of up to 72% (100%
28%). We should not assume that there must be more than two companies.
Hence the table does not help explain the statement.
The answer to part B is Would not help explain.

Solution, Part C

The question wants you to find out if Weston is ranked last in the regional sales for B/W TV
category, then the first ranker would gain a minimum of 46% regional sales share.
Currently, in B/W TV category, Weston is ranked fifth in the regional sales with a sales share of
12%. Westons fifth rank is the last rank (given in the question statement), implying that other
four competitors will share 88% sales (100%12%). So the first ranker will have more than 22% sales share
as 88%/4 = 22% will rank all the competitors at same rank, which is incorrect.
Similarly, the first ranker will command less than 52% sales share because assuming that com-
panies ranked from second to fourth, each have a minimum of a little over 12% sales share,
so collectively they will have more than 48% shareas each rank cannot have equal sales share.
So, the first ranker will gain less than 52% sales share [100% (48% & more)].
Hence the table does not help explain the statement.
The answer to part C is Would not help explain.

19992016 Manhattan Review 39 www.manhattanreview.com


40 Table Analysis: Concepts & Examples

Example 6

A gym has to choose a health drink from among eight health drinks in the market. The health
drinks are rated based on four criterion. For each of the criterion in the table, if the rating
value for a drink is more than or equal to the median rating value for that category, it is
considered a favored drink in that category. If a drink is favored in more than one criterion,
it is consideredmore favorable. A drink is considered more favorable than another drink if
it is favored in more criterions than does the other drink.

Vitamin B12 Vitamin B6 Vitamin B3 Vitamin C


Health Drink Brand
Rating (10) Rating (5) Rating (100) Rating (10)
Booster+ 3 3.2 34 4
Deli 6 2.2 47 7
Doz 6 3.8 74 7
Energy+ 5 3.2 52 5
MegaPower 5 3.2 54 5
MusclePower 6 4.2 74 7
PowerBooster 5 3.2 54 6
Topper 4 4.2 48 5

For each of the following statements, select Yes if the statement is true based on the infor-
mation provided; otherwise select No.

Yes No

No other drinks are more favored than Power-


A
Booster and MusclePower.

Topper and Booster+ are not favored for any of


B
the criterion.

There are at least three drinks that are favored


C
for all the four criterion.

www.manhattanreview.com 40 19992016 Manhattan Review


Table Analysis: Concepts & Examples 41

Solution, Part A

The question wants you to find out whether there is a drink more favored than PowerBooster
and MusclePower.
Since the question involves finding out the favored drinks in each category, first find out the
median values for each category. See the tables given below.

Sorted by column 2 Sorted by column 3


Col 1 Col 2 Col 1 Col 3

Vitamin B12 Vitamin B6


Health Drink Brand Health Drink Brand
Rating (10) Rating (5)

Booster+ 3 Deli 2.2

Topper 4 Booster+ 3.2


Energy+ 5 Energy+ 3.2
MegaPower 5 Median = 5 MegaPower 3.2 Median = 3.2
PowerBooster 5 PowerBooster 3.2
Deli 6 Doz 3.8
Doz 6 Topper 4.2
MusclePower 6 MusclePower 4.2

Sorted by column 4 Sorted by column 5


Col 1 Col 4 Col 1 Col 5

Health Drink Vitamin B3 Vitamin C


Health Drink Brand
Brand Rating (100) Rating (10)

Booster+ 34 Booster+ 4
Deli 47 Energy+ 5
Topper 48 MegaPower 5
Energy+ 52 Median = 53 Topper 5 Median = 5.5
MegaPower 54 PowerBooster 6
PowerBooster 54 Deli 7
Doz 74 Doz 7
MusclePower 74 MusclePower 7

In the table given below, the cells containing qualifying values for each criterion are shaded.

Col 1 Col 2 Col 3 Col 4 Col 5

Vitamin B12 Vitamin B6 Vitamin B3 Vitamin C


Health Drink Brand
Rating (10) Rating (5) Rating (100) Rating (10)

Booster+ 3 3.2 34 4
Topper 4 4.2 48 5
Energy+ 5 3.2 52 5
MegaPower 5 3.2 54 5
PowerBooster (All 4) 5 3.2 54 6
Deli 6 2.2 47 7
Doz (All 4) 6 3.8 74 7
MusclePower (All 4) 6 4.2 74 7
Median 5 3.2 53 5.5

You can rank the health drinks the number of times they are favorable. In this table, you will

19992016 Manhattan Review 41 www.manhattanreview.com


42 Graphics Interpretation: Concepts & Examples

find that PowerBooster and MusclePower are favored the maximum number of times (four).
Although Doz is also favored an equal number of times, it is not favored more number of
times than either PowerBooster or MusclePower.
The answer to part A is Yes.

Solution, Part B

The question asks if its true that Topper and Booster+ are not favored for any of the criterion.
Referring again to the given table, you will find that both Topper and Booster+ are favored
once: for Vitamin B6.
The answer to part B is No.

Solution, Part C

The question asks if there are at least three drinks favored for all criterion. We see that
PowerBooster, MusclePower, and Doz are favored for all the four criterion.
The answer for part C is Yes.

www.manhattanreview.com 42 19992016 Manhattan Review


Chapter 4

Graphics Interpretation

4.1 Strategies and Concepts

As stated earlier, the Graphics Interpretation questions ask you about a chart, graph, diagram,
or other visual form of information. It is vital that you learn how to read these.

4.1.1 Some Terminology

Legend Chart Title


Currency Rates
Rs to $ Real to $ $ to Dinar
Secondary
Primary axis
70 3.7
axis
Real to $ $ to Dinar

3.5
65
3.3
Rs. To $

60 3.1
Axis Title
55 2.9

2.7
50
2.5
Primary
axis 45 2.3
gridline
Secondary
Months axis
Category gridline
Axis Title
name

Figure 4.1: Key terms in graphs/charts.

43
44 Graphics Interpretation: Concepts & Examples

Chart Titles
See the Figure 4.1 example chart. The chart title briefly describes what the graph is all
about. Usually, it is a short phrase placed above the plot area of a graph.

Axis
There are two axes in Figure 4.1s the chart: the horizontal axis and the vertical axis. Few
questions use charts with two vertical axes.

Horizontal axis:
Horizontal axis usually shows categories such as years, months, cities, products,
and so on.
Vertical axis:
Vertical axis usually shows data-point values, such as population, inflation, number
of cars, revenue, etc. Most often, the charts only have one vertical axis, on the charts
left. A few questions may use charts with two vertical axes. If so, the second vertical
axis is on the charts right. The left axis is called the primary vertical axis and the
right one is called the secondary vertical axis.

Legends
Legends are the boxes on the right-hand-side, top, or bottom of the plot area. They
indicate to which entity each bar or type of line relates.

Scale
Usually only one scale is used: the vertical-axis scale. A few questions may use two
vertical scales. The vertical scale is marked with units (numerals) at equal intervals. For
example, if a graph represents the number of students in certain schools, the vertical
scale may be marked with 0, 50, 100, 150, 200, 250, and 300 units. So here, there are
50 units in each interval.
A chart will use two vertical scales if two or more entities cannot be represented in the
primary vertical axis. You must exercise caution while reading the values. For example,
if a chart shows the number of students in some schools along with the percentage of
passing students in those schools, the primary vertical axis may represent the number of
students while the secondary vertical axis may represent the percentage passing.
Pay attention to the legend and the question description so that you keep the primary
and secondary axes correctly identified.

Gridlines
Horizontal and vertical gridlines make it easier to read the data. Vertical-axis gridlines
extend up and Horizontal-axis gridlines extend across the plot area. Displayed for major
and sometimes minor units, horizontal gridlines align with major and minor tick marks
on the axis.
You cannot display gridlines for the graphs that do not display axes, such as pie charts.

www.manhattanreview.com 44 19992016 Manhattan Review


Graphics Interpretation: Concepts & Examples 45

4.1.2 How to Read a Chart

You need not try to extract an EXACT value of data from a chart/graph. Your best estimate will
do. For example, in Figure 4.2, the Rs./$ rate was approximately 57 in July, and the $/Dinar rate
was approximately 3.6 in September. Dont worry about the values being just approximations,
the options will be wide enough apart.

In Sept, $/Dinar
was ~3.6.
Currency Rates
Rs to $ Real to $ $ to Dinar

70 3.7

Real to $ $ to Dinar
In July, Rs./$ 3.5
65
was ~57. 3.3
Rs. To $

60
3.1

55 2.9
2.7
50
2.5
45 2.3

Months

Figure 4.2: How to read a graph/chart.


Be careful to choose the appropriate vertical scale while reading the values of Rs./$ (on the
primary vertical axis) and $/Dinar (on the secondary vertical axis).

19992016 Manhattan Review 45 www.manhattanreview.com


46 Graphics Interpretation: Concepts & Examples

4.2 Quantitative Concepts Tested in the Graphics Interpretation

Absolute increase or decrease


This is the simplest of all. You simply read the values of two or more data points and
figure out the difference. A word of caution: Distinguish Absolute increase or decrease
vs. Percentage increase or decrease.

Percentage increase or decrease


Suppose you are asked to calculate a percent value. Say, for example, you are asked:

For a city whose population rose from 45 million in the year 2011 to 50 million
in the year 2012, the percentage increase in the population would be %.

(new value base value)


 
% increase = 100%
base value
" #
population in 2012 population in 2011
% increase in population = 100%
population in 2011

50 45 5
% increase in population = 100% = 100% = 11.11%
45 45

Beware of the Word than


If a question asks:

By what percent is the population in the year 2011 less than the population in
the year 2012?

Will the answer be 11.11%? No. It is not. In this question, the base of comparison is
changed to the year 2012, so year 2012 will appear in the denominator as the base value:

50 45 5
% decrease in population = 100% = 100% = 10%
50 50

Its Not Always Percentage Increase or Decrease

If the question asks:


What percent is the population in the year 2011 of the population in the
year 2012?
You need not calculate the percentage increase or decrease. The answer would be
calculated as given below:

population in 2011 45
%= 100% = 100% = 90%
population in 2012 50
If the question asks:

www.manhattanreview.com 46 19992016 Manhattan Review


Graphics Interpretation: Concepts & Examples 47

What percent is the population in the year 2012 of the population in the
year 2011?
You should calculate the same in the following way:

population in 2012 50
%= 100% = 100% = 111.11%
population in 2011 45

Determine Increased or Decreased Value


If the question asks:
What would be the population in the year 2013 if the population grows at
the same rate as it did during the period 20112012?
You have already calculated the percentage increase in the population = 11.11%. Ap-
plying the traditional approach to solving this question is time-consuming. In the
traditional approach, you would calculate the increase first, and then add the in-
crease to the population in the year 2012 to get the population for the year 2013.

The following approach will save your precious time:

population|2013 = population|2012 (1 + % increase)


= 50 (1 + 11.11%) = 50 (1 + 0.111) = 55.55 million

Similarly, if the question is:


What would be company Xs revenue for next year if it decreased by 23.87%
from the current years revenue of $237.48 million?
The following approach will save your precious time:

revenue|next year = 237.48 (1 0.2387) = 237.48 0.7613 = $180.80 million

You can even save more time by truncating the decimals:

revenue|next year = 237 (1 0.24) = 237 0.76 $180 million

If the question asks:


A shopkeeper used to charge 15% margin on his products. Due to compe-
tition, he revised the margin to 10%. By what percent did the shopkeeper
decrease his margin?
The answer is certainly not 5%. It would rather be:

15% 10%
% decrease in margin = 100% = 33.33%
15%

So, what does 15% 10% represent?

Percent Percent = Percent Point

So, alternatively, you can also say that the shopkeeper reduced his margin by 5
percent points.

19992016 Manhattan Review 47 www.manhattanreview.com


48 Graphics Interpretation: Concepts & Examples

Remember,

percent percent 6= percent

Percent of What?
If the question asks:

If 80% of students from a school play soccer and 25% of those joined a profes-
sional soccer club, what percentage of total students from the school did not
join the club?

The answer is not 75% (100% 25%).


You must pay attention to the verbiage. It is not always the case that the answer is a
percentage of the total.
Here, the number of students who joined the club = 25% of 80% = 20%. So, if 20% of total
students joined the club, 80% of total students did not join the club.

Trend Lines
Scatterplots are tested in the GMAT-IR section. In scatterplots, data points are repre-
sented as dots, and these dots are usually many in number. A trend line is a straight line
that tries to best fit or represent the scattered dots. See Figure 4.3 below.

Age-wise number of patients


Age Vs. Disease X Age Vs. Disease Y
Tredline (Age Vs. Disease X) Tredline (Age Vs. Disease Y)
83 750
Number of patients for disease X

Number of patients for disease Y

73 650

63 550

53 450

43 350

33 250

23 150
12 27 42 57 72 87
Age

Figure 4.3: Scatter plot and Trend lines

Trend lines have slopes. If the slope moves up from bottom-left to top-right, it is called a
positive slope; whereas if it moves from top-left to bottom-right, it is called a negative
slope.

www.manhattanreview.com 48 19992016 Manhattan Review


Graphics Interpretation: Concepts & Examples 49

Correlation
We have seen Correlation in Table Analysis, but it is tested in GI too, so we reproduce its
definition again.
Correlationis a measure of relationships between any two entities. Two entities are posi-
tively correlated if both of them either increase in tandem or decrease in tandem, whereas
the entities are negatively correlated if one entity decreases in value while the other entity
increases in value, or vice versa. The entities are not correlated if the increase or decrease
in the value of one entity cannot be concluded relative to the increase or decrease in the
value of the other.
The Figure 4.4 shows the types and degrees of correlation.

Perfect positive correlation Perfect negative correlation

No correlation

Strong positive correlation Strong negative correlation

Figure 4.4: Degrees of Correlation

Probability

Basic Probability
As with Quants, the IR section also tests probability. The questions on probability
are based on the very basic concept of probability.

Number of occurrences of event A


Probability(Event A) =
Number of occurrence of all events
You must understand the meaning of all events. It is not always all the possible
events in the question, rather it is all the qualifying events.
So, if a question asks:
What is the probability of selecting a boy out of 10 boys, 20 girls, 3 male
teachers, and 2 female teachers; given that, in a single drawing, the selection
is to be made out of a set of only males?

10 10
P(Boys) = =
10 + 3 13
Going Other Way Around

19992016 Manhattan Review 49 www.manhattanreview.com


50 Graphics Interpretation: Concepts & Examples

Sometimes it is more time-consuming to calculate the probability of an event to


occur. In that case, it may be more convenient to calculate the probability of the
event not to occur and then deduce the probability of the event to occur.

Here is how:

P(event occurs) + P(event does not occur) = 1


P(event occurs) = 1 P(event does not occur)

If the question asks:


What is the probability of getting a yellow or green or red ball in a single
draw out of 5 yellow, 3 green, 4 red, and 2 black balls?
Instead of calculating the probability of getting a yellow or green or red ball, it
is better to calculate the probability of getting a black ball and then deduct the
calculated answer from 1.

2 2 1
P(Black) = = =
(5 + 3 + 4 + 2) 14 7

1 6
P(Yellow or Green or Red) = 1 =
7 7

www.manhattanreview.com 50 19992016 Manhattan Review


Graphics Interpretation: Concepts & Examples 51

4.3 Process of Solving GI Questions

(1) Understand the data set


Read the question narration and the elements of graph. Understand the relationship of
each element to the others. Sometimes legends are not given in the plot area. However,
you can get information from the question narration or some notes given outside the
plot area.

(2) Understand the question


As you know, that there are two Fill-In-the-Blank questions in a GI dataset. You have to
select the best option from a drop-down menu. Remember that there is no partial credit
for correct responses. Both responses must be correct to get the credit for the question.
Read the statement and stop to understand it. Mentally rephrase what is asked.

(3) Develop an approach


There could be two ways to approach a question. Most GI questions ask for a value in a
statement to be filled in. However, it is not necessary to actually calculate values every
single time. Even a logical approach can work and save your time.

(4) Apply the approach


Apply the approach you developed in previous step and select the correct option. Jot
down the results of the calculations, as it is likely that the results in previous question
may be required for intermediary steps in the question that follows.

19992016 Manhattan Review 51 www.manhattanreview.com


52 Graphics Interpretation: Concepts & Examples

4.4 Examples

Let us see some GMAT-like questions now. In an actual GMAT exam, you will have to select an
option in a drop-down menu, but in this book, we have just listed the options.

Example 1

The bar graph shows the number of unemployed youth in a country for seven years.

Unemployment
25000

20000
Unemployed

15000

10000

5000

0
2001 2002 2003 2004 2005 2006 2007

Based on the given information, use the drop-down menu to most accurately complete the
following statements:

(A) The greatest increase in the number of unemployed youth from any year to its succeeding
year is approximately .
(A) 3736
(B) 4152
(C) 6108
(D) 7542

(B) The number of unemployed youth in the year 2004 is approximately percent of the
number of unemployed youth in the year 2003.
(A) 16
(B) 19.5
(C) 84
(D) 119.5

www.manhattanreview.com 52 19992016 Manhattan Review


Graphics Interpretation: Concepts & Examples 53

SolutionPart A

The question asks you to determine the greatest absolute increase in the number of unem-
ployed from any year to its succeeding year.
Clearly, the answer would be one of the following years; 20012002, 20022003, 20042005,
and 20052006, as during the years 20032004, and 20062007, number of unemployed
youths in fact decreased.
It is not advisable to calculate the increase for all four probable years. So, which year would
have the maximum increase? Clearly, looking at the graph, you can see that the answer would
be one of years between 20012002 and 20022003, because these two years have the highest
jump over their previous year.
You see that the number of unemployed youths in the year 2001 is approximately 8000. You
need not be very precise. Read the value to your best of capability. Similarly, the number
of unemployed youths in the year 2002 is approximately 12000. The increase for the period
20012002 would be approximately 4000 ( 12000 8000). Likewise, for the period 2002
2003, the increase would be approximately 6000 ( 18000 12000). It is clear that the
answer would be 6000. The closest option is C, 6108.
The correct answer is option C.

SolutionPart B

Let the number of unemployed youth in the year 2004 = x% of unemployed youth in the
year 2003. You are supposed to calculate x.
Reading from the graph, you find that

unemployed2004 = 15000 and unemployed2003 = 18000.

Hence,

15000
 
x= 100% = 83.33%.
18000

The closest answer is 84%.


You must pay attention to the verbiage. The question does not ask you for percentage increase
or decrease. Had you made that mistake, the options A and B were the traps laid down for
you. Similarly, swapping 2003 with 2004 would also get you in trouble. Option D was another
trap: (18000/15000) 100% = 119.50%.
The correct answer is option C.

19992016 Manhattan Review 53 www.manhattanreview.com


54 Graphics Interpretation: Concepts & Examples

Example 2

The Venn diagram represents information about a group of 75 executives. The executives read
either one, two or all of the three newspapers: The Economics Times, The Wall Street Journal,
and Business Week.

= 5 Executives

The Economic Wall Street Journal


Times

Business week

Based on the given information, use the drop-down menus to most accurately complete the
following statements:

(A) An executive is randomly picked from a selected group of executives. The probability
that the executive reads only Business Week given that the selected group reads either
Business Week or The Wall Street Journal or both is .

(A) 0.27
(B) 0.31
(C) 0.50
(D) 0.87

(B) An executive is randomly picked from the group. The probability that the executive reads
Business Week or The Economic Times, but not The Wall Street Journal is .

(A) 1/15
(B) 1/5
(C) 7/15
(D) 4/5

www.manhattanreview.com 54 19992016 Manhattan Review


Graphics Interpretation: Concepts & Examples 55

SolutionPart A

You are to calculate the probability of selecting an executive who reads ONLY Business Week
among all the executives who read either Business Week or The Wall Street Journal, or both.

Number of executives who read only BW 4 4


P(Only BW ) = = = = 0.31.
Number of executives who read BW + W SJ both 15 2 13

See the figure given below.

Both ET & WSJ, = 5 Executives


but not BW

The Economic Wall Street Journal


Times

Only ET Only WSJ

Only ET & BW
Both BW & WSJ
Only BW
Business week

The correct answer is option B.


The value of number of executives who read BW + W SJ both alternatively could have been
calculated as 8 + 8 3 = 13, but that would be a lengthy calculation. We ignored the informa-
tion that one rhombus equals five executives, as it would have no impact on the calculation.
Multiplication by 5 in both the numerator and the denominator would have been cancelled out.

SolutionPart B

You are to calculate the probability of selecting an executive who reads anything except the WSJ.
The optimum approach would be to calculate the probability of selecting an executive who
reads the WSJ, and then deducting that value from 1:
8 7
P(only ET + only BW + both) = 1 P(W SJ) = 1 15 = 15 .

The correct answer is option C.

19992016 Manhattan Review 55 www.manhattanreview.com


56 Graphics Interpretation: Concepts & Examples

Example 3

The bubble diagram represents the performance of a Blue-Chip company. Its 6-year perfor-
mance is represented by six bubbles. Each bubble represents the ratio of revenue-to-profit.
(The figures are mentioned on the bubbles.) The horizontal axis represents profit ($K), and the
vertical axis, years of performance. The relative position of a bubbles center point indicates
profit on the X-axis, and the year of occurrence on the Y-axis.

Revenue to Profit Ratio


7
6 1.05

5 2.75
4 3.15
Years

3 1.26
2 2.85
1 1.85
0
1000 1500 2000 2500 3000
Profit ($K)

Based on the given information, use the drop-down menus to most accurately complete the
following statements:

(A) The least amount of expense observed in any year during a 6-year period is $ .

(A) 80K
(B) 520K
(C) 1250K
(D) 1600K

(B) The profit-to-expense ratio is the least for the year .


(A) 1
(B) 2
(C) 3
(D) 4
(E) 5
(F) 6

www.manhattanreview.com 56 19992016 Manhattan Review


Graphics Interpretation: Concepts & Examples 57

SolutionPart A and B

Understand the bubble chart first. You can get three data from it:

(1) Profit: the X-axis value of a bubbles center

(2) Year of occurrence: the Y-axis value of a bubbles center

(3) Ratio of revenue to profit: the number mentioned on a bubble

For part A, you have to find the least expense in any year of the 6-year period. Information
about expenses are not given in the graph. You can apply common knowledge here.

You know that:

Expense = Revenue Profit

Using this, you can find the value of the expense-to-profit ratio.

For example, for Year 1, revenue/profit = 1.85 or 1.85/1. For ratio purposes, you can assume
that revenue = 1.85, and profit = 1. This will give expense = 1.85 1 = 0.85. Or, the expense-
profit ratio is 0.85/1 = 0.85.

Similarly, you can calculate the values of the expense-profit ratio for all the years. The val-
ues for expense-profit ratios for all six years would be 0.85, 1.85, 0.26, 2.15, 1.75, and 0.05,
respectively.

For part B, you have to find out the year for which the profit-expense ratio is the least. The
correct answer would be the year for which expense-profit ratio is the highest. Referring to the
the calculated figures for expense-profit ratios (given above), you will find that the expense-
profit ratio is highest for Year 4; it equals 2.15.

Hence, option D is the correct answer for part B of the question.

Coming back to part A of the question, you have to find out the value of least expense that
occurred in any year of the 6-year period. You already know the value of expense-profit ratio.

Hence,

Expense = (Expense-to-Profit ratio) Profit

To get the least value of expense, the ideal year would be one that has the lowest value for its
expense-profit ratio, as well as its profit. Low values for the expense-profit ratio are for the
years 1, 3, and 6, while low values for profit are for the years 5, and 6. Clearly, the expenses
for year 6 is the answer for part A of the question. The expenses for year 6 = 0.05 profit.
We see that the approximate value of profit for year 6 = $1600K; thus he expenses for year 6 =
0.05 1600 = $80K.

If you are curious, here are the values of expenses for all six years:

19992016 Manhattan Review 57 www.manhattanreview.com


58 Graphics Interpretation: Concepts & Examples

Year 1 2 3 4 5 6

Expense-to-Profit Ratio 0.85 1.85 0.26 2.15 1.75 0.05

Profit ($K) 2400 1750 2000 3000 1250 1600

Expense ($K)
= (Expense-Profit ratio) 2040 3238 520 6450 2188 80
Profit

Hence, option A is the correct answer for part A of the question.

www.manhattanreview.com 58 19992016 Manhattan Review


Graphics Interpretation: Concepts & Examples 59

Example 4

In a town P, the natives are affected by two rare diseases X and Y . In the following scatterplot,
there are two sets of 21 points arranged vertically: one represents the comparison of age to
the number of patients of disease-X (black rhombus), and the other represents the comparison
of age to the number of patients of disease-Y (grey squares).

Age-wise number of patients


Age Vs. Disease X Age Vs. Disease Y
83 750

73 650
Number of disease X patients

Number of disease Y patients


63 550

53 450

43 350

33 250

23 150
12 27 42 57 72 87
Age

Based on the given information, use the drop-down menus to most accurately complete the
following statements:

(A) The lowest number of patients for disease X lie in the between age group .

(A) 12 and 27
(B) 27 and 42
(C) 42 and 57
(D) 57 and 72
(E) 72 and 87

(B) The correlation between the patients of disease X and those of disease Y is .
(A) a strong positive
(B) a strong negative
(C) negligible

19992016 Manhattan Review 59 www.manhattanreview.com


60 Graphics Interpretation: Concepts & Examples

SolutionPart A

You have to find the age group for which the total number of patients with disease X (black
rhombus) is the least. If you did not pay attention to details, you may do this seemingly easy
question wrong. Remember that the weight for each rhombus is different. The more you go up
the Y-axis, the higher the weight of a rhombus is. It means that two rhombuses lying between
the 7287 age group collectively may weigh more than the four rhombuses lying between the
1227 age group.
Had you attempted this question hurriedly, you would have marked option E as the correct
answer.
How do you approach this question? Should you read the value of the weight for each rhombus,
add them for each age group and compare the total? Answer is No. You cannot afford to do
this in the limited amount of time. GMAT never throws you a question that involves a lot of
calculation. Even if you have the luxury of using the calculator provided in the IR section, it is
not a wise approach to do the computation involving 21 values.
This question demands a logical approach. It is clear that the answer would NOT be among age
groups: 2742, 4257, or 5772 since each age group has either 4 or 6 rhombuses. Comparing
these age groups with the 1227 age group, you can safely infer that number of patients for
the 1227 age group would be the least as the age group 1227 also has 4 rhombuses. Note
that rhombuses lying in age group 1227 represent relatively less number of patients than the
ones lying in age groups: 2742, 4257, or 5772.
So, you have only two contenders: the 1227 and the 7287 age groups. Now must you cal-
culate values now? Well, not really. You can estimate that the average value for the four
rhombuses lying in the 1227 age group is around 33. So, the total number of patients for the
1227 age group 33 4 132. As far as the total number of patients for the 7287 age
group is concerned, it is 73 + 73 = 146. Clearly, the number of patients for 1227 age group is
the least in value.
The correct answer is option A.

www.manhattanreview.com 60 19992016 Manhattan Review


Graphics Interpretation: Concepts & Examples 61

SolutionPart B

We assume that you have gone through the theory part of correlation. Now, how do you
approach this question? Look at the image given below.

Age-wise number of patients


Age Vs. Disease X Age Vs. Disease Y
Linear (Age Vs. Disease X) Linear (Age Vs. Disease Y)
83 750

73 650
Number of disease X patients

Number of disease Y patients


63 550

53 450

43 350

33 250

23 150
12 27 42 57 72 87
Age

First, imagine a trend line that is the best fit for all the black rhombuses. Similarly, do the same
for the grey squares. You will observe that the trend line for disease X has a positive slope
(that is, it moves up from bottom-left to top-right), while that for disease Y has a negative slope
(that is, it moves down from top-left to bottom-right). Which means that while the number of
patients for disease X increases with the age, it decreases for disease Y , implying a strong
negative correlation.
The correct answer is option B.

19992016 Manhattan Review 61 www.manhattanreview.com


62 Graphics Interpretation: Concepts & Examples

Example 5

The chart given below shows a corporate strategy map of a customer services company. The ar-
rows connecting the ovals show how various entities are interconnected at the company level.

Corporate Strategy Map


Customer

Strive for Improve


Facilitate
customer stake-holder
delight choices
goodwill
Financial

Improve price More cost Increase net


competitiveness control revenue

Improve customer
More control
administration
Processes

over quality
process

Improve Improve
safety of marketing &
plant sales
Development

Improve Improve
CRM
Learning &

knowledge of
knowledge customers

Improve Increase
knowledge of production
management capacity

Based on the given information, use the drop-down menus to most accurately complete the
following statements:

(A) If the company cannot Increase production capacity, it can still achieve More control
over cost by .

(A) improving knowledge of management


(B) facilitating more choices
(C) improving marketing and sales
(D) improving CRM knowledge

(B) By improving , company can enhance Customer delight.

www.manhattanreview.com 62 19992016 Manhattan Review


Graphics Interpretation: Concepts & Examples 63

(A) Production capacity


(B) Marketing and sales
(C) CRM knowledge
(D) Stakeholders goodwill

SolutionPart A

The given chart is called a Strategy map or a Visual chart. In this chart, one activity leads
to one or more activities represented by the arrows.

In part A of the question:

If the company cannot Increase production capacity, it can still achieve More con-
trol over cost by .

Your objective is to achieve More control over cost, but the constraint is that the company
cannot Increase production capacity.

Objective: achieve More control over cost.

Constraint: the company cannot Increase production capacity.

Let us analyze each option.

(A) Improving knowledge of management leads to Improving safety of plants, which leads
to Improvement in net revenue. However, this does not lead to the objective of achieving
More control over cost.

(B) The objective of achieving More control over cost leads to Facilitating more choices,
not vice-versa.

(C) Improving marketing and sales leads to two activities:

(a) Increasing net revenue (but this does not lead to More control over cost)
(b) Improving the stake holders goodwill (but this does not lead to More control over
cost)

(D) Improving CRM knowledge leads to Improving the customer administration process,
which leads to the objective: More control over cost. This is the correct answer.

The correct answer is option D.

19992016 Manhattan Review 63 www.manhattanreview.com


64 Graphics Interpretation: Concepts & Examples

SolutionPart B

Objective: Enhance customer delight


Let us analyze each option:

(A) Improving production capacity does not lead to the objective of Enhancing customer
delight.

(B) Improving marketing and sales leads to two activities:

(a) Increasing net revenue; but this does not lead to the objective of Enhancing cus-
tomer delight.
(b) Improving stake holders goodwill; but again, this does not lead to the objective of
Enhancing customer delight.

(C) Improving CRM knowledge leads to improving the Customer administration process,
which leads to the objective: More control over cost. This is the correct answer.

The correct answer is option C.

www.manhattanreview.com 64 19992016 Manhattan Review


Graphics Interpretation: Concepts & Examples 65

Example 6

The PE Corporation has four warehouses that stock electric bulbs. The line-graph presents the
number of cartons stocked by the first two warehouses, and an average stock of all the four
warehouses for the months September to June.

Warehouse Stock
Average Warehouse 1 Warehouse 2

210

200
Units ('000 cartons)

190

180

170

160

150

Based on the given information, use the drop-down menus to most accurately complete the
following statements:

(A) The highest number of electric bulb cartons stocked by the other two warehouses (ware-
house 3 and 4) was in .
(A) October
(B) December
(C) February
(D) February and in May
(E) May

(B) The lowest number of electric bulb cartons stocked by the other two warehouses (ware-
house 3 and 4) was in .
(A) November
(B) December
(C) December and in May
(D) May
(E) June

19992016 Manhattan Review 65 www.manhattanreview.com


66 Graphics Interpretation: Concepts & Examples

SolutionPart A

Let us understand the question. The PE Corporation has four warehouses. In the graph, the
black line represents stocks for Warehouse 1, the dashed-line for Warehouse 2, and the grey
line for the averages of all the four warehouses.
Note there is no information about Warehouses 3 and 4. However, you still can find informa-
tion about their stocks.
As you know,

Stocks for WH 3 + WH 4 = Total stocks of all 4 warehouses


(Sum of WH 1 and WH 2 stocks)

Stocks for WH 3 + WH 4 = 4 Average stock of all 4 warehouses


(Sum of WH 1 and WH 2 stocks)

Look at the first part of the question:

The highest number of electric bulb cartons stocked by the other two warehouses
(warehouse 3 and 4) was in .

Looking at the equation, you can deduce that

WH 3 and WH 4 stocks = 4 average stock WH 1 stock WH 2 stock.

You can infer that to get the highest value for WH 3 and WH 4 stocks, the average stock value
must be the highest, as it would increase their values, whereas the WH 1 and WH 2 stocks
should be the lowest, as these otherwise would decrease their values to a minimum.
Instead of calculating the values for WH 3 and WH 4 stocks for all the given months in the
options, you can analyze the options by observing the graph.

www.manhattanreview.com 66 19992016 Manhattan Review


Graphics Interpretation: Concepts & Examples 67

Value of Value of Value of


Month Average Stock WH 1 stock WH 2 stock Remarks
(Ideal Highest) (Ideal Lowest) (Ideal Lowest)

October Too high High Highest October cannot be


(Positive for (Negative for (Most negative the answer, as the
WH 3 and WH 4 WH 3 and WH 4 for WH 3 and two factors are
to be highest) to be highest) WH 4 to be negative.
highest)

December Too low Too low Too low December can be


(Negative for (Positive for (Positive for eliminated as
WH 3 and WH 4 WH 3 and WH 4 WH 3 and WH 4 compared to
to be highest) to be highest) to be highest) February. It still has
one negative factor.

February Highest Lowest Mid-way February is the


(Most positive (Most positive (Neutral) correct answer. Out
for WH 3 and for WH 3 and of three factors,
WH 4 to be WH 4 to be two are the most
highest) highest) positive, while the
third one is neutral.

May Too high Highest Too high May cannot be the


(Positive for (Most negative (Negative for answer, as the two
WH 3 and WH 4 for WH 3 and WH 3 and WH 4 factors are negative.
to be highest) WH 4 to be to be highest)
highest)

So, the correct answer is C.


If you are curious, here are the stocks for all months.
WH 3 and WH 4 stocks for Feb. = 4 205 160 180 = 820 340 = 480.
Similarly, WH 3 and WH 4 stocks for October, December, and May months are: 415, 365, and
410, respectively.
The following image can help you understand how to do the above analysis mentally.

19992016 Manhattan Review 67 www.manhattanreview.com


68 Graphics Interpretation: Concepts & Examples

Average- Highest;
Average- High, but not the highest; Impact Positive; 4 times
Impact Positive; 4 times Warehouse Stock
Average Warehouse 1 Warehouse 2

210
WH2- Midway;
200 Impact No impact

Units ('000 cartons)


190
WH2- Highest;
Impact Negative 180

170

160

150
WH1- Midway;
Impact No impact

WH1- Lowest;
Impact Positive; 1 time

SolutionPart B

Look at the first part of the question:

The lowest number of electric bulb cartons stocked by the other two warehouses
(warehouse 3 and 4) was in .

This is a similar question to the previous one. You have to find the month that stocked the
lowest number of cartons in WH 3 and WH 4, instead of the highest.
The strategy would be opposite of what you applied in the previous question.
Looking again at:

WH 3 and WH 4 stocks = 4 average stock WH 1 stock WH 2 stock,

you can infer that to get the lowest values for WH 3 and WH 4, the average stock must be
the lowest, as that would decrease the values, whereas WH 1 and WH 2 stocks should be the
highest, as those, too, would decrease the values.
Inferring the probable values for the option monthsNovember, December, May, and June
you can infer that June is the answer, as the average stocks for June is the lowest of all the
months, while the stocks for WH 1and WH 2 are high.
So, the correct answer is E.
If you are curious, here are the stocks for all months.

www.manhattanreview.com 68 19992016 Manhattan Review


Graphics Interpretation: Concepts & Examples 69

WH 3 and WH 4 stocks for June = 4 170 190 190 = 680 380 = 300.
Similarly, the values for November, December, and May are: 380, 365, and 410, respectively.

Take Away: Remember that if the average stock is the most positive for a month, its effect
is four times that of each of WH 1 and WH 2 stocks.
Have a look at the following figure for more clarity.

Compared to the data for the month of June, Average is


High, and WH 1 & 2 are low. Collectively, the value in Dec.
for WH3 & 4 would be greater than that for the month of
June.
WH 1 & 2- High;
Warehouse Stock Impact Positive

Average Warehouse 1 Warehouse 2

210

200
Units ('000 cartons)

190

180

170

160

150

Average- Lowest, ;
Impact Positive; 4 times

19992016 Manhattan Review 69 www.manhattanreview.com


70 Graphics Interpretation: Concepts & Examples

Example 7

The bar chart shows the percentage distribution of expenditures for three families: A, B, and C,
on the primary vertical axis. The expenditure is categorized into four categories: food, cloth-
ing, home, and entertainment. In addition, the expenditures of individual families represented
by line graph are plotted (in dollars) on the secondary vertical axis.

% Distribution of expenditure
Food Clothing Home Entertainment Expenditure

100% 3750

90%
3500
80%

70% 3250

Expenditure($)
60%
3000
50%
2750
40%

30% 2500
20%
2250
10%

0% 2000
Family A Family B Family C

Based on the given information, use the drop-down menus to most accurately complete the
following statements:

(A) Family A needs to decrease its expenditure on their home by to match that of
family B.

(A) 25.00%
(B) 28.11%
(C) 47.84%
(D) 62.50%

(B) does not spend the most among the three families in any of the four categories.

(A) Family A
(B) Family B
(C) Family C

www.manhattanreview.com 70 19992016 Manhattan Review


Graphics Interpretation: Concepts & Examples 71

SolutionPart A

The expenditure on its home by a family would be given by the product of percentage expen-
diture on its home expenditure (in dollars).

Expenditure on its home by family A = 40% ( 2500) = $1000.


Expenditure on its home by family B = 15% ( 3600) = $540.

The percentage decrement in expenditure on its home by family A to match that by family B
equals:

expenditure by family A expenditure by family B (1000 540)


100% = 100% 46%.
expenditure by family A 1000

The closest option is 47.85%.


The correct answer is C.

SolutionPart B

The question wants you to find a family that does not spend the most in any of the categories:
food, home, clothing, and entertainment.
Looking at the graph above, you see that there is only one family that does not spend the most
in any category. Observe that family B has the highest expenditures among all three families,
and for entertainment, it spends the most (35%). Since family B spends the most in at least
in one category, option B is ruled out as an answer, and the correct one is between families A
and C.
In Part A of the question, you have already calculated how much family A spends on its home.
Its expenditure ($1000) is more than that for family B ($540). You should also calculate the
figure for family C. Expenditure on its home by family C = 25% 2900 = $725. So we
conclude that family A spends the most in the home category.
This implies that the correct answer must be family C.
The correct answer is option C.

19992016 Manhattan Review 71 www.manhattanreview.com


72 Graphics Interpretation: Concepts & Examples

Example 8

The graph given below shows 60-monthly data for the mean deviation of Brazilian currency,
the Real, observed for the years 20082013. The mean deviation of the Real is measured by
the current months rate vs. the U.S. Dollar less the five-year mean rate vs. the U.S. Dollar. The
graph also shows a line of best fit for the data points.

Mean deviation of Brazilian Real


8
Difference from 5-year mean rate

6
4
2
0
-2
-4
-6
-8
-10
-12
2008 2009 2010 2011 2012 2013

Based on the given information, use the drop-down menus to most accurately complete the
following statements:

(A) To conclude that mean value of the Brazilian Real is decreasing in the international mar-
ket, it would be helpful to see a more comprehensive graph of the same period that
shows .
(A) weeks on the horizontal axis
(B) a broader range for the vertical axis
(C) a similar graph for the mean deviation of the Brazilian Real versus other popular
currencies in the international market
(D) a similar graph for the mean deviation of the U.S. Dollar versus the Brazilian Real

(B) The conclusion that the mean value of Brazilian Real has decreased over the years would
be most strengthened if the set of measurements in the year were
revised towards the current line of best fit.
(A) 2008
(B) 2009
(C) 2010
(D) 2011
(E) 2012

www.manhattanreview.com 72 19992016 Manhattan Review


Graphics Interpretation: Concepts & Examples 73

SolutionPart A

What does it mean that the mean value of Brazilian Real is decreasing in the international
market? The questions wants you to make sure that the Real is depreciating in the international
market. The graph shows the mean deviation of the Real only versus U.S. Dollar currency. The
trend line or the current line of best fit for mean deviation shows that the slope is negative
implying that the mean value of the Real is decreasing in the international market.

(A) Having a more comprehensive graph of the same period that shows weeks on the hor-
izontal axis would not help, as it would not impact the trend line. The 60 data points
plotted on the graph would not be impacted, either.

(B) By the similar reasoning as in option A, seeing the more comprehensive graph of the
same period that shows broader range for the vertical axis would not help, as it would
not impact the trend line.

(C) It could be possible that the Real is not devaluating, but the U.S. Dollar is appreciating
because of a reviving U.S. economy. So, seeing a similar graph for the mean deviation of
Brazilian Real versus other popular currencies in the international market would make
sure the Real is decreasing.

(D) A similar graph for the mean deviation of the U.S. Dollar versus the Real is nothing but
the same graph with inverted values. It does not provide any additional information.

The correct answer is option C.

SolutionPart B

First, understand the meaning of the question: it wants you to verify the claim that the mean
value of the Brazilian Real has decreased over the years. The trend line has a negative slope,
but it is not very pronounced. Had the slope been more steep, you could have safely concluded
that the Real had been decreasing in value.
So, to strengthen the claim, you must increase the magnitude of the slope of trend line. How
do you do this? Well, you should bring up the data-points above the trend line closer to itself.
The years 2008, 2010, and 2012 have a good number of points below the trend line. Bringing
these data points closer to the trend line would rather decrease the magnitude of its negative
slope, but you do not want this.
Since you can alter the data points for ONLY one year (see the options), you must choose the
year whose collective deviation of 12 data points is more than that of others. Both the years
2009 and 2011 have data points lying significantly above the trend line. You would observe
that the year 2011 was the best year in terms of the economy as the mean deviations for as
many as seven months were positive (seven rhombuses are above the major grid-line of 0),
and negative for only three months (three rhombuses are below the major grid-line of 0). If
you move these points toward the trend line, it would strengthen the claim that the Real has
decreased over the years.
Currently, more data points of the year 2011 than of the year 2009 are working against your
goalBrazilian Real has decreased over the years, so moving the data points of 2009 though

19992016 Manhattan Review 73 www.manhattanreview.com


74 Graphics Interpretation: Concepts & Examples

would also strengthen the claim, it would not be to the extent the data points of year 2011
would do.
The correct answer is option D.
Look at the graphic given below and notes for more clarification.

2 points
increase
6 points
the slope
Mean deviation of Brazilian Real
increase
7 points
3 points the slope
decrease
increase
8
the slope
the slope,
and 4 points
Difference from 5-year mean rate

6 increase the
slope.
4
2
0
-2
-4 9 points
increase
-6 the slope,
4 points and 1 point
-8 9 points decrease decrease
decrease the slope
-10 the slope.
the slope
-12
2008 2009 2010 2011 2012 2013

7 points
decrease
the slope

www.manhattanreview.com 74 19992016 Manhattan Review


Graphics Interpretation: Concepts & Examples 75

Example 9

The graph represents data on the non-vegetarian foods habits of different age group natives
living in a country. The graph shows the percentage of natives preferring vegetarian food,
and the percentage of non-vegetarians preferring varieties of non-vegetarian foods across all
groups.

Age-wise % distribution of non-vegetarian


food preference
Chicken Fish Red Meat
Pork Sea Food Only Vegetarian
80

70

60
Percentage

50

40

30

20

10

0
5-10 10-20 20-30 30-40 40-50 50-60 60+
Age-Group

Based on the given information, use the drop-down menus to most accurately complete the
following statements:

(A) Approximately % of the natives in the country belonging to 4050 age group eat
red meat.
(A) 22
(B) 44
(C) 50
(D) 88

(B) The greatest percentage increase in the number of vegetarians calculated from one age
group to the subsequent is %. (Assume that the population is same for all age
groups.)

(A) 36
(B) 48
(C) 100
(D) 120

19992016 Manhattan Review 75 www.manhattanreview.com


76 Graphics Interpretation: Concepts & Examples

SolutionPart A

In the 4050 age group, 50% natives prefer vegetarian food, hence the same percentage of
natives, i.e., 50% prefer non-vegetarian food. Out of these 50% non-vegetarian natives, 44%
prefer red meat. It means that 50% of 44% = 22% natives prefer red meat.

The correct answer is option A.

SolutionPart B

The percent increase from one age group to the succeeding one is calculated as:

vegetarians in the nth age group vegetarians in the (n 1)th age group
100%
vegetarians in the (n 1)th age group

or

(% vegetarians in the nth age group population)(% vegetarians in the (n1)th age group population)
% vegetarians in the (n1)th age group population
100%

Since the population is same for all the age groups, we can rewrite the above as:

% vegetarians in the nth age group % vegetarians in the (n 1)th age group
100%
% vegetarians in the (n 1)th age group

It would be time-consuming to calculate the percent increase of vegetarians for all groups,
hence not advisable. You can observe the graph to shortlist the probable contenders.

Looking at the formula above, you can infer that to get the greatest value the numerator, or
the difference of percentages between two succeeding age groups, should be high, while the
denominator, or the percent value for the predecessor age group, should be low.

Based on this analysis, you will find that the answer is the 3040 age group as the percent jump
in the number of vegetarians from the age group 2030 to 3040 age group is 33% 15% =
18%. This is the highest numerator that any of the groups have. Although the numerator
( 18%) of 3040 age group is equal to that of age group 4050, denominator of age group
4050 is 33%, which is greater than for age group 2030 (15%). So, the answer would be:

18
100% = 120%.
% 15%

The correct answer is option D.

If you are curious, here are the values of percent increase for all age groups:

www.manhattanreview.com 76 19992016 Manhattan Review


Graphics Interpretation: Concepts & Examples 77

Age Group 510 1020 2030 3040 4050 5060 60+

% Value for age group 9 11 15 33 50 63 70


% Increase over 119 1511 3315 5033 6350 7063
= 9 11 15 33 50 63
the previous group (100%)

= 22% 36% 120% 51% 29% 11%

19992016 Manhattan Review 77 www.manhattanreview.com


78 Two-Part Analysis: Concepts & Examples

www.manhattanreview.com 78 19992016 Manhattan Review


Chapter 5

Two-Part Analysis

5.1 Strategies and Concepts

Out of four parts in the IR sectionthe GI, TA, MSR, and Two-Partyou will find the Two-
Part section more familiar than the others. However, you have to answer two parts in each
questionas opposed to one part in Quantitative Ability and Verbal Ability questions.
There are three types of questions asked in Two-Part part:

(1) Quants-based

(2) Logic-based

(3) Critical-Reasoning-based

A Two-Part question ask two aspects of a question. The answers to both parts must be com-
patible with each other.

5.1.1 Two-Part Quants

Two-Part-Quant questions are more focused than one-part GMAT Quant PS/DS questions,
which are more trickier and vaster in scope. Two-Part Quant questions have comparatively
lengthy word problems that focus on basic arithmetic and preliminary algebra. You must
possess good problem-solving skills to solve them.
A word of caution: there are two answers, so be careful to mark each part with its respective
answer.
Those who are good at Quants will find Two-Part Quant-based questions relatively easy. The
key is to prepare well for Quants.

5.2 Process of Solving Two-Part Quant Questions

(1) Understand the question

79
80 Two-Part Analysis: Concepts & Examples

Solving Two-Part-Quant questions is similar to solving any Quant questions:


Read the information and understand it; translate it into mathematical language, and
write it on the scratch board.

(2) Develop an approach


There could be two or more ways to approach a question. Many times, solving one part
leads to the solution of the other part.

(3) Apply the approach


Apply the approach you developed in the previous step. There may be a question where
you end up with only one linear equation with two variables. Though here, there is no
unique solution possible for the question, but many consistent and compatible solutions
are always possible. Only one such consistent and compatible solution would be given in
the options.
Your last step is to click correct radio buttons.

5.2.1 Two-Part Logic Games

Two-Part Logic-based questions are like those you might have come across in the LSAT or other
aptitude tests. The narration of these questions is comparatively lengthy, and involves a mix
of quantitative and verbal skills.
This book contains a couple of good logic-based questions.
The process of solving logic-based questions is similar to that of quant-based questions, except
that you have to think twice.

5.2.2 Two-Part Critical Reasoning

Two-Part CR-based questions are like Critical Reasoning questions you come across in the
GMAT Verbal section. The questions may test your critical reasoning skills by asking about
two aspects of widely known concepts such as assumption, strengthen and weaken, evaluate,
strategize, and role play.
In a typical Verbal-CR question, if you are asked to strengthen the argument, you can simply
ignore the options that weaken the argument. However, in Two-Part CR-based questions, you
will be asked to both strengthen and weaken the argument. Out of 56 option choices, 23 of
them may strengthen the argument, while the other 23 options may weaken the argument.
Nevertheless, compared to Verbal-CR questions, Two-Part CR-based questions are relatively
easy. There might be only 23 options for each aspect, as opposed to Verbal-CR questions, in
which all five options may be crafted to test only one aspect.
Those who are good at answering Verbal-CR questions will find the Two-Part CR section rela-
tively easy. The key is to prepare well for the Verbal-CR section.
Let us look at some GMAT-like questions now.

www.manhattanreview.com 80 19992016 Manhattan Review


Two-Part Analysis: Concepts & Examples 81

5.3 Examples: Two-Part Quant-based Questions

Example 1

For a right circular cylinder, r stands for the radius of its base, h for its height, A for its curved
surface area, and V for its volume. In terms of the variables A and V, select in the table the
expressions that represent height h and radius r .
Make only two selections, one in each column.

A B

r h

A2
A 4 V
2V
B A
A2 V
C 8
2A
D V
A
E V

SolutionPart A

Let us understand the question. This is a variable-based two-part question on solidscylinders.


The question tests your knowledge about relationships between the radius of the base, height,
surface area, and volume of a cylinder. You have to select the expressions in the table that
represent its height h and radius r only in terms of volume V and curved surface area A.
An approach would be to write down the formulas for the curved surface area and volume
of a cylinder, then manipulate them to get r and h in terms of volume V and curved surface
area A.
First, shortlist the options to probable ones, or eliminate a few non-probable ones. Here,
though, there is no such option that can be easily eliminated. So, follow the next step:
Write down the formulas for curved surface area and volume. The formulas are:

Curved surface area, A = dh = 2 r h


d2 h
Volume, V = = r 2h
4

r
Observe the formula for the volume, V = r 2 h. If you rewrite it as V = 2 r h , you find
2
that you can replace the term 2 r h with the curved surface area A of a cylinder. So, replace
it with A.

19992016 Manhattan Review 81 www.manhattanreview.com


82 Two-Part Analysis: Concepts & Examples

Ar 2V
Hence, the relationship becomes V = r = .
2 A
So the answer for the first part is B.

SolutionPart B

As stated in previous question, observe the given options to shortlist probables, or eliminate a
few options. However, again in this part, there is no such option that can be easily eliminated.
So, follow the next step:
Rewrite the formulas of curved surface area and volume. The formulas are:

Curved surface area, A = 2 r h


Volume, V = r 2 h

Manipulate the formulas to get h only in terms of A and V .


A
Observe the formula for the curved surface area, A = 2 r h. If you rewrite it as r = h, and
2
plug the value of r into the formula for V , then the relationship becomes:

!
A2
V = h
4 2 h2

A2 A2
After the calculation, it becomes V = 4 h or h = 4 V .

So the answer for the second part is A.


The correct answers are options B and A.

www.manhattanreview.com 82 19992016 Manhattan Review


Two-Part Analysis: Concepts & Examples 83

Example 2

For positive values of n, M is defined such that

2Mn+2 + (Mn )2 Mn+1 = 1.

1
Also given that M6 = . Select the values of M4 , and M5 in the table such that they are jointly
2
compatible with these conditions.

Make only two selections, one in each column.

A B

M4 M5

A 2

B 2

C 1

D 2

E 4

SolutionBoth Parts

Let us understand the question. The equation given is a quadratic equation with three vari-
ables. The value of one variable is given. You have to find the jointly compatible values of the
other two variables.

Since the equation is a generic equation, given for any three successive applicable variables,
you must approach the question such that you are able to build a relationship among M4 , M5 ,
and M6 .

Observe the given options to shortlist probable or eliminate a few non-probable options. How-
ever, there is no option that can be easily eliminated because the desired equation(s) are not
yet established. Once you form the equation(s), you can then eliminate options.

Follow the next step: Establish a relationship only among the variables M4 , M5 , and M6 .

As discussed previously, 2Mn+2 + (Mn )2 Mn+1 = 1 is a relationship among three successive


terms, so to establish the relationship among M4 , M5 , and M6 , consider n = 4.

By plugging in n = 4, you get 2M6 + (M4 )2 M5 = 1

Again, by plugging in the value of M6 = 12 , you get:

19992016 Manhattan Review 83 www.manhattanreview.com


84 Two-Part Analysis: Concepts & Examples

 
1
2 2 + (M4 )2 M5 = 1
(M4 )2 M5 = 0
q
(M4 )2 = M5 or M4 = M5 .

The above relationship has two variables, so a unique solution is not possible. However, you
can still get consistent solutions. The question only asks for a set of compatible values of M4
and M5 present in the table.
By hit and trial, you can find a set of compatible values from the table: M4 = 2 and M5 = 4,
so the correct answers for both the parts are options A and E respectively. Mark answers in
the correct order; marking answers E and A would be wrong.
The correct answers are options A and E.

Point to Remember

Unique Solutions vs. Consistent Solutions

Say you have an equation: x + 2y = 8. It has two variables, x and y. If you plug in x = 2 in
the equation, you get y = 3. Is that a solution to the equation? Well, Yes!
Now, if you plug in x = 4, you get y = 2. Is it a solution to the equation? Well, Yes! Similarly,
you can plug any number for a variable and get the corresponding value of the other variable.
Both the solutions (x = 2, y = 3 and x = 4, y = 2) are consistent solutions to the equation
x + 2y = 8, however they are NOT unique solutions.

What is a unique solution?

Well, to get the unique solution, if the equation has two variables, then you need two simulta-
neous equations. In general, if an equation has n variables, you need n independent equations
to get a unique solution.
So, suppose you bring in another equation: 2x + y = 7. Solving both the equations simultane-
ously, you get x = 2 and y = 3. That is a unique solution, as each variable can have only one
value.

www.manhattanreview.com 84 19992016 Manhattan Review


Two-Part Analysis: Concepts & Examples 85

Example 3

Suppose you have the following list of executives in a companys departments:

Department 1: Robin, Betty, Yasin, Tom

Department 2: Ponting, Tony, Olga

Department 3: Palvi, George, Brown, Steve, Jack

Department 4: Balki, Whitney, Silvy

The HR department of the company is selecting executives from the above departments to
form three new teams consisting of four executives in each team. For each team, no more than
two executives can be selected from one department, and executives from no more than three
departments can be selected. Each team must have at least a manager-level executive. Betty,
Palvy, and Brown are manager-level executives.

The first three executives selected for each of the teams are:

Team 1 Team 2 Team 3

Yasin Palvy George

Ponting Steve Brown

Silvy Robin Tony

Select an executive that could be selected as the fourth executive for all three teams, and select
an executive that could not be selected as the fourth executive for any of the teams.

A B

Can be selected Cannot be selected

A Olga

B Jack

C Betty

D Balky

E Tom

19992016 Manhattan Review 85 www.manhattanreview.com


86 Two-Part Analysis: Concepts & Examples

SolutionBoth Parts

Analyze each option and apply a process of elimination.

(A) Olga: Olga cannot be in Team 1, as it lacks a manager and Olga is not the one. The only
executive, Team 1 can have is Betty (a manager). She can also be in Team 2 or 3.

(B) Jack: Jack cannot in be Team 1, as it lacks a manager and Jack is not the one. Neither can
be be in Team 2 as Palvy and Steve are already there from his department nor can be be
in Team 3, as George and Brown are already there from his department. (There cannot
be more than two executives from a department.)
So, Jack is the answer for second part, Cannot be selected.

(C) Betty: We concluded in the option A that Betty must be in Team 1. Betty can also be in
either Team 2 or Team 3.
Betty is the correct answer for the first part, Can be selected.

The correct answers are options C and B.

www.manhattanreview.com 86 19992016 Manhattan Review


Two-Part Analysis: Concepts & Examples 87

Example 4

Working individually, Alex, Betty, and Cherry can do a job in 8, 24, and 48 hours, respectively.
It was decided that each of them will work on the job successively for an hour. Anyone can
start the work and others will follow. Select in the table Minimum hours and Maximum
hours required to complete the job.
Make only two selections, one in each column.

A B

Minimum hours Maximum hours

A 15

B 15.5

C 16.17

D 17

E 17.33

SolutionBoth Parts

You must identify that Alex is the fastest worker, finishing 1/8 of the work in an hour, while
Cherry is the slowest worker, finishing 1/48 of the work in an hour.
Also, note that Anyone can start the work and the others will follow. It means that anybody:
Alex, Betty, or Cherry can start or finish the job.
When Alex, Betty, and Cherry each work, in that order, for an hour, then in a total of three
hours, they accomplish:

th
1 1 1 9 3

+ + = = of the job.
8 24 48 48 16
3 15
After five rounds, in a total of 5 3 = 15 hours, they complete 16 5 = 16 of the job. So,
15 1
1 16 = 16 of the job remains.
To find the maximum time to do the job, choose the slowest worker (Cherry) to take the next
 th
1
turn. She does another 48 of the work in an hour. Then choose the next slower worker,
 th
1
Betty, to do another 24 of the job in the next hour. This covers the remaining part of the
job after the initial five rounds:

1 1 3 1
+ = = = remaining job
48 24 48 16

19992016 Manhattan Review 87 www.manhattanreview.com


88 Two-Part Analysis: Concepts & Examples

So, the maximum time to do the job would be 15 + 1 + 1 = 17 hours. The answer for the second
part of the question is option D, 17 hrs.
To find the minimum time to do the job, after the initial five rounds, select the fastest worker,
 th
1 1
Alex, to take the next turn. He would need just 2 an hour to do the remaining 16 of the
job.
So, the minimum time to do the job is 15 + 0.5 = 15.5 hours. The answer for the first part of
the question is option B, 15.5 hrs.
The correct answers are options B and D.

www.manhattanreview.com 88 19992016 Manhattan Review


Two-Part Analysis: Concepts & Examples 89

5.4 Examples: Two-Part Verbal-based Questions

Example 5

Railway Safety Commissioner: The Railways Ministry claims that the railways safety lapses
have been caused by my policies and that I am responsible for the lapses. Although I admit
that the railways has suffered a couple of fatal accidents during my tenure, I do not agree
that I am at fault. The safety policies of the previous Railway Safety Commissioner are to
blame, and had it not been for the safety policies of my administration, the current safety
system would have been even worse.
In the argument above, the two phrases in boldface play specific roles. Select the appropriate
Boldface 1 or 2 to be consistent with the role it plays in the argument.
Make two selections, one in each column.

A B

Boldface 1 Boldface 2

A is a consequence of the Railway Ministrys claims.

B is the main conclusion

C supports the conclusion

D is a fact, which the Railway Safety Commissioner


believes does not contradict his own conclusion

E is evidence of unethical activity on the part of the


Railway Safety Commissioner

SolutionBoth Parts

Understanding the argument


The Railway Ministry claims that the current Railway Safety Commissioners safety policy is the
reason for security lapses. However, the Railway Safety Commissioner contradicts the claim.
Railway Safety Commissioners position: I am not responsible for it (security lapses).
Although he admits that two major accidents occurred during his tenure, he reasons that his
predecessors policies were to blame and not his. Furthermore, he strengthens his position by
stating that had his own policies not been implemented, the situation could have been worse
than what it now is.
Conclusion: I (Railway Safety Commissioner) am not at fault for this problem.
Let us dissect the argument and understand the positions of Ministry and the Railway Safety
Commissioner:

19992016 Manhattan Review 89 www.manhattanreview.com


90 Two-Part Analysis: Concepts & Examples

Ministry: the current Railway Safety Commissioners policies are at fault.

Railway Safety Commissioner: I am not at fault. I admit that a problem occurred during
my tenure, but my predecessor is at fault.

Boldface 1: is a fact because of which the Railway Ministry considers the Railway Safety
Commissioner liable.

Boldface 2: is the Railway Safety Commissioners opinion, with which he strengthens his
position and supports the conclusion.

Answer Choice Explanation:

Of the given options, the Boldface 1 phrase is an admission of a fact that the Railway Safety
Commissioner believes does not contradict his conclusion, while Boldface 2 supports his con-
clusion of I am not at fault.
The correct answers are options D and C.

www.manhattanreview.com 90 19992016 Manhattan Review


Two-Part Analysis: Concepts & Examples 91

Example 6

Consultant: The decline in market share of the Whito 3-kg detergent pack is a matter of
concern. There could be many predictive reasons. The competing brand, EcoWash, is always
on the lookout to gain Whitos substantial market share. Also, due to adverse judgments in
consumer courts against WhiteMagic detergentthe brand that kept EcoWashs detergent sales
in checkWhiteMagics sales are suffering, providing an opportunity for EcoWash. Usually,
customers prefer the detergent from the brand that already has an established soap cake
selling in the market.

Indicate in the table which cause-and-effect sequence would most likely, according to the con-
sultant, result in a decline of market share of Whito detergent.

Make two selections, one in each column.

A B

Cause Effect

A An increase in the market share of EcoWash soap cake

B An increase in the market share of EcoWash detergent

C An increase in the market share of WhiteMagic detergent

D A decrease in the market share of EcoWash soap cake

E A decrease in the market share of EcoWash detergent

SolutionBoth Parts

Understanding the argument

You are asked to provide a cause-and-effect sequence that would, according to the consultant,
result in a decline of market share of the Whito 3-kg detergent pack. The question suggests
that the answers are dependent on each other, so the correct answer of part A will depend on
the correct answer of part B, and vice versa. The passage states that one factor contributing to
decline of market share of the Whito 3-kg detergent pack is the success of the detergent made
by a competing brand named EcoWash.

EcoWash brands detergent eats up the market share of Whito brand, and so a causal sequence
that has as its effectan increase in market share of EcoWash detergent may produce a decline
in the market share of Whito 3-kgs detergent. This suggests that option B may be the correct
response for part B of the question. However, this depends on whether one of the other
statements in the table describes a cause suggested by the consultant.

The last sentence of the argument implies that if a detergent brand does well in the market, its
cake variant also does well. This also applies to the EcoWash brand. The consultant may have

19992016 Manhattan Review 91 www.manhattanreview.com


92 Two-Part Analysis: Concepts & Examples

taken an increase in the market share of EcoWash soap cake as the cause (Part A, option A)
to produce the effect (Part B, option B) of increase in the market share of EcoWash detergent,
thereby causing a decline in Whitos sales (the conclusion).
The correct answers are option A and B.

www.manhattanreview.com 92 19992016 Manhattan Review


Two-Part Analysis: Concepts & Examples 93

Example 7

ABC Farms has two farms in the city: one in Somanaphate, and the other in Valsad. The
situation at both the farms were alike in all aspects, including the amount of exposure to
sunlight, the amount of water, the quality of the seeds, and the degree of manure useexcept
for the degree of fertilizing of soil with biochar. The degree of fertilization was twice as much
for the Valsad farm as for the Somanaphate farm. The final crop yield from both farms was
approximately the same. ABC Farms claims that fertilizing the soil with biochar was ineffective.

In the table, select the changes that the farmer could make in the Somanaphate and Valsad
farms, respectively, that together would be most helpful in testing ABC Farms claim.

Make only two selections, one in each column.

A B

Somanaphate Valsad

A Double the quantity of the manure

B Test the quality of the seeds

C Change the brand of biochar

D Do away with biochar altogether

E Exercise weed control

SolutionBoth Parts

First understand the argument and its conclusion.

Claim: Fertilizing the soil with biochar is ineffective.

This asserts that fertilizing the soil with biochar does not affect crop yield. You have to
evaluate whether this claim made by the farmer is correct.

Answer Choice Explanation

(A) Option A is out of scope. Doubling the quantity of the manure will not help. The effec-
tiveness of the manure is not called in question.

(B) The argument states that quality of seeds was same for both the farms; hence testing the
quality of seeds is of no significance.

(C) Changing the brand of biochar will not help explain why the biochar was not effective, as
the farmers assertion is that biochar itself is ineffective, not just a specific brand.

19992016 Manhattan Review 93 www.manhattanreview.com


94 Two-Part Analysis: Concepts & Examples

(D) Option D is the correct answer for both parts: do away with biochar in both the farms.
By not amending the soils with biochar, the farmer can evaluate his assumption by mea-
suring the yields of both the farms while keeping other parameters the same. If the
difference in yield is insignificant, his claim is right; otherwise it is wrong.

(E) Exercising weed control is out of scope.

The correct answers are option D and D.

www.manhattanreview.com 94 19992016 Manhattan Review


Chapter 6

Multi-Source Reasoning

6.1 Strategies and Concepts

Out of four parts of the IR-GI, TA, MSR, and Two-Part sections, the MSR section may look
more scary, but in reality, it is not so. The MSR format is more along the lines of Reading
Comprehension format. Unlike dataset of TA, Two-Part, and GI sections, an MSR dataset may
have 35 questions. You will find that one MSR dataset in Official Guide online companion has
as many as six questions. However, during an exam, you will usually come across only three
questions.
As stated earlier, that there are 23 tabs of information in an MSR dataset, but you can only see
one tab at a time. This will not be a great challenge if you have sifted through spreadsheets in
Excel, in which you view only one sheet at a time. You can view 23 tabs with a click of mouse.
As with the TA section, the MSR part has interactive prompts.
There are three types of questions asked in the MSR section:

(1) Quants-based

(2) Verbal-based

(3) A mix of Quants and Verbal-based

95
96 Multi-Source Reasoning: Concepts & Examples

6.2 Process of Solving MSR Questions

(1) Understand the data set


Read the information given in each tab. Understand it, but do not read it more than once
at this stage. Then read the question. Remember that only one question will be visible
at a time. Keep in mind that you have on average 7.5 minutes to answer all three MSR
questions.

(2) Understand the question


Read the question and make sure you understand it, so you know what you are looking
for. Read the tabs again to gather information for the answer.
Repeat this for other two questions. By the time you answer the third question, you
should have read the tabs at least three times.

(3) Develop an approach to solve the question


You will have to gather information from more than one tab. Say, for example, a question
asks about a companys profit. Suppose Tab 1 gives details about revenue and sales,
whereas Tab 2 gives details about budget and cost. You must sift through both tabs to
get the answer.

(4) Apply the approach


Apply the previous step to finally solve the problem. Do your scratch work neatly, as you
may need to use some calculations again to answer another question.

www.manhattanreview.com 96 19992016 Manhattan Review


Multi-Source Reasoning: Concepts & Examples 97

6.3 Examples

Examples 13

Tab 1

Sales Volume
distribution of Sales Volume
Price
companies of Apex Corp

In the mosquito-repellent product category, there are five major companies in the retail
business. The pie chart represents company-wide sales volume distribution of mosquito-
repellent products.

Sales volume distribution of Companies

Simpson,
15%
Apex, 22%

Gelton, 21%

Austere, 32%

Pedro,
10%

19992016 Manhattan Review 97 www.manhattanreview.com


98 Multi-Source Reasoning: Concepts & Examples

Tab 2

Sales Volume
distribution of Sales Volume
Price
companies of Apex Corp

The table below presents the region-wise and product-wise sales volume distribution of
the Apex Corporation.

Products North South East West Total

Mosquito Repellent Coil 124 140 106 50 420

Mosquito Repellent Refill 100 110 90 70 370

Mosquito Repellent Cream 46 140 98 6 290

Mosquito Repellent Gel 24 98 32 16 170

Total 294 488 326 142 1250

Unit: 000 cartons

www.manhattanreview.com 98 19992016 Manhattan Review


Multi-Source Reasoning: Concepts & Examples 99

Tab 3

Sales Volume
distribution of Sales Volume
Price
companies of Apex Corp

Pricing of products for Apex


Corporation
250

200
$ per carton

150

100

50

0
Mosquito Mosquito Mosquito Mosquito
Coil Refill Repellent Repellent
Cream Gel
Products

19992016 Manhattan Review 99 www.manhattanreview.com


100 Multi-Source Reasoning: Concepts & Examples

Question 1

For each of the following statements, select Yes if the statement is true based on the infor-
mation provided; otherwise select No.

Yes No

A The difference between the sales volume of Austere


Corporation and Pedro Corporation is more than 1,250,000
cartons.

B For the Apex Corporation, sales revenue from the Eastern


region is more than that from the Northern region.

C For the Apex Corporation, sales revenue from Mosquito Coil


is more than that from Mosquito Refill.

Solution

Part A: From Tab 1, you know that the percent sales volume for the Apex Corporation
is 22% of the total sales volume of all five companies. From Tab 2, you know that the
sales volume of the Apex Corporation is 1,250,000 cartons. So, 22% (total sales) =
1, 250, 000.
Again, from Tab 1, you will find that the percent difference of the sales volume of the
Austere Corporation and the Pedro Corporation is 32 10 = 22%, which is equal to the
percent sales volume of Apex. Hence, the answer is exactly 1,250,000, not more.
The correct answer is No.

Part B: In a nutshell, you are asked to find out whether


Revenue from the Eastern region > Revenue from the Northern region
This question may appear to be time-consuming, but the GMAT never asks a question
that is too time-consuming. So, there must be an alternate approach.
We find that the sales volumes of the Coil and Refill are more for the Northern region
than for the Eastern region, while the opposite is true for the Cream and Gel. You must
analyze which mosquito repellent(s) will contribute more in revenue to a region when
sales volumes are multiplied with their respective prices. Clearly, the impact of the
Cream type is very high, as the Eastern region sells 98, 000 46, 000 = 52, 000 more
mosquito repellent Cream than does the Northern region. Also, its price ($150 per car-
ton) is second highest among all the products. This is going to contribute more in rev-
enue to the Eastern region compared to the Northern region. Though Northern region
sells 100, 000 90, 000 = 10, 000 more units of the highest priced ($195 per carton)
mosquito repellent Refill, its impact is relatively low as 10, 000  52, 000. Similarly,
though Northern region sells 124, 000 106, 000 = 18, 000 more units of the low priced
($95 per carton) mosquito repellent Coil, its impact is relatively low. So collectively, we

www.manhattanreview.com 100 19992016 Manhattan Review


Multi-Source Reasoning: Concepts & Examples 101

find that (10, 000 + 24, 000 = 34, 000)  52, 000. So, the answer would be Yes. In fact,
the Eastern-region revenue is more than Northern-regions by $4,760,000.
The correct answer is Yes.
The table below presents the computation.

Products & Price North East

Mosquito Repellent Coil ($95) 124 95 = 11780 106 95 = 10070

Mosquito Repellent Refill ($195) 100 195 = 19500 90 195 = 17550

Mosquito Repellent Cream ($155) 46 155 = 7130 98 155 = 15190

Mosquito Repellent Gel ($45) 24 45 = 1080 32 45 = 1440

Total ($000) 39490 44250

Part C: Fetching the data from Tabs 2 and 3, you will find that the carton price of the
Refill ($195 per carton) is almost double that of the Coil ($95 per carton) , while sales
volume of the Refill is a mere 10% less than that of the Coil. Collectively, the revenue
from the Refill sales would be more than that from the Coil sales.
The correct answer is No.

19992016 Manhattan Review 101 www.manhattanreview.com


102 Multi-Source Reasoning: Concepts & Examples

Question 2

If the Pedro Corporation sells exactly the same kinds of mosquito repellent products as does
Apex Corporation: Coils, Refills, Cream, and Gel, and it has the same proportion of sales vol-
ume for the four products as does Apex Corporation, then how many Gel cartons did the Pedro
Corporation sell?

(A) 38,000

(B) 77,000

(C) 170,000

(D) 374,000

(E) 1250,000

Solution

The sales volume of Gel for the Apex Corporation is 170,000. The percent share of sales
volume for the Apex Corporation is 22% of all five companies, while for Pedro, it is 10%.
Clearly, the sales volume of Gel for Pedro was

10%
170, 000 = 77, 000 cartons
22%

The correct answer is option B.

Question 3

Keeping the total sales volume the same for the Apex Corporation, which of the following
rearrangement of sales volume of products will generate more sales revenue than currently?

(A) Reduce the Mosquito Coil sales volume by half and increase the sales volume of Mosquito
Repellent Gel by the same amount.

(B) Reduce the Mosquito Refill sales volume by one-third and increase the sales volume of
Mosquito Repellent Cream by the same amount.

(C) Reduce the Mosquito Cream sales volume by 20% and increase the sales volume of
Mosquito Repellent Gel by the same amount.

(D) Reduce the Mosquito Coil sales volume by 33.33% and increase the sales volume of
Mosquito Repellent Cream by the same amount.

(E) Reduce the Mosquito Repellent Cream sales volume by 66.66% and increase the sales
volume of Mosquito Coil by the same amount.

www.manhattanreview.com 102 19992016 Manhattan Review


Multi-Source Reasoning: Concepts & Examples 103

Solution

In all the options, the sales volume of one kind of mosquito repellent is reduced and that of
another kind is increased by the same amount. So, the revenue can be increased if a product
with a lower price is replaced by a product with a higher price. In one instance, reduced sales
volume of the Coil ($95 per carton) is offset by the relatively high-priced Mosquito Cream ($155
per carton). This move will certainly increase the revenue. For the other options, the revenue
will decrease.
The correct answer is option D.

19992016 Manhattan Review 103 www.manhattanreview.com


104 Multi-Source Reasoning: Concepts & Examples

Examples 46

Tab 1

Mike, Chief economist, on BRICS countries Professor Walter on BRICS countries

The notion that the era of emerging BRICS countries is overand that among them only
China would make it to the group of high-income countriesis outlandish. No doubt the
growth rates in the BRICS group of Brazil, Russia, India, China, and South Africa have been
affected by the global slowdown, and countries such as India have been further singed by
capital-flow reversals. However, this is a temporary phenomenon that will peter out sooner
rather than later. The BRICS countries economies are bound to reinvent the global eco-
nomic orderand even fashion it in their own imageonce the macroeconomic balances
are restored and foreign investment flows rebound, boosted by reforms.
Similarly, the argument that the rest of the BRICS countries will fall by the wayside while
China continues to march ahead is flawed. Chinas growth rate is almost half that of peak
levels. Its also facing a double whammy: 1) its export-led economy has been badly hit by
the slowdown in advanced country markets, and 2) rising wages and a shortage of skills
erode its competitive base, as it struggles to shift over to a domestic consumption-based
growth model. This will probably help other BRICS countries, such as India, make new
inroads into the global markets for manufactured goods, and thus close in fast on China.
In fact, the most recent trends on the global Greenfield investments, which exclude merg-
ers and acquisitions, validate this argument. The numbers show that while the new FDI
(Foreign Direct Investment) projects in China have almost halved after the global slow-
down, the other BRICS countries havent been so badly bruised. On the contrary, the gap
between China and other BRICS countries has in fact shrunk, with India accounting for
30% of the Greenfield FDI projects, as compared to Chinas 40% share.

www.manhattanreview.com 104 19992016 Manhattan Review


Multi-Source Reasoning: Concepts & Examples 105

Tab 2

Mike, Chief economist, on BRICS countries Professor Walter on BRICS countries

The economists assessment that the BRICS era is at an end is right on the money. De-
spite witnessing robust economic growth in the last decade, each of the BRICS countries
faces a unique set of problems today. The recent global economic downturn has exposed
structural infirmities that will prevent these economies from returning to a high-growth
trajectory anytime soon. Besides, it is not realistic to expect these emerging markets to
grow faster from a higher GDP base rather than from their previous low threshold.
In India, the economy is wracked by a Rupee in free-fall, high inflation, and a burgeoning
current account deficit. Recent months have seen significant capital outflows, with foreign
investors opting to park their funds in a recovering U.S. economy. The petering-out of
growth sentiments is directly related to the political leaderships failure to affect a much-
needed second wave of economic reforms. Furthermore, with policy paralysis expected to
continue, the India-growth story remains in limbo. In both Brazil and Russia, the weaken-
ing of commodity prices has hit the economies hard, exposing their over-reliance on nat-
ural resources as cash cows. Meanwhile, South Africas economy has been hurting since a
recession that affected several crucial industries. In China, the economy is transitioning
from resource-intensive, investment-led growth to a consumption-oriented pattern. Add
to this the massive global pressure to appreciate the Yuan, and it is clear that China would
need to affect a not-so-easy overhaul of its economic model to maintain high growth. How-
ever, as the economist points out, given Chinas planned economic model and ability to
move resources without political missteps, it is best placed from among the BRICS nations
to pull out of the middle-income trap. Taken together, the global heft that the BRICS bloc
wielded is over. While these emerging markets will continue to grow, they will need to get
used to moderate rates of growth.

19992016 Manhattan Review 105 www.manhattanreview.com


106 Multi-Source Reasoning: Concepts & Examples

Question 4

For each of the following issues, select Agree if, based on the information provided, it can
be inferred that both the commentators would hold similar positions on the issue. Otherwise,
select Disagree.

Agree Disagree

A Acknowledging that the BRICS countries had experienced


good economic growth in the past

B The timeframe in which the BRICS countries will bounce


back to good economic growth

C Chinas ability to transform into a consumption-oriented


economy soon

Solution

Part A:
In Tab 1, the second sentence (paragraph 1), (Mike) says, No doubt the growth rates in
the BRICS group of Brazil, Russia, India, China, and South Africa have been affected by
the global slowdown. . . The fourth sentence (paragraph 1), says, The BRICS countries
economies are bound to reinvent the global economic orderand even fashion it in their
own image. . .
Look at Tab 2, the second sentence (paragraph 1), Professor Walter says Despite witness-
ing robust economic growth in the last decade, each of the BRICS countries faces a unique
set of problems today.
Both would agree on experiencing good economic growth in the past.

Part B:
In Tab 1, the third sentence in the first paragraph says, However, this is a temporary
phenomenon that will peter out sooner rather than later. So, Mike opines that the BRICS
economies will bounce back soon.
In Tab 2, the first paragraph, third sentence, says, The recent global economic downturn
has exposed structural infirmities that will prevent these economies from returning to
a high-growth trajectory anytime soon. So, Professor Walter opines that the BRICS
economies will not be able to bounce back soon.
Both would disagree on the recovery timeframe.

Part C:
In Tab 1, the second sentence in the second paragraph says, its [Chinas] competitive
base, as it struggles to shift over to a domestic consumption-based growth model. So,
Mike opines that it will be difficult for China to shift over to a consumption-based econ-
omy.

www.manhattanreview.com 106 19992016 Manhattan Review


Multi-Source Reasoning: Concepts & Examples 107

In Tab 2, the sixth sentence in the second paragraph says, In China, the economy is
transitioning from resource-intensive, investment-led growth to a consumption-oriented
pattern. So, Professor Walter opines that Chinese economy on the course transforming
to consumption-oriented economy.
Both would disagree on this issue.

Question 5

For each of the following issues, select India if, based on the information provided, it can be
inferred that the issue pertains to India; otherwise, select China.

India China

A The degree of political impediment in the country to


economic growth

B The severity of impact on the country due to withdrawal of


new FDI projects

C The global slowdown, along with capital-flow reversal

Solution

Part A:
Refer to Tab 2, the fifth sentence in the second paragraph, which says, Furthermore, with
policy paralysis expected to continue, the India growth story remains in limbo. Profes-
sor Walter clearly expresses concern over the economic recovery in India due to policy
paralysis, which implies political impediments.
To confirm that China doesnt have political impediments, again refer to Tab 2, the eighth
sentence in the second paragraph, which says, However, as the economist points out,
given Chinas planned economic model and ability to move resources without political
missteps, . . .
India is the correct answer for option A.

Part B:
Refer to Tab 1, the second sentence of the third paragraph, which says, The numbers
show that while the new FDI (Foreign Direct Investment) projects in China have almost
halved after the global slowdown, the other BRICS countries havent been so badly
bruised. Mike clearly expresses that, due to withdrawal of new FDI projects, China is
bruised badlymeaning impacted severely.
China is the correct answer for option B.

19992016 Manhattan Review 107 www.manhattanreview.com


108 Multi-Source Reasoning: Concepts & Examples

Part C:
Refer to Tab 1, the second sentence in the first paragraph, which says, No doubt the
growth rates in the BRICS group of Brazil, Russia, India, China, and South Africa have been
affected by the global slowdown, and countries such as India have been further singed by
capital-flow reversals.
Refer to Tab 2, the second sentence of the second paragraph, which says, Recent months
have seen significant capital outflows, with foreign investors opting to park their funds in
a recovering U.S. economy.
It can be inferred from both that India has been impacted by global slowdown, along with
capital flow reversal.
India is the correct answer for option C.

Question 6

Which of the following cannot be inferred based on the information provided?

(A) Brazils and Russias economies are dependent on natural resources.

(B) The U.S. economy is benefitting in some way from current economic problems in one of
the BRICS countries.

(C) Weakening of commodity prices in Brazil and Russia benefitted China.

(D) Export-led economy is affected by slowdown in advance country markets.

(E) Russia, Brazil, and South Africa each account for much less than 40% of Greenfield FDI
projects in BRICS.

Solution

Refer to Tab 2, the fifth sentence in the second paragraph, which says, In both Brazil and
Russia, the weakening of commodity prices has hit the economies hard, exposing their over-
reliance on natural resources as cash cows. From this, you can infer that Brazils and Russias
economies are dependent on natural resources (option A). However, regarding the weakening
of commodity prices, you can only infer that it has impacted Brazil and Russia hard, not that
it has benefitted China.
Option B can also be inferred. Refer to Tab 2, the first sentence of the second paragraph,
which says, Recent months have seen significant capital outflows, with foreign investors opting
to park their funds in a recovering U.S. economy.
Option D can also be inferred. Refer to Tab 1, the second and third sentences of the second
paragraph, which say, Chinas growth rate is almost half that of peak levels. Its also facing a
double whammy: 1) its export-led economy has been badly hit by the slowdown in advanced
country markets, . . .
Option E can also be inferred. Refer to Tab 1, the last sentence of the third paragraph, which
says, On the contrary, the gap between China and other BRICS countries has in fact shrunk,

www.manhattanreview.com 108 19992016 Manhattan Review


Multi-Source Reasoning: Concepts & Examples 109

with India accounting for 30% of the Greenfield FDI projects, as compared to Chinas 40% share.
Among the BRICS countries, only India is closest to China, with 30% of the Greenfield FDI
projects in BRICS. That means the other BRICS countries must account for much less than 40%
of the Greenfield FDI projects in BRICS.
All options can be inferred except option C.
The correct answer is option C.

19992016 Manhattan Review 109 www.manhattanreview.com


110 Multi-Source Reasoning: Concepts & Examples

www.manhattanreview.com 110 19992016 Manhattan Review


Chapter 7

Practice Questions

111
112 Table Analysis Practice Questions

7.1 Table Analysis


1. The table presents the average maximum score, average minimum score, and the Intel-
ligence Quotient (IQ) for Grade X and XII students of City International School for six
months.

Average Average
Intelligence
Months Grade maximum minimum
Quotient
Score Score
August X 66 58 116
August XII 68 55 120
December XII 70 49 114
December X 71 52 117
July X 71 55 115
July XII 72 45 116
November XII 69 51 118
November X 81 62 119
October XII 69 45 120
October X 68 54 121
September XII 75 56 114
September X 72 52 121

For each of the following statements, select Yes if the statement is true based on the
information provided; otherwise, select No.

Yes No

A Compared to grade XII students, grade X students are more con-


sistent with regards to the Intelligence Quotients attribute, taking
range as a parameter to measure the consistency.

B For any of the given months, the greatest deviation between the
average maximum score and average minimum score for grade XII
students is more than that for grade X students.

C The mean average minimum score for August is more than that
for July and for December.

www.manhattanreview.com 112 19992016 Manhattan Review


Table Analysis Practice Questions 113

The table sorted by column 2 and by column 5 are given below:

Sorted by column 2
Col 1 Col 2 Col 3 Col 4 Col 5
Average Average
Intelligence
Months Grade maximum minimum
Quotient
Score Score
July X 71 55 115
August X 66 58 116
September X 72 52 121
October X 68 54 121
November X 81 62 119
December X 71 52 117
July XII 72 45 116
August XII 68 55 120
September XII 75 56 114
October XII 69 45 120
November XII 69 51 118
December XII 70 49 114

Sorted by column 5
Col 1 Col 2 Col 3 Col 4 Col 5
Average Average
Intelligence
Months Grade maximum minimum
Quotient
Score Score
September XII 75 56 114
December XII 70 49 114
July X 71 55 115
August X 66 58 116
July XII 72 45 116
December X 71 52 117
November XII 69 51 118
November X 81 62 119
August XII 68 55 120
October XII 69 45 120
September X 72 52 121
October X 68 54 121

Solve yourself:

19992016 Manhattan Review 113 www.manhattanreview.com


114 Table Analysis Practice Questions

2. Twelve students from a school secured marks in four subjects. They were scored on
Math and Science (out of a possible 100 points), Crafts (out of a possible 25 points), and
English (out of a possible 50 points). The table below shows the data.

Math Craft English Science


Students
(100) (25) (50) (100)
Angelo 84 9 35 35
Elad 74 5 49 84
Garrison 59 15 37 33
Jack 69 7 41 27
Joe 64 16 31 31
Justin 12 18 5 35
Kevin 89 20 39 37
Lucy 79 7 33 29
Marie 10 20 6 45
Matt 54 22 45 39
Mehul 94 13 43 41
Sean 8 22 50 47

For each of the following statements, select Yes if the statement is true based on the
information provided; otherwise select No.

Yes No

A The mean score for Math is greater than its median score.

B The Math and Science scores for Justine were swapped by


mistake. Correcting the mistake will impact neither the
median score for Math nor that for Science.

C If two new students Suzi and McDonald are included, whose


Science scores are 10 and 45, respectively, the median score
for Science will increase by 2 points over its current value.

www.manhattanreview.com 114 19992016 Manhattan Review


Table Analysis Practice Questions 115

The tables sorted by column 2 and by column 5 are given below:

Sorted by column 2
Col 1 Col 2 Col 3 Col 4 Col 5
Math Craft English Science
Students
(100) (25) (50) (100)
Sean 8 22 50 47
Marrie 10 20 6 45
Justin 12 18 5 35
Matt 54 22 45 39
Garysson 59 15 37 33
Joe 64 16 31 31
Jack 69 7 41 27
Elad 74 5 49 84
Lucy 79 7 33 29
Angelo 84 9 35 35
Kevin 89 20 39 37
Mehul 94 13 43 41

Sorted by column 5
Col 1 Col 2 Col 3 Col 4 Col 5

Math Craft English Science


Students
(100) (25) (50) (100)
Jack 69 7 41 27
Lucy 79 7 33 29
Joe 64 16 31 31
Garysson 59 15 37 33
Justin 12 18 5 35
Angelo 84 9 35 35
Kevin 89 20 39 37
Matt 54 22 45 39
Mehul 94 13 43 41
Marrie 10 20 6 45
Sean 8 22 50 47
Elad 74 5 49 84

Solve yourself:

19992016 Manhattan Review 115 www.manhattanreview.com


116 Table Analysis Practice Questions

3. The table below shows the sales performance of few cars sold by a few car dealerships. It
provides information on maximum car sales per month, minimum car sales per month,
and the car showroom managers comments on the months car sales.

Maximum sales Minimum sales Manager's comments


Cars Company
per month per month on sales

3 Series BMW 10 1 Average


370Z Nissan 19 11 High
5 Series BMW 11 2 Average
7 Series BMW 9 3 Average
Cruze General Motors 21 12 High
Cube Nissan 26 20 High
Dayz Nissan 23 16 High
Expedition Ford Motors 10 5 Average
Fiesta Ford Motors 7 2 Low
Grandeur Hyundai 21 10 High
JM Hyundai 21 15 High
Juke Nissan 25 18 High
M Model BMW 6 0 Low
Maliba General Motors 10 6 Average
Micra Nissan 16 8 High
Mini Seven BMW 8 4 Below average
Moco Nissan 20 11 High
Mondeo Ford Motors 21 10 High
Serena Nissan 21 11 Average
Sonata Hyundai 20 13 High
Sonic General Motors 22 16 High
Sunny Nissan 11 6 Average
Trajet Hyundai 27 19 High
X5 BMW 22 12 High

For each of the following statements, select Yes if the statement is true based on the
information provided; otherwise, select No.

Yes No

A At least two cars reporting High sales had their minimum


sales per month less than the highest minimum sales per
month for a car among those cars that did not report High
sales.

B The maximum sales per month and minimum sales per


month are negatively correlated.

C The mean (average) maximum sale per month for Hyundai


cars is more than that for BMW cars.

www.manhattanreview.com 116 19992016 Manhattan Review


Table Analysis Practice Questions 117

The tables sorted by column 2, column 3, and column 5 are given below:

Sorted by column 2 Sorted by column 3 Sorted by column 5


Col 2 Col 3 Col 3 Col 4 Col 4 Col 5
Maximum Minimum
Company sales per Maximum sales Minimum sales Manager's comments
sales per
month per month per month on sales
month
BMW 6 6 0 3 Average
BMW 8 7 2 1 Average
BMW 9 8 4 5 Average
BMW 10 9 3 6 Average
BMW 11 10 1 2 Average
BMW 22 10 5 6 Average
Ford Motors 7 10 6 11 Average
Ford Motors 10 11 2 4 Below average
Ford Motors 21 11 6 8 High
General Motors 10 16 8 11 High
General Motors 21 19 11 11 High
General Motors 22 20 11 13 High
Hyundai 20 20 13 12 High
Hyundai 21 21 11 10 High
Hyundai 21 21 12 15 High
Hyundai 27 21 10 10 High
Nissan 11 21 15 16 High
Nissan 16 21 10 12 High
Nissan 19 22 16 16 High
Nissan 20 22 12 18 High
Nissan 21 23 16 20 High
Nissan 23 25 18 19 High
Nissan 25 26 20 0 Low
Nissan 26 27 19 2 Low

Solve yourself:

19992016 Manhattan Review 117 www.manhattanreview.com


118 Table Analysis Practice Questions

4. The table shows statistics for a few T20 Cricket teams that played matches throughout
the year. A player can play for more than one teams. The result of any match is either a
win or a loss.

Number of Average % of
Most Valuable
Teams Home Ground Country matches score per matches
Player
played match won
Chennai Super Kings Chennai India Dhoni, Nannes 85 144 46
Deccan Chargers Hyderabad India Kumara 77 128 46
Delhi Daredevils Delhi India Mahela, Sehwag 81 145 54
Kolkata Knight Riders Kolkata India Gambhir 80 154 81
Mumbai Indians Mumbai India Sachin, Malinga 83 165 83
Nagenahira Nagas Eastern Province Sri Lanka Clarke, Kumara 14 144 43
New South Wales Blues Sydney Australia Kumara 24 124 83
Punjab Kings XI Mohali India Clarke 74 125 42
Royal Challengers Bangalore India Gayle 74 121 45
Ruhuna Royals Southern Province Sri Lanka Malinga, Gayle 15 153 47
Southern Redbacks Adelaide Australia Mahela 22 121 87
Tasmanian Tigers Hobart Australia Clarke, Gayle 21 154 43
Uthura Rudra Dambula Sri Lanka Gambhir, Nannes 11 152 46
Uva Next Badulla Sri Lanka Sachin, Mahela 12 154 91
Victorian Bushrangers Melbourne Australia Sachin 26 155 46

For each of the following statements, select Yes if the statement is true based on the
information provided; otherwise select No.

Yes No

A The teams that included Sachin have the higher average


scores per match than the teams that do not have him.

B It is likely that the teams that included Clarke did not win a
tournament.

C The mean average score for Australian teams is more than


that for Sri Lankan teams.

www.manhattanreview.com 118 19992016 Manhattan Review


Table Analysis Practice Questions 119

The tables sorted by column 3 and by column 6 are given below:

Sorted by column 6 Sorted by column 3


Col 4 Col 5 Col 6 Col 3 Col 4 Col 5 Col 6 Col 7
Average Average
Number of Number of
score score % of
Most Valuable Player matches Country Most Valuable Player matches
per per matches
played played
match match won
Mahela 22 121 Australia Mahela 22 121 87
Gayle 74 121 Australia Kumara 24 124 83
Kumara 24 124 Australia Clarke, Gayle 21 154 43
Clarke 74 125 Australia Sachin 26 155 46
Kumara 77 128 India Gayle 74 121 45
Dhoni, Nannes 85 144 India Clarke 74 125 42
Clarke, Kumara 14 144 India Kumara 77 128 46
Mahela, Sehwag 81 145 India Dhoni, Nannes 85 144 46
Gambhir, Nannes 11 152 India Mahela, Sehwag 81 145 54
Malinga, Gayle 15 153 India Gambhir 80 154 81
Clarke, Gayle 21 154 India Sachin, Malinga 83 165 83
Gambhir 80 154 Sri Lanka Clarke, Kumara 14 144 43
Sachin, Mahela 12 154 Sri Lanka Gambhir, Nannes 11 152 43
Sachin 26 155 Sri Lanka Malinga, Gayle 15 153 47
Sachin, Malinga 83 165 Sri Lanka Sachin, Mahela 12 154 91

Solve yourself:

19992016 Manhattan Review 119 www.manhattanreview.com


120 Graphics Interpretation Practice Questions

7.2 Graphics Interpretation

1. The clustered column chart represents commuters preferences for buses, trains, and
taxis, during a five-year period.

Commuting
450

400
Commuters (Million)

350

300

250

200

150

100

50

0
2005 2006 2007 2008 2009

Intra-City Bus Local Train Private Taxi

Based on the given information, use the drop-down menus to most accurately complete
the following statements.

Part A:
In the year 2006, the average bus fare per commuter was $0.75 and that for taxi
was $30. The ratio of total revenue from taxi commuters to total revenue from bus
commuters in the year 2006 was .
(A) 10 : 1
(B) 20 : 1
(C) 30 : 1
(D) 80 : 1
Solve yourself:

www.manhattanreview.com 120 19992016 Manhattan Review


Graphics Interpretation Practice Questions 121

Part B:
The number of taxi commuters in the year 2010 would be approximately
million, if the number of taxi commuters from the year 2009 to the year 2010 grows
at the same rate as from the year 2008 to the year 2009.
(A) 174
(B) 224
(C) 264
(D) 289
Solve yourself:

19992016 Manhattan Review 121 www.manhattanreview.com


122 Graphics Interpretation Practice Questions

2. The Pie diagram represents the percentage distribution of undergraduate degrees earned
by current Post-Graduate B-School students in a select few faculties.

Undergraduate degree distribution

Art
Others
6%
12%
Science
12%

Humanities
13%

Engineering
Management 18%
12%

Commerce
27%

Based on the given information, use the drop-down menus to most accurately complete
the following statements.

Part A:
If 126 students earned their undergraduate degrees from the Art stream, then the
number of students who earned their undergraduate degrees from the Engineering
stream exceeds that from the Management stream by .
(A) 126
(B) 252
(C) 378
(D) 504
(E) 2100
Solve yourself:

www.manhattanreview.com 122 19992016 Manhattan Review


Graphics Interpretation Practice Questions 123

Part B:
The number of undergraduate students from the Engineering stream is more than
that from the Science stream by .
(A) 6%
(B) 33.33%
(C) 50%
Solve yourself:

19992016 Manhattan Review 123 www.manhattanreview.com


124 Graphics Interpretation Practice Questions

3. The cluster-column chart shows percentage distribution of household owners for seven
electronic goods, for the year 2009 and 2012.

% distribution of owners
100
90
80
70
% owners

60
50
40
2009
30
2012
20
10
0

Based on the given information, use the drop-down menus to most accurately complete
the following statements.
Part A:
During the three-year period, the maximum percentage increase in popularity regis-
tered by an electronic goods equals %. (Assume that the population of house-
hold owners was the same for both the years).
(A) 3
(B) 22
(C) 31
(D) 55
Solve yourself:

Part B:
If the number of owners increased from 606 million in the year 2009 to 1210 million
in the year 2012, then the number of new fridges purchased between 2009 and 2012
is million. (Assume that no household owner has sold/discarded a fridge
during the period.)
(A) 22
(B) 160.7
(C) 266

www.manhattanreview.com 124 19992016 Manhattan Review


Graphics Interpretation Practice Questions 125

(D) 713
Solve yourself:

19992016 Manhattan Review 125 www.manhattanreview.com


126 Graphics Interpretation Practice Questions

4. The graph represents the ratios of currencies: Rupees-to-Dollar, Brazilian-Real-to-Dollar,


and Dollar-to-Kuwaiti-Dinar, for the period of five months. Rupees-to-Dollar is repre-
sented by the primary vertical axis, while Real-to-Dollar and Dollar-to-Dinar are repre-
sented by the secondary vertical axis.

Currency Rates
Rs to $ Real to $ $ to Dinar
70 3.7

Real to $ $ to Dinar
3.5
65
3.3
Rs. To $

60 3.1

55 2.9

2.7
50
2.5

45 2.3

Months

Based on the given information, use the drop-down menus to most accurately complete
the following statements.

Part A:
On July 1, Rupee 5500 amounts to Dinar .
(A) 27.81
(B) 35
(C) 337.7
(D) 350
(E) 1571
Solve yourself:

www.manhattanreview.com 126 19992016 Manhattan Review


Graphics Interpretation Practice Questions 127

Part B:
A Mac computer costs Real 2400 on September 1. A man has $1100, Dinar 250, and
Rupees 50,000. He would be able to buy a Mac using currency(ies). (Assume
that he can use only one type of currency for the whole transaction.)
(A) Dollar
(B) Dollar and Rupee
(C) Rupee and Dinar
(D) Dinar
(E) Dinar and Dollar
Solve yourself:

19992016 Manhattan Review 127 www.manhattanreview.com


128 Graphics Interpretation Practice Questions

5. In a school, students major only in Management, Engineering, and Humanities. The


Humanities students play only one of the four sports: Soccer, Basketball, Hockey, and
Tennis. The Soccer-playing students are Brazilian, Mexican, and Argentinian nationals.
The graph shows the percentage distribution of students for majors, the Humanities
students sport preferences, and the nationalities of Soccer playing Humanities students.

100%
Argentinian,
Tennis,
20%
25%
80%
Humanities,
Hockey,
60%
15%
60% Mexican,
Basketball, 50%
20%
40%

Engineering,
25%
20%
Soccer,
40% Brazilian,
Management, 30%
15%
0%
Major Humanities Soccer

Based on the given information, use the drop-down menus to most accurately complete
the following statements.

Part A:
The humanities students who play tennis are of the total students in the
school.
(A) 7.50%
(B) 15%
(C) 25%
(D) 60%
Solve yourself:

www.manhattanreview.com 128 19992016 Manhattan Review


Graphics Interpretation Practice Questions 129

Part B:
If there are 80 Mexican Humanities students in the school who play soccer, then the
number of students studying Management is .
(A) 50
(B) 100
(C) 160
(D) 400
Solve yourself:

19992016 Manhattan Review 129 www.manhattanreview.com


130 2-Part Analysis Practice Questions

7.3 2-Part Analysis

Practice Questions (Quants)

1. The average speed of a car was 72 miles/hr for a long journey of d miles. The cost of the
fuel was $3.60/gallon. The average consumption of the fuel was c gallons/hour.
In terms of the variables d and c, select an expression that represents the total fuel
consumption (in gallons) and the total cost of fuel (in dollars) for the entire journey.
Make only two selections, one in each column.

A B

Total Fuel Total Cost


Consumption of Fuel
(gallons) (dollars)

A cd/72

B cd/20

C c/72d

D c/20d

E 20cd

Solve yourself:

www.manhattanreview.com 130 19992016 Manhattan Review


2-Part Analysis Practice Questions 131

2. Given that n people can sit in r chairs arranged in a row in 56 ways; find the possible
values of n and r that are consistent with the information provided.
Make one selection in each column.

A B

n r

A 2

B 3

C 4

D 5

E 6

F 8

Solve yourself:

19992016 Manhattan Review 131 www.manhattanreview.com


132 2-Part Analysis Practice Questions

3. Alex can beat Betty in an 800-yard race by 120 yards, Betty can beat Cherry in a 1700-yard
race by 200 yards, and Cherry can beat Dirk in a 1200-yard race by 80 yards.
Select the values in yards for Alex beats Cherry and Betty beats Dirk in a 400-yard
race that are consistent with the information provided.
Make only two selections, one in each column.

A B

Alex Betty
beats beats
Cherry Dirk

A 30

B 50

C 70

D 90

E 100

Solve yourself:

www.manhattanreview.com 132 19992016 Manhattan Review


2-Part Analysis Practice Questions 133

4. Mike, Garry, and Jack are given an assignment to collectively solve a tricky questios. If
the probabilities of Mike, Garry, and Jack solving similar questions individually are 1/3,
1/4, and 1/5, respectively, then select the probabilities that the question will be solved
and that it will not be solved.
Make only two selections, one in each column.

A B

Probability Probability
that the that the
question will question will
be solved not be solved

A 1/60

B 1/12

C 11/12

D 2/5

E 3/5

F 59/60

Solve yourself:

19992016 Manhattan Review 133 www.manhattanreview.com


134 2-Part Analysis Practice Questions

5. The Midtown City Centre wishes to organize two plays: a Broadway and a Musical. It
wants to invite five foreign artists for each play. Four foreign artists for each play have
already been finalized.
The center has laid down few conditions for selecting the fifth foreign artist for each
play. The conditions are:

(A) No more than one musician should be in any play


(B) A minimum of two theatre artists must be in any play
(C) A maximum of two artists of the same country can be in any play

Broadway Play Musical

Alicia Diego
(Cinema artist, Kid, U.S.) (Theatre artist, Singer, Argentina)

Joe Pesky
(Cinema artist, Adolescent, UK) (Cinema artist, Senior, Argentina)

Xiang Lue
(Cinema artist, Teenager, China) (Theatre artist, Teenager, China)

Nobita Leon
(Theatre artist, Musician, Japan) (Cinema artist, Kid, U.S.)

Select which foreign artist given in the table is not eligible to participate in any play, and
is eligible to participate in both the plays per the information provided.

www.manhattanreview.com 134 19992016 Manhattan Review


2-Part Analysis Practice Questions 135

Make only two selections, one in each column.

A B

No Play Both Plays

A Podolsky (Theatre artist, Musician, U.S.)

B Parker (Cinema artist, Adolescent, U.K.)

C Li Mao (Cinema artist, Teenager, China)

D Balki (Theatre artist, Singer, Argentina)

E Willis (Theatre artist, Kid, New Zealand)

Solve yourself:

19992016 Manhattan Review 135 www.manhattanreview.com


136 2-Part Analysis Practice Questions

Practice Questions (Verbal)

6. Scientists at Indiana University in the U.S. claim to have pinpointed a molecule in the
bloodstream that identifies peoples intent on taking their own lives. These scientists
claim that raised levels of this biomarker can predict and thereby prevent suicide at-
tempts. Which of the following information, if true, would best support the argument,
and which would best not support the argument?
Make only two selections, one in each column.

A B

Support Not Support

A Although medical science has made great


strides in identifying biological predictors,
claims of predicting exclusively human
attributes, such as ethics, love, and free will,
and like are overstated.

B There are cultural factors. In Japan, for


example, suicide can be recognized as noble.
In China, unlike in most other countries, more
women kill themselves than do men.

C The discovery of a blood test that can


determine the propensity of a person to
commit suicide needs to be hailed as a
significant medical breakthrough.

D Unlike with the diagnostic tests, the biggest


challenge with psychiatry is that those
experiencing depression often suppress their
suicidal thoughts, making it extremely difficult
to gauge the mental health of a patient
through traditional psychiatric practices.

E In a country where psychiatric problems are


perceived as taboo, such diagnostic tests will
help people see emotional disorders as
treatable medical conditions.

www.manhattanreview.com 136 19992016 Manhattan Review


2-Part Analysis Practice Questions 137

Solve yourself:

19992016 Manhattan Review 137 www.manhattanreview.com


138 2-Part Analysis Practice Questions

7. Two similar farms were planted with the same number of groundnut plants. Organic
fertilizer was added only to the first farms fields. While the first farm harvested 150 tons
of groundnuts, the second farm harvested 100 tons. Since nothing else except water was
added to both the farms fields, the first farms greater yield must have been due to the
organic fertilizer.
Select Strengthen for the statement that would, if true, most strengthen the argument,
and select Weaken for the statement that would, if true, most weaken the argument.

A B

Strengthen Weaken

A Another similar farm ad


harvested 220 tons of g

B The size of the nuts har


farm is no smaller than
first farm.

C The two similar farms d


soil composition and ba
farm.

D Many weeds that compe


could not subsist due to
on the first farm.

E Some useful weeds (e.g.


second farmhouse drew
Make only two selections, one in each column.

Solve yourself:

www.manhattanreview.com 138 19992016 Manhattan Review


Multi-Source Reasoning Practice Questions 139

7.4 Multi-Source Reasoning

Information for Questions 13

Tab 1

E-mail # 1 E-mail # 2 E-mail # 3

Indian Consultant to Director:


Our team has concluded the analysis of market conditions in India for TopGMAT Inc. The
companys 10 existing franchises continue to perform very well. Given recent performance
and market trends, we project that these franchises will continue to perform well in the
conceivable future. Furthermore, our analysis recommends that scheduled openings of
five additional Indian franchises should be fruitful. However, our research further sug-
gests that those five additional franchises will fully saturate the Indian market with your
unique brand of the GMAT test preparation course.

Tab 2

E-mail # 1 E-mail # 2 E-mail # 3

Director to Employees:
The company is exceedingly proud of our state-of-the-art GMAT test preparation course.
In less than five years, we have gone from one small franchise in Mumbai to over 10
franchises across India. We must hate the word stagnancy. Over the next few months,
the president and I will be aggressively planning the next phase of our growth. We are
considering all options and, as always, welcome any input you may have.

19992016 Manhattan Review 139 www.manhattanreview.com


140 Multi-Source Reasoning Practice Questions

Tab 3

E-mail # 1 E-mail # 2 E-mail # 3

Franchising Advisor to President:


I thank you and TopGMAT Inc. for allowing us to serve you. Over the past five years,
our company has provided sound franchising guidance to over 50 companies in the US.
We look forward to working with you in the years ahead. You have done tremendously
well so far with your 10 Indian franchises. However, we understand your need to expand
beyond India. Franchising is an excellent method of expanding rapidly. By franchising,
you can grow on the investment of entrepreneurs across the country. These entrepreneurs
provide vital knowledge of their home market, while you provide in-depth knowledge of
TopGMATs innovative curriculum and matchless branding.

www.manhattanreview.com 140 19992016 Manhattan Review


Multi-Source Reasoning Practice Questions 141

Questions 13

1. What does the Director mean when he says, We must hate the word stagnancy?

(A) An aggressive plan to open more TopGMAT test-prep franchises in India.


(B) An aggressive plan to open more TopGMAT test-prep franchises in other countries,
including India.
(C) An aggressive plan to offer other courses apart from GMAT courses in other coun-
tries, including India.
(D) An aggressive plan to expand business in generalnothing specific.
(E) An aggressive plan to venture into school and college business.

Solve yourself:

2. Consider each of the following statements. Does the information in the three e-mails
support the following inferences?

Yes No

A The Indian consultant believes that the franchise option will work
better if TopGMAT Inc. leverages on the crucial knowledge that
entrepreneurs have about their home market.

B The Indian consultant is less optimistic about opening more than


15 franchises in India.

C If TopGMAT Inc. expands by franchising in other countries, it is


certain to succeed.

Solve yourself:

19992016 Manhattan Review 141 www.manhattanreview.com


142 Multi-Source Reasoning Practice Questions

3. Consider each of the following statements. Does the information in the three e-mails
support the following inferences?

Yes No

A The Indian market can accommodate only 10


TopGMAT franchise centers.

B The Director has decided that franchising is the


next phase of expansion.

C Expansion beyond India is important to the future


of TopGMAT Inc.

Solve yourself:

www.manhattanreview.com 142 19992016 Manhattan Review


Multi-Source Reasoning Practice Questions 143

Information for Questions 46

Tab 1

Incentive Sales
A super-mall, S-Mart, wants to promote the availability of value-for-money food. To pro-
mote caterers, S-Mart does not charge commission to the vendors; instead it offers incen-
tives. The S-Mart Incentive Program

Regular pizzas Incentive


sold per week (dollars)

100 or more 10

200 or more 20

300 or more 50

500 or more 100

Tab 2

Incentive Sales
Ben is a small time caterer who sells fresh pizzas at S-Mart. Ben spent $1750 to set up the
kitchen and spends $20 per week on operating expenses.

Pizza Prices and Units Sold by Ben

Price per piece Average number of


Pizza
(dollars) units sold per week

Regular Pizza 3 40

Premium Topping 0.75 10

Popular Topping 0.50 15

19992016 Manhattan Review 143 www.manhattanreview.com


144 Multi-Source Reasoning Practice Questions

Questions 46

4. What would be the difference between the maximum and the minimum sales for Ben if
he sold 700 regular pizzas in four weeks?

(A) $40
(B) $50
(C) $60
(D) $70
(E) $80

Solve yourself:

5. If Ben sells 150 regular pizzas, 10 premium toppings, and 5 popular toppings per week,
from which week onwards, he will start making a profit?

(A) Third
(B) Fourth
(C) Fifth
(D) Sixth
(E) Seventh

Solve yourself:

www.manhattanreview.com 144 19992016 Manhattan Review


Multi-Source Reasoning Practice Questions 145

6. For each of the following statements, select Earn more, if the following scenario makes
Ben earn more compared to the current sales and pricing structure as given in tab 2;
otherwise select Earn less.

Earn more Earn less

A Ben decreases the price of regular pizza to


$1.15 and leaves the prices of toppings the
same. His regular pizza sales increase to
100/week but toppings sales remain the same.

B Ben decreases the price of regular pizza to


$1.00 and increases the prices of each topping
by $0.25. His regular pizza sales increase to
120/week; his premium topping sales remain
the same but popular topping sales become
zero.

C Ben increases the price of regular pizza to $6


and increases the prices of each topping by
$0.25. His regular pizza sales decrease to
20/week, premium topping sales decrease to 5
and popular topping sales decrease to 10.

Solve yourself:

19992016 Manhattan Review 145 www.manhattanreview.com


146 Answer Key

www.manhattanreview.com 146 19992016 Manhattan Review


Chapter 8

Answer Key

147
148 Answer Key

Table Analysis

Q1 Q2 Q3 Q4

No, Yes, Yes No, No, No Yes, No, Yes Yes, Yes, No

Graphics Interpretation

Q1 Q2 Q3 Q4 Q5

B, D A, C D, D A, A B, B

2-Part Analysis

Q1 Q2 Q3 Q4 Q5 Q6 Q7

A, B A, F E, C E, D C, E D, A D, C

Multi-Source Reasoning

Q1 Q2 Q3 Q4 Q5 Q6

D No, Yes, No No, No, Yes E B More, More, Less

www.manhattanreview.com 148 19992016 Manhattan Review


Chapter 9

Solutions

149
150 Solutions: Table Analysis Practice Questions

9.1 Table Analysis

1. SolutionPart A

What does consistency mean? It means the data values are relatively close to each
other. You can measure the consistency using many parameters, such as range of data,
standard deviation, mean deviation, and others. The question asks you to measure the
consistency using the parameter, Range. Note that:

Range = Highest value in a dataset Lowest value in a dataset

To approach this question, you can sort the table either by column 2 (Grade), or by
column 5 (IQ). We prefer sorting by column 2 (Grade). Have a look at the image given
below.

Sorted by column 2
Col 1 Col 2 Col 3 Col 4 Col 5

Average Average
Intelligence
Months Grade maximum minimum
Quotient
Score Score

August X 66 58 116
December X 71 52 117
July X 71 55 115
November X 81 62 119 Range (X) = 121 115 = 6
October X 68 54 121
September X 72 52 121
August XII 68 55 120
December XII 70 49 114
July XII 72 45 116
Range (XII) = 120 114 = 6
November XII 69 51 118
October XII 69 45 120
September XII 75 56 114

It is clear from the sorted table that the range for both the grades are equal (Range = 6).
The correct answer to part A is No.

SolutionPart B

Deviation means the difference between two values. Here, the question wants you to
compare the greatest deviations between the average maximum score and the average
minimum score for both the gradesX and XII. Here,

Deviation = Average maximum score Average minimum score

It is not advisable to calculate the deviations for all the 12 data points. An efficient ap-
proach is to quickly scan through columns 4 and 5 and observe which of the differences
could be the highest. The deviation 72 45 = 27 for the month of July for grade XII may

www.manhattanreview.com 150 19992016 Manhattan Review


Solutions: Table Analysis Practice Questions 151

be the highest. The closest to this for grade X seems to be the deviation 72 52 = 20 for
the month of September, but it is smaller.
The correct answer to part B is Yes.

SolutionPart C

The mean average minimum score for August is for grades X and XII: (58 + 55)/2 = 56.5.
Similarly, the mean average minimum score for July is (45 + 55)/2 = 50, and that for
December is (52 + 49)/2 = 50.5.
Since 56.5 is greater than both 50 and 50.5, the statement is correct. Remember that
had the mean average minimum score for August been greater than that for either of the
other two months, and lesser than that for the other month, the answer would have been
No.
The correct answer to part C is Yes.

19992016 Manhattan Review 151 www.manhattanreview.com


152 Solutions: Table Analysis Practice Questions

2. SolutionPart A

The question wants you to find out whether or not the mean score for Math is greater
than its median score.
Calculating median is a comparatively easy task than calculating the mean as mean may
involve calculation. Hence, you should calculate the median first.

n+1 th
 
Median of Math scores = Value of the 2 students score,

where n is number of students


To get the values, sort the table by column 2 (Math). See the sorted table given below.
n+1
Note that if 2 is a fraction, take the average of the two values at the integer indices on
either side of the result.

Sorted by column 2
Col 1 Col 2 Col 3 Col 4 Col 5

Math Craft English Science


Students
(100) (25) (50) (100)
Sean 8 22 50 47
Marrie 10 20 6 45
Justin 12 18 5 35
Matt 54 22 45 39
Garysson 59 15 37 33
Median of Math
Joe 64 16
= (64+69)/2 31 31
Jack 69 7
= 66.5 41 27
Elad 74 5 49 84
Lucy 79 7 33 29
Angelo 84 9 35 35
Kevin 89 20 39 37
Mehul 94 13 43 41

According to the chart, the median score for Math is 66.5.


Now, calculate the mean score for Math.
To calculate the mean, you might think that you have to sum all the 12 scores and
divide the sum by 12. Despite having the luxury of an online calculator, this is a time-
consuming process. The GMAT never asks a question that consumes a disproportionately
high amount of time. So, there must be an alternate approach.
Observe that the last nine cells in column 2 have scores ranging from 54 to 94, in a
sequence with an interval of 5. The middle-most value is 74. You can use this to quickly
calculate the sum of all the scores: (8 + 10 + 12) + 74 9 = 696.
You should use the online calculator for this. The mean score: 696/12 = 58.
It is clear that the mean score, 58, is less than its median score of 66.5.
The correct answer to part A is No.

www.manhattanreview.com 152 19992016 Manhattan Review


Solutions: Table Analysis Practice Questions 153

SolutionPart B

A question asks whether it would affect the median scores for Math and for Science, if
Math and Science scores for Justine were swapped.
Currently, Justines Math and Science scores are 12 and 35, respectively. After swapping
them, they would become 35 and 12, respectively.
You have already calculated the median score for Math in Part A of the question: 66.5.
Since the wrong value (35) and the correct value (12) are less than the median value (66.5),
the median score for Math would not be impacted. More precisely, when the column 2
values are arranged in ascending order, both 12 and 35 would appear before the median
value of 66.5, irrespective of wether they are: 12 and 35 or 35 and 12.
Now, calculate the median score for Science.
First, sort the table by column 5 (Science), given below.

Sorted by column 5
Col 1 Col 2 Col 3 Col 4 Col 5
Math Craft English Science
Students
(100) (25) (50) (100)
Jack 69 7 41 27
Lucy 79 7 33 29
Joe 64 16 31 31
Garysson 59 15 37 33
Justin 12 18 5 35
Angelo 84 9 35 35
Median of Science
Kevin 89 20
= (35+37)/2 39 37
Matt 54 = 36
22 45 39
Mehul 94 13 43 41
Marrie 10 20 6 45
Sean 8 22 50 47
Elad 74 5 49 84

You can see in the chart that the median score for Science is 36. Again, both the wrong
and the correct values (12 and 35) are less than the median value 36. With the same
reasoning as used above, the median score for Science would not be impacted.
The correct answer to part B is No.
What if the median score for Math was impacted but that for Science was not? In that
case, the answer still would have been No. You will seldom find this kind of ambiguous
question in GMAT.
What if the same mixup had occurred with Jacks scores instead of Justines?
Jacks scores for Math and Science are 69 and 27, respectively. Well, in that case, the
median scores for Math as well as for Science would have been impacted.

19992016 Manhattan Review 153 www.manhattanreview.com


154 Solutions: Table Analysis Practice Questions

SolutionPart C

The question asks whether the median score for Science would increase by 2 points over
its current value, if scores of two new students, Suzi and McDonald, were included in the
table. Their scores for Science are 10 and 45, respectively.
You know that median is the middle-most element(s) of a set of elements sorted in as-
cending/descending order. Adding one element above and one below the median does
not change the middle-most element(s) of the list. Since you know the median score for
Science is 36, so by adding 10 (less than 36) and 45 (greater than 36) would not change
the median.
The correct answer to part C is No.
What if the Science scores for Suzi and McDonald were 40 and 45 respectively? Well, in
that case, the median score for science would have certainly increased by 2 points.

www.manhattanreview.com 154 19992016 Manhattan Review


Solutions: Table Analysis Practice Questions 155

3. SolutionPart A

Sorted by column 5
Col 4 Col 5
Minimum
Manager's comments The question asks whether there are two or more
sales per
on sales
month High-sales cars (Managers criterion) whose minimum
3 Average sales per month are less than the highest minimum
1 Average sales per month of a car reported either Average-sales,
5 Average Below-average-sales, or Low-sales.
6 Average
2 Average The best approach to sort the table by column 5 (Man-
6 Average agerss comments on sales).
11 Average
4 Below average
Scan through column 4 (Minimum sales per month)
8 High
in the chart and pinpoint the highest values for cars
11 High that reported Average-sales, Below-average-sales,
11 High or Low-sales. There are eight qualifying cells (seven
13 High Average-sales; one Below-average-sales) lying on the
12 High
top and two cells (two Low-sales) lying at the bottom.
10 High
The highest value among these 10 cells is 11.
15 High
10 High
Again, quickly run through the column 4 and find the
16 High
values less than 11 for the cars that reported High
12 High
sales. These values are 8, 10, and 10. So, three (at least
16 High
two) cars reporting High sales had minimum sales per
18 High
month lower than the highest minimum sales per month
20 High
of cars which did not have High sales.
19 High
0 Low
The correct answer to part A is Yes.
2 Low

19992016 Manhattan Review 155 www.manhattanreview.com


156 Solutions: Table Analysis Practice Questions

SolutionPart B
Sorted by column 3
Col 3 Col 4

Maximum sales Minimum sales This is a question on the concept of correlation.


per month per month Hopefully, you have already familiarized yourself with
the concept of correlation discussed in this book. Heres
6 0
a recap:
7 2
8 4
Two entities are positively correlated if both
9 3
of them either increase or decrease in tandem.
10 1
Two entities are negatively correlated if one
10 5
entity decreases in value when the other
10 6
increases in value. The two entities are not
11 2 correlated if the increase or the decrease in the
11 6 value of one entity cannot be related to the in-
16 8 crease or the decrease in the value of the other.
19 11
20 11 Sort the table by column 3 (Maximum sales per month).
20 13 The values will be arranged in ascending order or, in
21 11 other words, in an increasing trend.
21 12
21 10 Now observe how the other entity in column 4 (Minimum
sales per month) behaves. In the chart, the downward
21 15
arrows mark runs of values that are increasing, while the
21 10
arrows going up do so for values that are decreasing. As
22 16
there are enough upward arrows and enough downward
22 12
arrows, so, for these two entities, you cannot conclude
23 16 that they are perfectly correlated. You need to analyze
25 18 further.
26 20
27 19 Notice that the downward arrows are longer in length
than are upward arrows. This means that most values in
column 4 are increasing with the increases in column 3s
values, implying that they havethough not perfecta
strong positive correlation.

The correct answer to part B is No.

www.manhattanreview.com 156 19992016 Manhattan Review


Solutions: Table Analysis Practice Questions 157

SolutionPart C

Sorted by column 2
Col 2 Col 3
Maximum The question wants you to find out if the mean maxi-
Company sales per mum sales per month for Hyundai cars is greater than
month
the mean maximum sales per month for BMW cars.
BMW 6
BMW 8
BMW 9 Sort the table by column 2 (Company).
BMW 10
BMW 11 In the chart, there are six BMW cars and four Hyundai
BMW 22
cars.
Ford Motors 7
Ford Motors 10
Ford Motors 21
Observe that the highest value for Hyundai cars (27) is
General Motors 10 greater than that for BMW cars (22), and that all other
General Motors 21 values for Hyundai cars are greater than those for BMW
General Motors 22 cars. Hence, without calculation, you can conclude that
Hyundai 20 the mean maximum sales per month for Hyundai cars is
Hyundai 21
greater than that for BMW cars.
Hyundai 21
Hyundai 27
Nissan 11
The correct answer to part C is Yes.
Nissan 16
Nissan 19
Nissan 20
Nissan 21
Nissan 23
Nissan 25
Nissan 26

19992016 Manhattan Review 157 www.manhattanreview.com


158 Solutions: Table Analysis Practice Questions

4. SolutionPart A

The question wants you to find out whether the average scores per match for Sachins
teams are more than those for other teams (Teams that do not include Sachin).
To get the answer, sort the table by column 6 (Average score per match). This will make
sure that matches with higher average scores group in the bottom cells.

Sorted by column 6
Col 4 Col 5 Col 6
Average
Number of
score
Most Valuable Player matches
per
played
match
Mahela 22 121
Gayle 74 121
Kumara 24 124
Clarke 74 125
Kumara 77 128
Dhoni, Nannes 85 144
Clarke, Kumara 14 144
Mahela, Sehwag 81 145
Gambhir, Nannes 11 152
Malinga, Gayle 15 153
Clarke, Gayle 21 154
Gambhir 80 154
Sachin, Mahela 12 154
Sachin 26 155
Sachin, Malinga 83 165

Note that in column 4, the last three cells are occupied by Sachin, implying that each
team in which Sachin plays scored higher average score.
The correct answer for part A is Yes.

www.manhattanreview.com 158 19992016 Manhattan Review


Solutions: Table Analysis Practice Questions 159

SolutionPart B

The question wants you to find out whether it is likely that the teams that included
Clarke did not win a tournament.

Sorted by column 3
Col 3 Col 4 Col 5 Col 6 Col 7
Number of Average % of
Country Most Valuable Player matches score per matches
played match won
Australia Mahela 22 121 87
Australia Kumara 24 124 83
Australia Clarke, Gayle 21 154 43
Australia Sachin 26 155 46
India Gayle 74 121 45
India Clarke 74 125 42
India Kumara 77 128 46
India Dhoni, Nannes 85 144 46
India Mahela, Sehwag 81 145 54
India Gambhir 80 154 81
India Sachin, Malinga 83 165 83
Sri Lanka Clarke, Kumara 14 144 43
Sri Lanka Gambhir, Nannes 11 152 43
Sri Lanka Malinga, Gayle 15 153 47
Sri Lanka Sachin, Mahela 12 154 91

In the chart, you would observe that column 7 (% of matches won) indicates that none of
Clarkes teams have won more than 50% of their matches. Since the result of any match
is a win/loss (less than 50% win implies more loss), it is likely that no team that included
Clarke would have won the tournament.
The correct answer for part B is Yes.

SolutionPart C

The question wants you to find out whether the mean average score for the Australian
teams is more than the mean average score for the Sri Lankan teams.
To get this, sort the table by column 3 (Country). This will group rows of Australia and Sri
Lanka. Refer to the image given in SolutionPart B of the question. To actually calculate
the mean for each team, you would have to use eight data; however here is no need for
this, mere observation is sufficient. This question does not want a numerical answer. It
simply wants you to compare two values in terms of more or less.
Refer column 6, Sri Lankan teams have scored between 150-155 three times, and more
than 140 once, while Australian teams have scored 150-155 only two times, and the other
two scores being less than 125. You can thus deduce that the mean for Sri Lankan teams
would be more than that for Australian teams.
The correct answer for part C is No.

19992016 Manhattan Review 159 www.manhattanreview.com


160 Solutions: Graphics Interpretation Practice Questions

9.2 Graphics Interpretation

1. SolutionPart A

You have to calculate:

Total revenue from taxi in yr. 2006


Total revenue from bus in yr. 2006

Total revenue from taxi in yr. 2006 # of taxi commuters Taxi fare/passenger
=
Total revenue from bus in yr. 2006 # of bus commuters Bus fare/passenger

160M 30
20 : 1
320M 0.75

The correct answer for part A is option B.

SolutionPart B

You have to find out what the number of taxi passengers in the year 2010 would be if the
growth rate in the number of such passengers during 2009-2010 was the same as from
2008-2009. You have to calculate:
 
Number of passengers 2010 = Number of passengers 2009 1 + Growth rate 20082009

First calculate:
(Taxi commuters in 2009 Taxi commuters in 2008)
Growth rate 20082009 = 100%
Taxi commuters in 2008
225M 175M
= 100%
175M
200
%
7
So,

200 2
   

Number of passengers 2010 225M 1 +
% 225M 1 + 289.3M.
7 7

Do use the calculator.


The correct answer for part B is option D.

www.manhattanreview.com 160 19992016 Manhattan Review


Solutions: Graphics Interpretation Practice Questions 161

2. SolutionPart A

You have to calculate:

Number of Engineering undergrads Number of Management undergrads

In the pie chart, you will find that the percentage of Engineering undergrads and Man-
agement undergrads are 18% and 12%, respectively. So,

Number of Engineering Number of Management undergrads = 18% 12% = 6%

Had you known the value of total number of undergrads, you can get the answer. How-
ever, you dont really need to know it.

You already know that the number of the Art undergrads equals 6% of the total student
population, and since this is the number youre looking for, you dont need to calculate
it. Simply make use of number of Art undergrads, which is 126.

The correct answer to part A is option A.

SolutionPart B

After answering Part A, you might be tempted to simply calculate:

Number of Engineering Number of Science undergrads = 18% 12% = 6%,

as the answer, however 6% is wrong. What you really need to calculate is:

Number of Engineering undergrads Number of Science undergrads


100%
Number of Science undergrads

18% of total undergrads 12% of total undergrads


= 100%
12% of total undergrads

18% 12%
 
= 100% = 50%
12%

You need not know the value of total number of undergrads, as it cancels out.

Notice that the denominator is the number of Science undergrads (12% of total under-
grads) and not the number of Engineering undergrads (18% of total undergrads). Had you
mistakenly put the number of Engineering undergrads in the denominator, you would
have wrongly answered the questions as 33.33%.

The correct answer to part B is option C.

19992016 Manhattan Review 161 www.manhattanreview.com


162 Solutions: Graphics Interpretation Practice Questions

3. SolutionPart A

The question is pretty simple. It is a multiple-column chart presenting the percent dis-
tribution of household owners of seven electronic gadgets, for the years 2009 and 2012.
Look at the first part of the question.

During the three-year period, the maximum percentage increase in the pop-
ularity registered by an electronic goods equals %. (Assume that the
population of household owners was same for both the years).

The question wants you to find out two things:

(1) The goods that had the maximum percent growth from the year 2009 to 2012, and
(2) The percent growth for that goods during the period

Start with the first part.


The percent growth is calculated by:

Growth Value in 2012 Value in 2009


100% = 100%
Base value Value in 2009

Since the population of household owners was same in both the years, you can ignore its
effect.
Looking at the chart, you can infer that fridges, smartphones, and cameras had good
absolute growth in the years 2009 to 2012. So, the goods with maximum percent-growth
would be one among these.
Now, which goods among the three would be the answer? Would it be the one which
had maximum absolute growth in the years 2009 to 2012? The answer is, No, not
necessarily. The goods would be the one for which the ratio of

Growth
is the highest.
Value in 2009

To deduce that, the numerator (Growth) should be maximized and the denominator
(Value in 2009) should be minimized.
It is obvious that the answer would be Camera.
So, what is the value of percent growth for Camera?
We know that:

Value in 2012 Value in 2009 62 40 22


Percent growth = 100% = 100% = 100% 55%.
Value in 2009 40 40

The closest answer is option D.


Though the absolute growth for Fridge is also 22, its Percent growth is comparatively
less.
95 72 23
Percent growth = 100% = 100% 32%.
72 72

www.manhattanreview.com 162 19992016 Manhattan Review


Solutions: Graphics Interpretation Practice Questions 163

SolutionPart B

Look at the second part of the question.

If the number of owners increased from 606 million in the year 2009 to
1210 million in the year 2012, then the number of new fridges purchased be-
tween 2009 and 2012 is million. (Assume that no household owner has
sold/discarded a fridge during the period.)

Calculate:

# of new fridges = (% owners12 population12) (% owners09 population09)

= (95% 1210) (72% 606) 713M. (Use calculator)

The closest answer is option D.

19992016 Manhattan Review 163 www.manhattanreview.com


164 Solutions: Graphics Interpretation Practice Questions

4. SolutionPart A

To find the value of Rupee 5500 in Dinar, first you need to convert Rupees 5500 to Dollar
and then Dollar to Dinar since the Rupees-to-Dinar conversion is not given in the graph.
Beware that the secondary-axis is calibrated using solid grey lines, and the primary axis
is calibrated using dashed black lines.
From the graph, the Rupees-to-Dollar rate as on July 1 is 57. So, Rupees 5500 would
equal 5500/57 = $96.49. Again, Dollars-to-Dinar rate on July 1 is 3.5. Beware! You must
read the values from the secondary axis. Hence, Rupees 5500 or $96.49 = 96.49/3.5 =
Dinar 27.57. The closest answer in the options is Dinar 27.81.
The correct answer to part A is option A.

SolutionPart B

The person can buy the Mac if he has the money in a particular currency greater than or
equal to the cost of the Mac (Real 2400). Let us calculate the cost of the Mac in different
currencies.
On Sept 1, the Real-to-Dollar rate is 2.4. So, the cost of Mac in equivalent Dollars for Real
2400 equals:

2400/2.4 = $1000.

The man has $1100 ( more than $1000), so he can buy the Mac spending only Dollars.
Similarly, the Rupees-to-Dollar rate is 62. So, the cost of Mac in equivalent Rupees
equals

$1000 = Rupees (1000 62) = Rupees 62, 000.

The man has only Rupees 50,000 (less than Rupees 62000), so he cannot buy the Mac
spending only Rupees.
Similarly, the Dollar-to-Dinar rate is 3.4. So, the equivalent Dinar equals

$1000 = (1000/3.4) Dinar = 277.8 Dinar.

The man has only Dinar 250 (less than Dinar 277.8), so he cannot buy the Mac spending
only Dinar.
Hence, he can buy the Mac only using Dollars.
The correct answer to part B is option A.

www.manhattanreview.com 164 19992016 Manhattan Review


Solutions: Graphics Interpretation Practice Questions 165

5. SolutionPart A

From the second 100% stacked bar, you find that 25% of the Humanities students play
Tennis. From the first 100% stacked bar, you find that 60% of the students are belong to
Humanities majors.
This means that of the total students in the school, 25% of 60% = 15% play Tennis.
The correct answer is option B.

SolutionPart B

From the chart, you can see that 50% Humanities students who play soccer are Mexican.
Hence, you can deduce that the total number of soccer playing Humanities students is:
80
= 160. These 160 students are in fact 40% of the Humanities students. (Read the
50%
data from the second bar.) Hence, you can deduce that the total number of Humanities
160
students is: = 400.
40%
Again, from the first bar, you can see that 60% of the students in the school are Humani-
ties students.
Hence, you can deduce that the total number of Management students is

15%
 
400 = 100.
60%

The correct answer to part B is option B.

19992016 Manhattan Review 165 www.manhattanreview.com


166 Solutions: 2-Part Analysis Practice Questions

9.3 2-Part Analysis

Practice Questions (Quants)

1. SolutionBoth Parts

Understand the question. It is pretty simple. You have to find out the total fuel con-
sumption (gallons) and the total cost of fuel (dollars) for the entire journey of d miles.
The information given is the average speed (72 miles/hour), the fuel consumption per
hour (c gallons/hr.), and the cost of fuel ($3.60/gallon).
You can infer that this is a typical speed-time-distance question, with an addition infor-
mation about the cost of fuel.
First, you should always observe the given options, to short-list probable options or
eliminate a few non-probable options. However, here there is no option that you can
easily eliminate.
Determine the total fuel consumption. You can calculate the total fuel consumption (in
gallons) by multiplying the average fuel consumed per hour with the total hours of travel.

Total fuel consumption (gallons) = (Average fuel consumed/hour) (Total hours of travel)
= c (d/72) = cd/72 gallons

So, option A is the answer for the first part.


You calculate the total cost of fuel by:

Total cost of fuel (dollars) = Total fuel consumption (gallons) Cost of fuel
= (cd/72) 3.60 = $cd/20

So, option B is the answer for the second part.


The correct answers are options A and B.

www.manhattanreview.com 166 19992016 Manhattan Review


Solutions: 2-Part Analysis Practice Questions 167

2. SolutionBoth Parts

This is a question on arrangement; hence, you must apply permutation instead of com-
bination. Note that the number of people, n, must be less than or equal to the number
of chairs, r .
Given that Pnr = 56. Note that it is not Prn = 56.
By hit and trial, we can deduce that Pnr = 56 = 8.7 = P28 . The correct answers are n = 2
and r = 8. Beware that answering n = 8 and r = 2 would be wrong.
Finally, note that you must not solve this problem using combination, in which you would
use Cnr = 56. This gives r = 8 and n = 3, which are the wrong values.

The correct answers are options A and F.

3. SolutionBoth Parts

This is a question answered by applying the concept of continued proportion. You can
get the proportion of Alex : Betty : Cherry : Dirk by making a table such as following:

Alex Betty Cherry Dirk

800 800 120 = 680

1700 1700 200 = 1500

1200 1200 80 = 1120

Continued 1500 1120


800 680 1700 680 = 600 1200 600 = 560
Proportion

So, Alex : Betty : Cherry : Dirk :: 800 : 680 : 600 : 560.


This means that Alex will beat Cherry by 800 600 = 200 yards in an 800-yard race. In
other words, Alex can beat Cherry by 100 yards in a 400-yard race.
Similarly, Betty can beat Dirkby 680
 560 = 120 yards in a 680-yard race. In other
120
words, Betty will beat Dirk by 400 = 70 yards in a 400-yard race.
680
The correct answers are options E and C.
Make sure that you mark the answers correctly. Marking options C and E would be wrong.

19992016 Manhattan Review 167 www.manhattanreview.com


168 Solutions: 2-Part Analysis Practice Questions

4. SolutionBoth Parts

You know that:

Probability that an event will occur + Probability that an event will not occur = 1.

You can write it as p + q = 1, where

p = Probability of an even to occur

and
q = Probability of an even not to occur
You should first calculate q, because the questions will not be solved under only one
condition: when each of Mike, Garry, and Jack are not able to solve the questions. There-
fore,

Probability of not solving the questions

= Product of the probabilities that Mike could not solve,


Garry could not solve, and Jack could not solve
   
= 1 31 1 14 1 15 = 2
3 3
4 4
5 = 2
5

The correct answer for second part of the question is option D. Hence,

Probability of solving the questions = 1 Probability of not solving the questions


2
=1 5 = 35 .

The correct answer for first part of the question is option E.


The correct answers are options E and D.

www.manhattanreview.com 168 19992016 Manhattan Review


Solutions: 2-Part Analysis Practice Questions 169

5. SolutionBoth Parts

The question is pretty simple. You are given certain constraints on selecting the fifth
artist for each play.
The conditions are:

(A) Musicians 1
(B) Theatre artists 2
(C) Same nationality artists 2

Let us disciss each option one by one.

(A) Parker (Cinema artist, Adolescent, UK) cannot be selected for the first play, as he is
not a theatre artist. There must be at least two theatre artists in each play. He can,
however, be selected for the second play.
(B) Li Mao (Cinema artist, Teenager, China) cannot be selected for the first play, as he is
not a theatre artist. There must be at least two theatre artists in each play. He can,
however, be selected for the second play.
(C) Balki (Theatre artist, Singer, Argentina) cannot be selected for any play. He cannot
be selected for the first play, as he is a musician. There must not be two musicians
in any play. He also cannot be selected for the second play, as he is an Argentinean.
There must not be more than two Argentineans in any play. This is the correct
answer for Part A.
(D) Podolsky (Theatre artist, Musician, U.S.) cannot be selected for the first play, as he
is a musician. There must not be two musicians in any play. He can, however, be
selected for the second play.
(E) Willis (Theatre artist, Kid, New Zealand) can be selected for both the plays. This is
the correct answer for Part B.

The correct answers are options C and E.

19992016 Manhattan Review 169 www.manhattanreview.com


170 Solutions: 2-Part Analysis Practice Questions

Practice Questions (Verbal)

6. Claim: A diagnostic test measuring the levels of biomarker (a molecule in the blood-
stream) can predict and prevent suicide attempts.

Answer Choice Explanation

(A) This is the correct answer for the second part of the question. It clearly goes against
the argument. It raises doubt over the tests capability to measure such humanist
attributes as ethics, love, free will, and like. It can be inferred that biological predic-
tors will not be able to measure emotions, such as suicidal tendencies.
(B) Cultural factors are out of scope. It neither supports nor weakens the claim.
(C) How the discovery of this blood test should be viewed is irrelevant. There is no
logical reason to believe that it is pertinent to the claim. It neither supports nor
weakens the claim.
(D) This is the correct answer for the first part of the question. It argues that traditional
psychiatry falters at detecting suicidal tendencies, implying that this diagnostic test
would help gauge a subjects mental health.
(E) This option talks about emotional disorders in general and is not focused on sui-
cides. It neither supports nor weakens the claim.

Between options D and E, D is the best choice of one that supports the argument.
The correct answers are options D and A.

www.manhattanreview.com 170 19992016 Manhattan Review


Solutions: 2-Part Analysis Practice Questions 171

7. Conclusion: The larger harvest from the first farm must have been caused by the organic
fertilizer.

Predicting a Strengthener and a Weakener

Predictive Strengthener: A similar farm that treated its crop with organic fertilizer
harvested nearly 150 tons.
Predictive Weakener 1: There is a factor which aids in increasing groundnut harvest on
the first farm.
Predictive Weakener 2: There is a factor which impedes groundnut harvest on the
second farm.

Answer Choice Explanation

(A) The effects of synthetic fertilizers are outside the scope of the argument.
(B) The size of nuts is outside the scope of the argument. The argument is concerned
only about the size of harvests.
(C) This option is a weakener. Better soil properties may have aided the first farms
harvest. This option weakens the conclusion. This is the answer of the second part.
(D) This option is a strengthener. It explains why organic fertilizer is effective against
weeds, and thus helped the harvest. It strengthens the conclusion. This is the
answer of the first part.
(E) Weed harvest and its commercial value are outside the scope of the argument.

The correct answers are options D and C.

19992016 Manhattan Review 171 www.manhattanreview.com


172 Solutions: Multi-Source Reasoning Practice Questions

9.4 Multi-Source Reasoning

Questions 13

1. Refer to the last sentence in Tab 2, where the director states, We are considering all our
options and, as always, welcome any input you may have. His intent is to expand the
business. There is no specific plan laid down so far. He will come up with one once he
discusses with the president.
The correct answer is option D.

2. Part A:
No, that is the opinion of the Franchise Advisor.

The correct answer is No.

Part B:
Yes. He states, However, our research further suggests that those five additional
franchises will fully saturate the Indian market. . .

The correct answer is Yes.

Part C:
You cannot infer this. The success of franchises is talked about with reference to
the Indian market only.

The correct answer is No.

3. Part A:
Both the director and franchising consultant only discuss the existing 10 franchises,
but the Indian consultant explicitly states that the Indian market has room for an
additional five franchises.

The correct answer is No.

Part B:
While the franchising advisor assumes that expansion must occur through fran-
chising, the director only states, We are considering all of our options. . .

The correct answer is No.

www.manhattanreview.com 172 19992016 Manhattan Review


Solutions: Multi-Source Reasoning Practice Questions 173

Part C:
All three stakeholdersthe director, the franchise advisor, and the Indian
consultantexplicitly express their opinion about growing outside India market.

The correct answer is Yes.

19992016 Manhattan Review 173 www.manhattanreview.com


174 Solutions: Multi-Source Reasoning Practice Questions

Questions 46

4. A week-wise distribution of 700 pizzas in four weeks is not given in the question. As for
Bens gross sales, he is going to earn the same despite selling more pizzas in any week(s)
and fewer in other week(s).
However, a week-wise distribution of 700 units would impact his incentives from S-Mart.
Consider some possible scenarios of Ben selling 700 pizzas in four weeks:

(W 1, W 2, W 3, W 4) : (0, 0, 0, 700); (200, 200, 200, 100); (100, 100, 200, 300); (0, 200, 200, 300).

There could be many other scenarios too. Since the maximum incentive by S-Mart is
when Ben sells 500 or more pizza in a week. To maximize his incentives, Ben could sell
in the following way:
(W 1, W 2, W 3, W 4) : (500, 200, 0, 0)
That would earn him 100 + 20 + 0 + 0 = $120, whereas if he had sold the following way:

(W 1, W 2, W 3, W 4) : (99, 99, 251, 251)

Bens incentive would be minimum, and he will earn 0 + 0 + 20 + 20 = $40. The difference
is $120 $40 = $80.
The correct answer is option E.

5. Ben will start making a profit starting the week he fully recovers his investment. In other
words, when his sales exceed his investment, he will start to make a profit.
Say, he starts making profit in x th week, then:

Investment = 1750 + 20x

Sales = Sales from pizzas, premium toppings, and popular toppings, + Incentives

= (150 3 x + 10 0.75 x + 5 0.50 x) + 10 x

So, if

(1750 + 20x) < (150 3x + 10 0.75x + 5 0.50x + 10x), Ben makes profit.

Solving above, we get x > 3.88. Since x > 3.88 weeks, Ben will make profit from the
fourth week onwards.
The correct answer is option B.

www.manhattanreview.com 174 19992016 Manhattan Review


Solutions: Multi-Source Reasoning Practice Questions 175

6. First calculate Bens current earning. It is 3 40 + 0.75 10 + 0.50 15 = $135.

Part A:
Earning in this scenario would be 1.15 100 + 0.75 10 + 0.50 15 = $130. If you
think the earnings would decrease, you missed the incentive part. Ben is eligible
for a $10 incentive, because he sold 100 pizzas in a week. So his earnings are
130 + 10 = $140.

The answer is: Ben earns more ($140 > $135).

Part B:
Earning in this scenario would be [1 120 + (0.75 + 0.25) 10 + (0.50 + 0.25)
0] + 10 = $140. Ben is eligible for a $10 incentive, because he sold 100 pizzas in a
week.

The answer is: Ben earns more ($140 > $135).

Part C:
Earning in this scenario would be 6.20 + 1.5 + 0.75 10 = $132.50.

The answer is: Ben earns less ($132.50 < $135).

19992016 Manhattan Review 175 www.manhattanreview.com


176 Solutions: Multi-Source Reasoning Practice Questions

www.manhattanreview.com 176 19992016 Manhattan Review


Chapter 10

Talk to Us

Email us if you want us to send you an Excel file with 10 tables of Table Analysis questions.
We will send you filter-enabled tables so you can practice in a computer-enabled environment.
Email us in the following format:

To: [email protected]

Subject: TA tables

Hi,
Please mail me Excel spreadsheets of TA tables.

hYour namei
hYour contact numberi

Have a Question?

Email your questions to [email protected]. We will be happy to answer you. Your


questions can be related to a concept, an application of a concept, an explanation of a question,
a suggestion for an alternate approach, or anything else you wish to ask regarding the GMAT.
Please mention the page number when quoting from the book.

Best of luck!

Prof. Dr. Joern Meissner


& The Manhattan Review Team

177

You are a unique candidate with unique experience.
We help you to sell your story to the admissions committee.

Manhattan Admissions is an educational consulting firm that


guides academic candidates through the complex process of
applying to the worlds top educational programs. We work
with applicants from around the world to ensure that they
represent their personal advantages and strength well and get
our clients admitted to the worlds best business schools, gra-
duate programs and colleges.

We will guide you through the whole admissions process:

Personal Assessment and School Selection


Definition of your Application Strategy
Help in Structuring your Application Essays
Unlimited Rounds of Improvement
Letter of Recommendation Advice
Interview Preparation and Mock Sessions
Scholarship Consulting

To schedule a free 30-minute consulting and candidacy evalua-


tion session or read more about our services, please visit or call:

www.manhattanadmissions.com +1.212.334.2500

About the Turbocharge Your GMAT Series (6th Edition)
The Turbocharge Your GMAT Series is carefully designed accessible manner. We delve deeply into the content of
to be clear, comprehensive, and content-driven. Long every single testing area and zero in on exactly what you
regarded as the gold standard in GMAT prep worldwide, need to know to raise your score. The full series is
Manhattan Reviews GMAT prep books offer professional comprised of 16 guides that cover concepts in mathemat-
GMAT instruction for dramatic score improvement. Now in ics and grammar from the most basic through the most
its updated 6th edition, the full series is designed to advanced levels, making them a great study resource for
provide GMAT test-takers with complete guidance for all stages of GMAT preparation. Students who work
highly successful outcomes. As many students have through all of our books benefit from a substantial boost
discovered, Manhattan Review's GMAT books break down to their GMAT knowledge and develop a thorough and
the different test sections in a coherent, concise, and strategic approach to taking the GMAT.

Turbocharge Your GMAT Series


About Manhattan Review About the Author


Manhattan Reviews origin can be traced directly back to Professor Dr. Joern Meissner has more than 25 years of
an Ivy League MBA classroom in 1999. While teaching teaching experience at the graduate and undergraduate
advanced quantitative subjects to MBAs at Columbia levels. He is the founder of Manhattan Review, a world-
Business School in New York City, Professor Dr. Joern wide leader in test prep services, and he created the origi-
Meissner developed a reputation for explaining compli- nal lectures for its first GMAT preparation classes. Prof.
cated concepts in an understandable way. Remembering Meissner is a graduate of Columbia Business School in
their own less-than-optimal experiences preparing for New York City, where he received a PhD in Management
the GMAT, Prof. Meissner's students challenged him to Science. He has since served on the faculties of prestigious
assist their friends, who were frustrated with conventional business schools in the United Kingdom and Germany. He
GMAT preparation options. In response, Prof. Meissner is a recognized authority in the areas of supply chain man-
created original lectures that focused on presenting GMAT agement, logistics, and pricing strategy. Prof. Meissner
content in a simplified and intelligible manner, a method thoroughly enjoys his research, but he believes that
vastly different from the voluminous memorization and grasping an idea is only half of the fun. Conveying knowl-
so-called tricks commonly offered by others. The new edge to others is even more fulfilling. This philosophy was
approach immediately proved highly popular with GMAT crucial to the establishment of Manhattan Review, and
students, inspiring the birth of Manhattan Review. remains its most cherished principle.

Since its founding, Manhattan Review has grown into a www.manhattanreview.com


multi-national educational services firm, focusing on [email protected]
+1 (212) 316 -2000
GMAT preparation, MBA admissions consulting, and appli- +1 (800) 246-4600
cation advisory services, with thousands of highly satis-
Manhattan Review, 275 Madison Avenue, Suite 1429, New York, NY 10016.
fied students all over the world. The original lectures have
been continuously expanded and updated by the Manhat-
tan Review team, an enthusiastic group of master GMAT
professionals and senior academics. Our team ensures
that Manhattan Review offers the most time-efficient and
cost-effective preparation available for the GMAT. Please
visit www.ManhattanReview.com for further details.

You might also like